Está en la página 1de 176

iii MAT3711/1

Contents

Page
Preface v

Chapter 0 Preliminaries 1
0.1 Notation 1
0.2 Real numbers 1
0.3 Denseness of Q in R 6
0.4 Cardinality 7

Chapter 1 Metric Spaces 17


1.1 Metric spaces 17
1.2 Open sets and interior 19
1.3 Closed sets and closure 30
1.4 Bounded sets, diameters and distances 38
1.5 The Bolzano-Weierstrass Theorem 41

Chapter 2 Convergence and Completeness 45


2.1 Sequences and subsequences 45
2.2 Cauchy sequences and completeness 50
2.3 Sequences with terms in R 55

Chapter 3 Compactness 61
3.1 Compact sets 61
3.2 Characterisations of compactness 65

Chapter 4 Connectedness 75
4.1 Connected sets 75
iv

Chapter 5 Continuity 81
5.1 Continuous functions 81
5.2 Uniform continuity 85
5.3 Continuity and compactness 88
5.4 Continuity and connectedness 91
5.5 Contraction mappings 94

Chapter 6 Function Spaces 99


6.1 Spaces of bounded functions 99
6.2 Pointwise and uniform convergence 105
6.3 Miscellaneous examples 115

Chapter 7 Linear Analysis 121


7.1 Basic inequalities 121
7.2 Normed vector spaces 129
7.3 Bounded linear operators 135

Chapter 8 The Riemann-Stieltjes Integral 149


8.1 The integral 149
8.2 The Fundamental Theorem of Calculus 165
8.3 Integration and convergence 169
8.4 Improper integrals 171
v MAT3711/1

Preface

1. About the Study Guide


This study guide is a self-contained treatment of a standard syllabus on Real Analysis as cur-
rently taught to undergraduate students in South Africa. In compiling it I have assumed famili-
arity with elementary Real Analysis (MAT213-T) and rudiments of Linear Algebra.

All concepts discussed herein have been carefully defined. A few exceptions (such as subset",
element of", dierentiable function", etc.) are things you have learnt at first and second year
levels.

The notation I have used is fairly standard. In particular

(a) If A and B are sets, A B means A is a subset of B.

(b) If A and B are sets, A\B = {x | x A but x B}.

(c) If f : X Y is a function and A X, B Y , then

f [A] = {f (x) | x A}, the image of A under f

f 1 [B] = {x X | f (x) B}, the pre-image of B under f .

It should be noted that in some books f [A] and f 1 [B] are denoted by f (A) and f 1 (B)
respectively.
V
(d) If C is a collection of sets, C denotes the union of all the sets in C.

(e) The set of all x that satisfy some property P is denoted by

{x | x satisfies P } or {x : x satisfies P },

whichever is convenient.
vi

2. About mastering this course


The first step of course, as with all branches of mathematics, is to understand clearly what each
definition says. Definitions are the building blocks for this course.

Real Analysis is typical of the Definition-Theorem-Proof type of mathematics. Indeed, in most


instances, after defining a particular concept I have given some examples of objects with pro-
perties provided by the definition. Where it is illustrative to do so, I have also given examples
of objects that do not satisfy a particular definition.

Having fully grasped a definition (or a number of definitions) you will encounter a theorem and
its proof. Once again you must study the statement of a theorem thoroughly and make sure
you understand it before studying the proof. Some proofs are fairly routine and easy to follow,
others are quite involved. In all instances make sure you see why a particular statement follows
from what has gone before it.

Apropos of proofs, there are cases where you will need to fill in the gaps. Some such cases are
where a proof (or part of a proof) is stated to be an exercise; others are instances where it is
stated it is easy to see that ..." or one checks easily that ...".

Exercises are part of the course. There are results which are stated in the exercises and used in
the development of the theory or in proofs of certain theorems. You must attempt all exercises.

Themba Dube
Unisa, April 2009.
1 MAT3711/1

Chapter 0

Preliminaries
We commence with properties of real numbers that are relevant for this module. Some of
these properties will be familiar to you from MAT2613 or MAT2615. Throughout the study
guide we shall use the following notation pertaining certain subsets of the set of real numbers.

0.1 Notation
N = {1, 2, 3, . . .} = the set of natural numbers
Z = {. . . 3, 2, 1, 0, 1, 2, . . .} = the set of integers
m
Q= n | m, n Z, n = 0 = the set of rational numbers
R = {x | x is a real number}

0.2 Real numbers


The real number system R is an ordered field which has the least-upper-bound property. What
do we mean by this? We need to explain the terms field", ordered" and has the least upper-
bound property".

To say R is a field means R is equipped with two operations + : R R R (called addition)


and : R R R (called multiplication) such that the following axioms hold:

A1. For any x, y R, x + y R;

A2. x + y = y + x for all x, y R;

A3. x + (y + z) = (x + y) + z for all x, y, z R;

A4. There exists a unique element 0 R (called zero) such that x + 0 = x for all x R;

A5. For every x R there exists a unique element y R such that x + y = 0. [We denote such
an element by x];

M1. For any x, y R, x y R. [We write xy in place of x y];

M2. xy = yx for all x, y R;


2

M3. x (yz) = (xy) z for all x, y, z R;

M4. x (y + z) = xy + xz for all x, y, z R;

M5. There exists a unique element 1 R such that x 1 = x for all x R;

1
M6. For each nonzero x R there exists a unique element (denoted by x or x1 ) such that
1
x x = 1.

To say R is ordered means R is equipped with an order relation < which satisfies the following
axioms:

O1. For all x, y R, either x < y, y < x or x = y;

O2. If x < y,then x + z < y + z for all z R;

O3. If x < y and 0 < z, then xz < yz;

O4. x < y and y < z x < z for all x, y, z R.

If x < y we also write y > x. The relation x y (also written as y x) is defined by:
x y x < y or x = y.

NOTES:

(a) Given x, y R; either x y, or x y (prove this).

(b) The elements 0, 1 are distinct; i.e. 0 = 1.

Exercise 0.2.1 Show that for every x, y R

(a) x 0 = 0

(b) x = (1) x

(c) (x) = x

(d) (x + y) = (x) + (y)

(e) (1) (1) = 1

(f) xy = 0 x = 0 or y = 0

(g) (x) (y) = xy


3 MAT3711/1

Theorem 0.2.2 If a R and a = 0 then a2 > 0. Hence 1 > 0.

Proof: Either a > 0 or a < 0. If a > 0, then by O3 and the exercise above, a a > a 0,
i.e. a2 > 0. If a < 0 then a > 0 (verify this), so that (a) (a) > 0 where a2 > 0. Now
1 = 1 1 = 12 ; so 1 > 0. 

The theorem above shows that some of the properties of real numbers that we have always"
known (and even taken for granted) can actually be deduced from a carefully selected set of
axioms that characterise the real number system. Note that we have not yet explained what we
mean by least-upper-bound property". Before we do that, let us prove another property of the
reals that is frequently useful.

Theorem 0.2.3 Let a, b R. If a b + for every > 0, then a b.

Proof: We prove the result by contradiction. So suppose, by way of contradiction, that a b.


1 1
Then b < a, which implies that a b > 0. So 2 (a b) > 0. Now put = 2 (a b) in the
statement of the theorem to obtain
ab
a b+
2
a+b
=
2
a+a
<
2
= a.

We get the absurdity that says a < a. 

Next we prepare the ground for the explanation of the last property we stated for R.

Definition 0.2.4 Let S be a nonempty subset of R. We say S is

bounded above in case b R such that x b x S;

bounded below in case c R such that c x x S;

bounded if S is bounded above and below.

Thus a nonempty subset S of R is bounded above (resp. below) if there is a real number which
is bigger than or equal to (resp. smaller than or equal to) all elements of the set S. Such a real
number is called an upper bound (resp. lower bound) for the set S.
4

Quite clearly, if b is an upper bound for S, then any real number bigger than b is also an upper
bound for S. Likewise, any number smaller than a lower bound is itself a lower bound. Among
upper (resp. lower) bounds, we are interested in the smallest (resp. largest). It is given a special
name mentioned in the following definition.

Definition 0.2.5 Let S be a nonempty subset of R which is bounded above. A real number is
called the supremum of S, denoted = sup S, if

is an upper bound for S; and

for any upper bound b of S; b.

Thus, the supremum of a set is an upper bound which is smaller than any other upper bound.
It is for that reason that if = sup S we also say is the least upper bound (abbreviated
l.u.b.) of S.

On the other hand we have the following definition.

Definition 0.2.6 Let S be a nonempty subset of R which is bounded below. A real number is
called the infimum of S, denoted = inf S, if

is a lower bound for S; and

for any lower bound d of S, d.

The infimum of a set S is also called the greatest lower bound (abbreviated g.l.b.) of S.

CAUTION: There is nothing in the definition that says the supremum (or infimum) of a set
must be an element of the set. It may or may not belong to the set. If sup S S then in fact
sup S is the largest element of S. In this case we say sup S is the maximum element of S and
(frequently) write max S in place of sup S. Similarly, if inf S S we say it is the minimum
element of S and may write min S instead of inf S.

CAUTION: You should never write min S (or max S) if the infimum (or supremum) is not in
S.
5 MAT3711/1

Theorem 0.2.7

(a) = sup S (i) x x S; and

(ii) a < , s S such that a < s .

(b) = inf S (i) x x S; and

(ii) b > , w S such that w < b.

Proof: Exercise. 

The following axiom characterizes R.

Least-upper-bound property. Every nonempty subset of R that is bounded above has the
supremum.

What the l.u.b. property says is that if S R is nonempty and bounded above, then there is a
real number such that = sup S. In order to fully appreciate the power of this axiom, let us
consider a situation where we pretend the only numbers we know are rational numbers. Then
the concepts of bounded above and supremum are definable in Q exactly as we did in R.

Now Q does not have the l.u.b. property. Indeed if we let

S = x Q | x2 < 2

then S is a nonempty subset of Q which is bounded above. However there is no rational number

r such that r = sup S. However working in R, we have that sup S = 2.

Remark: The l.u.b. property is also known as the Compleness Axiom for R.

Exercise 0.2.8

1 1
(a) Let S = m n | m, n N . Show that S is bounded and find sup S and inf S.

(b) Let S, T be subsets of R that are bounded above. Put R = {x + y | x S, y T } . Show


that R is bounded above and express sup R in terms of sup S and sup T.

(c) Let S, T R with S bounded above and T bounded below. Put R = {s t | s S, t T }.


Prove that R is bounded above and sup R = sup S inf T.
6

(d) Let A R be bounded below. Show that = inf A is a lower bound of A and
> 0, a A s.t. a < + .

0.3 Denseness of Q in R
In this section our main goal is to prove a very important result which states that between any
two real numbers there is a rational number.

Theorem 0.3.1 The set N is not bounded above.

Proof: Assume N is bounded above. Then by the Compleness Axiom, sup N exists. Put
= sup N. Since 1 < , 1 is not an upper bound for N, so there exists k N
such that 1 < k. This implies that < k + 1. But k N k + 1 N. So k + 1 since
is an upper bound for N. Thus < k + 1 , which is absurd. 

We deduce from the theorem above that given any real number r, there is a natural number m
such that m > r. Likewise, given any real number t, there is an integer n such that n < t. (Show
this.)

Corollary 0.3.2 Given a real number r, there exists an integer k such that r 1 k < r.

Proof: Find an integer m such that m < r; and a natural number n such that n > r. Thus
m < r < n. Notice that, since m < r and r < n, in the set {m, m + 1, . . . , n} there is the largest
integer k such that k < r. So k + 1 r and therefore r 1 k < r. 

Corollary 0.3.3 Let A be a real number and > 0. Then there exists n N such that n > A.
(Informally: no matter how large A is and how small is (as long as > 0) we can add enough
s to obtain a result larger than A).

Proof: If not, then n A


for every n N, contrary to the fact that N is not bounded above. 

1
We note in particular that for every > 0 there exists n N such that n < . This, of course,
is familiar to you in the form lim 1 = 0.
n n

We are now in a position to state the main result.


7 MAT3711/1

Theorem 0.3.4 Let x, y R with x < y. Then there is a rational number r such that x < r < y.

1
Proof: Choose n N such that n < y x. Then

1
x<y .
n

By Corollary 0.3.2 we can find an integer k such that

1 k
ny 1 k < ny, i.e. y < y.
n n

Combining these results we obtain

1 k
x<y <y
n n

which implies that x < k


n < y. 

Corollary 0.3.5 Let b R and S = {x Q | x < b} . Then sup S = b.

Proof: Clearly, S is bounded above and so has a supremum, say c. Then c b since b is an upper
bound for S. If we assume that c < b, then there is a rational number r such that c < r < b. So r
is a member of S that is strictly larger than sup S. That is not possible. So c b, hence c = b. 

Now let us go back to the case S = x Q | x2 < 2 mentioned earlier. Note that

x2 < 2 2 < x < 2.

So, by the foregoing corollary, we have that sup S = 2.

0.4 Cardinality
Our treatment of this topic will be rather superficial; the main purpose begin to establish that:

(i) Any interval (a, b), with a < b, has as many elements" as R;

(ii) Between any two real numbers there is an irrational number.

By a finite set we mean an empty set or a set that has n elements for some n N. Now if S and
T are finite sets with m and n elements respectively, then it is clear that there is a one-to-one
function f : S T mapping S onto T if and only if m = n. Generalising to arbitrary sets
(including those with infinitely many elements) it is natural to think of two sets as having the
8

same number of elements provided there is a one-to-one onto function between them.

We caution that this may go against intuition in the sense that a proper subset of a set can have
the same number of elements" as the set. Witness the following example.

Example 0.4.1 Let S be the set of all even natural numbers. Then S is a proper subset of N.
The function f : N S given by f (n) = 2n is one-to-one onto.

Clearly, then, in the case of infinite sets, to say two sets have the same number of elements"
might be problematic. We therefore speak of sets having the same cardinal number (or car-
dinality), where, in the case of finite sets, the cardinality of a set is simply the number of its
elements. We shall not define precisely what a cardinal number is; that is a matter normally
discussed in a course on Set Theory.

Definition 0.4.2 Let S be a set. A symbol, called the cardinal number of S, denoted
card(S) or |S|, is associated with S in such a way that card(X) = card(Y ) if and only if there
is a one-to-one onto function f : X Y. If S is a finite set with m elements, we set card(S) = m.

As special numbers we mention card(N) which is normally denoted by 0 (the symbol is a


Hebrew letter pronounced aleph), and card(R) which we denote by .

Next we define what it means to say one cardinal number is smaller (or smaller than or equal
to) another. Once again the motivation comes from finite sets. Note that if S and T are fi-
nite sets with card(S) card(T ), then there is a one-to-one function g : S T. Conversely,
if there is a one-to-one function g : S T, then S cannot have more elements than T, i.e.
card(S) card(T ) .
NOTE: The function g mentioned is only assumed to be one-to-one but not necessarily onto.

Definition 0.4.3 If X and Y are sets, we say card(X) is less than or equal to card(Y ), and
write card(X) card(Y ), in case there is a one-to-one function g : X Y. If

card (X) card (Y ) , but card (X) = card (Y ) ,

we write card (X) < card(Y ) .


9 MAT3711/1

Exercise 0.4.4 Prove that card(X) card(Y ) there exists Z Y such that card(X) =
card(Z).

Example 0.4.5 Let S be a set and T S. The map i : T S given by i (x) = x is one-to-one.
Thus card(T ) card(S) . So every subset has cardinality less than or equal to the cardinality
of the containing set.

The relation defined for cardinal numbers above is easily seen to be:

reflexive, i.e. for every cardinal number . Indeed if X is a set with card(X) = ,
then the map i : X X given by i (x) = x is one-to-one.

transitive, i.e. implies . Take sets X, Y and Z with card(X) = ,


card(Y ) = and card(Z) = . Let f : X Y and g : Y Z be one-to-one functions.
Then g f : X Z is one-to-one.

It is true that is also antisymmetric, i.e. and together imply that = . The
proof of this is not trivial; and the result is known as the Cantor-Bernstein theorem.

Theorem 0.4.6 Let A and B be sets with card (A) card(B) and card(B) card(A). Then
card(A) = card(B).

Proof: Let us denote set dierence by S\T ; i.e. S\T = {x S | x T }. Now we are given that
there is a one-to-one function f : A B and a one-to-one function g : B A. We must show
that there is a one-to-one onto function : A B.
For any S A let K (X) denote the set

K (X) = A\g [B\f [S]] ,

where f [X] = {f (x) | x X} for X A. Now note that if U V A then K (U ) K (V )


since U V implies f [U ] f [V ] which in turn implies B\f [U ] B\f [V ] . Next let

A = {X A | X K (X)}

and put C = A, i.e. C is the union of all sets in A. For any c C there exists X A
such that c X K (X) K (C) . [Note that K (X) K (C) since X C]. Since c is an
arbitrary element of C we conclude that C K (C) ; which in turn yields K (C) K (K (C)) .
Thus K (C) A, and therefore by the definition of C we have K (C) C. This then shows
10

that K (C) = C. The function g is one-to-one; so its restriction g on B\K [C] is also one-to-
one. We claim that g maps B\f [C] onto A\C. To see this let y A\C. Then y
/ C, i.e.
y
/ K [C] = A\g [B\f [C]] . So then it follows that y g [B\f [C]], and hence y = g (x) for some
x B\f [C] . Now define : A B by

f (x) , if x C
(x) =
g 1 (x) , if x A\C

Then is a one-to-one onto function. That it is one-to-one is clear. To see ontoness, notice that
B = (B\f [C]) f [C] and f is onto f [C] whilst g is onto B\f [C] . 

In fact it can be shown (we will not prove it see any book on advanced Set Theory) that any
two cardinal numbers are comparable. More precisely we have the following theorem.

Theorem 0.4.7 Given any two sets X and Y either card(X) card(Y ) or card(Y ) card (X).

Definition 0.4.8 A set X is said to be countably infinite in case card(X) = card(N); i.e. if
it has the same cardinality as the set of natural numbers. A set is said to be countable if it is
either finite or countably infinite. A set is uncountable if it is not countable.

Let us examine closer what it means for a set X to be countably infinite. In such a case there is
a one-to-one onto function f : N X. For each k N let xk denote f (k) . Thus the elements of
X are precisely x1 , x2 , x3 , . . . ; i.e. X = {x1 , x2 , x3 , . . .} . So then the elements of X can be writ-
ten out with labels" 1, 2, . . . by means of which we can count" all the elements of X without
missing any. Of course if Y is a finite set, say with m elements, then the elements of Y can be
written out as y1 , y2 , . . . , ym . Thus, whether a set is finite or countably infinite, we can count"
its elements; hence the term countable".

There are sets which are not countable. Before we give examples we first prove a result which
tells us that the set of all subsets of any given set S has cardinality strictly bigger than the
cardinality of S. Recall that the set of all subsets of S is denoted by P (S).

Theorem 0.4.9 For every set S, card(S) < card(P (S)) .

Proof: The function j : S P (S) given by j (x) = {x} is one-to-one. So card(S) card(P (S)) .
So we must show that card(S) = card(P (S)). Suppose, by way of contradiction, that there is a
11 MAT3711/1

one-to-one onto function g : S P (S) . Let A = {x S | x


/ g (x)} . Then A P (S), so there
exists s S such that g (s) = A because g is onto. Now either s A or s
/ A. A contradiction
arises as follows: if we suppose that s A, then by the definition of A we have that s
/ g (s),
i.e. s
/ A. On the other hand, if we suppose that s
/ A, then we must have s g (s) , i.e. s A.
This contradiction proves that there cannot be a one-to-one onto function from S onto P (S) .
Thus card(S) < card(P (S)). 

Example 0.4.10 The foregoing result gives card(N) < card(P (N)). Therefore the set of all
subsets of N is not countable.

Example 0.4.11 The interval (0, 1) = {x R | 0 < x < 1} is uncountable. To prove this we
suppose, on the contrary, that (0, 1) is countable. Then (0, 1) = {s1 , s2 , . . .}. We then show that
this is impossible by constructing a real number x (0, 1) which cannot be one of the numbers
s1 , s2 , . . .. Write each sn as an infinite decimal sn = 0 un1 un2 un3 . . . where each uni is 0, 1, . . .,
1 2
or 9. (For instance 3 = 0.333 . . ., 5 = 0.4000 . . .). Now define x to be 0 t1 t2 t3 . . . where

3 if u = 1
nn
tn =
2 if unn = 1

Now notice that 0 < x < 1 and that for each n, x = sn since the decimal expansion of x diers
from that of sn at the nth decimal place. (A situation like sn = 0.19999 . . . and x = 0.2000 . . .
cannot occur here because each tn is neither 0 nor 9).

In the next series of results we want to determine the countability or otherwise of the sets Q, R
and R\Q. We start with R. The following theorem is useful in this regard.

Theorem 0.4.12 Every subset of a countable set is countable.

Proof: Let S be a countable set and T S. If T is finite then T is countable. So suppose T is


countably infinite, which of course means S is also countably infinite. Write S = {s1 , s2 , . . .} .
Now define k : N N as follows:
k (1) is the smallest positive integer m such that sm T. Having defined k (1) , . . . , k (n 1) ,
define k (n) to be the smallest positive integer m > k (n 1) such that sm T. (For instance
if T = {s7 , s22 , s59 , . . .} then k (1) = 7, k (2) = 22, k (3) = 59, etc.) Now let : N S be
given by (n) = sn . Note that both and k are one-to-one. Therefore the composite function
k : N T is one-to-one onto (check this). This completes the proof. 
12

We note immediately from the foregoing theorem that if A is an uncountable subset of B, then
B is also uncountable. Now having shown that (0, 1) is uncountable, we deduce that:

Theorem 0.4.13 R is uncountable.

Actually we can do better than that. We shall show that for any real numbers a and b with
a < b, the interval (a, b) has the same cardinality as R. Let a, b R with a < b. Define

bt
: (a, b) (0, 1) by (t) = .
ba

One checks easily that is one-to-one onto. So card(0, 1) = card(a, b) . Now define

t
: R (1, 1) by (t) = .
1 + |t|

Again it is not dicult to verify that is one-to-one onto. Consequently we have:

card (R) = card (1, 1) = card (a, b) for all a, b R with a < b.

Now let I denote any of the intervals [a, b] , [a, b) or (a, b]. Then I R; so that

card (I) card (a, b) .

So by Theorem 0.4.6 we have:

Theorem 0.4.14 For any real numbers a and b with a < b, we have

card (R) = card (a, b) = card(a, b] = card[a, b) = card[a, b].

Next we turn our attention to rational numbers.

Lemma 0.4.15 Let A be a set and S be a countable set. If there is a one-to-one function
g : A S mapping A into S (note that we are not assuming g to be onto), then A is countable.

Proof: Note that f [A] is countable since it is a subset of a countable set. Now f : A f [A] is
one-to-one onto. Thus A is countable. 
13 MAT3711/1

Theorem 0.4.16 The product set N N is countable.

Proof: Let f : N N N be given by f (m, n) = 2m 3n . Then f is one-to-one as can easily be


checked. So in view of Lemma 0.4.15 the result follows. 

Let X1 , X2 , . . . be nonempty sets. For each n N let Xn = {(x, n) | x Xn } . We therefore


have

X1 = {(x, 1) | x X1 }
X2 = {(x, 2) | x X2 } , etc.

Notice that for any k, m N with k = m, Xk Xm = because the second coordinate of each
element of Xk is dierent from the second coordinate of any element of Xm . Thus the sets Xn

are mutually disjoint. We claim that card( Xn ) card Xn . Indeed, for every z Xn ,
n=1
let i (z) be the smallest positive integer such that z Xi(z) . Defining


: Xn Xn by (z) = (z, i (z))
n=1 n=1

gives a one-to-one function ; which proves the claim. The sets Xn are called the disjointifi-
cation of the sets Xn .

Next, let S1 , S2 , . . . be disjoint sets and for each n let gn : Sn Tn be a function. Define

g: Sn Tn as follows: If x Sn , then there exists exactly one k such that x Sk .
n=1 n=1 n=1

Then put g (x) = gk (x). Check that g is indeed a well-defined function mapping Sn into
n=1

Tn . Furthermore, if each gn is one-to-one, then so is g.


n=1


Theorem 0.4.17 Let S1 , S2 , . . . be countable sets. Then Sn is countable.
n=1


Proof: For each fixed n N, let Tn = {(m, n) | m N}. Then note that Tn = N N. The
n=1
function f : Tn N given by f (m, n) = m is one-to-one. So Tn is countable for each n. Because
Sn is countable for each n, each of the sets Sn in the disjointification of the Sn s is countable.
We can thus find a one-to-one function gn : Sn Tn , for every n N. Letting g be defined as

above, we obtain a one-to-one function mapping Sn into Tn = N N. Because N N is
n=1 n=1

countable, we conclude that Sn is countable. But card Sn card Sn , so the
n=1 n=1 n=1
14

result follows. 

If is a set and for each we have an object a , we say the objects a ( ) are
countably many provided is countable. In this terminology the foregoing theorem can be
stated as:
The union of countably many countable sets is countable.

Now Z = {n | n N} {0} N. Since {n | n N} is obviously countable, we have expressed


Z as a countable union (i.e. a union of countably many) of countable set. So Z is countable.

Theorem 0.4.18 Q is countable.

m m
Proof: For each n N let Tn = n | m Z . The function f : Tn Z given by f n = m is

one-to-one onto. Since Z is countable, Tn is countable. But now Q = Tn . So it follows from
n=1
Theorem 0.4.17 that Q is countable. 

Corollary 0.4.19 The set R\Q of irrational numbers is not countable.

Proof: If it were, then, in view of R = Q (R\Q), we would have that R is countable. 

Corollary 0.4.20 Between any two real numbers there is an irrational number.

Proof: Let a, b R with a < b. Then (a, b) is not countable since card(a, b) = card(R). Now if
there were no irrational number between a and b we would have (a, b) Q, which would make
(a, b) countable. 

Exercise 0.4.21

(a) Let A1 , A2 , . . . , An be countable sets. Prove that the product

A1 A2 . . . An = {(a1 , a2 , . . . , an ) | ai Ai }

is countable.

(b) The Axiom of Choice states that: If {S | } is a collection of nonempty sets, then
there exists a set S consisting of exactly one element from each S . Equivalently; there
exists a function f : S such that f () S for every . Such a function is

called a choice function.
15 MAT3711/1

(i) Now let X and Y be sets such that there exists an onto function g : X Y. Show
that card(Y ) card(X) .

(ii) On the other hand, if Y = and card(Y ) card(X), show that there is a function
mapping X onto Y .

(c) Let S be the collection of all sequences whose terms are the integers 0 and 1. Show that
S is not countable. (Hint: mimic the proof of Example 0.4.11.)
16
17 MAT3711/1

Chapter 1

Metric Spaces

1.1 Metric Spaces


The idea of a metric on a set is an abstraction and generalisation of the geometric concept of
distance. On the Euclidean plane, let us write (x, y) to denote the distance from a point x to a
point y. Then it is intuitively clear that (x, y) 0 for all x and y because distance is measured
in terms of nonnegative numbers. Also, the distance between x and y is the same as the distance
between y and x. In terms of , this says (x, y) = (y, x). Further, the distance between x
and y is zero precisely when x is y. Lastly, in any triangle, the length of any side is less than
or equal to the sum of the lengths of the other two sides. On the basis of these observations we
formulate the following definition.

Definition 1.1.1 Let X be a nonempty set. A metric on X is a function d : X X R that


satisfies the following conditions:
M1. d(x, y) 0 for all x, y X.
M2. d(x, y) = 0 x = y.
M3. d(x, y) = d(y, x) for all x, y X.
M4. d(x, y) d(x, z) + d(z, y) for all x, y, z X.
If d is a metric on a set X, the pair (X, d) is called a metric space, and d(x, y) the distance
from x to y.

Property M4 in the above definition is called the triangle inequality. We emphasise that a
metric space is simply a set (any nonempty set) together with a metric (which can be thought
of as a distance function") on the set. The properties that the distance function must satisfy
parallel the familiar properties of the usual distance function on R or R2 .

A point to note is that on any set many metrics can be defined, as the following examples show.
Where we only state that a given function is a metric, you must verify that indeed it is a metric.
18

Example 1.1.2 Let X be any nonempty set. Define d : X X R by



0 if x = y
d (x, y) = .
1 if x = y

Then d is a metric on X. M1 through M3 are easily verified. To show M4, let x, y, z X. If


x = y, then d(x, y) = 0 d(x, z) + d(z, y) since each of d(x, z) and d(z, y) is nonnegative. On
the other hand, if x = y, then we cannot have z = x and z = y, because that would mean x = y.
So, say z = x. Thus:

d(x, z) + d(z + y) = 1 + d(z, y)


1 since d(z, y) 0

= d(x, y).

So the triangle inequality also holds.

The metric given in Example 1.1.2 is called the discrete metric on X.

Example 1.1.3 The function d : R R R defined by d(x, y) = |x y| satisfies M1 through


M4. It is called the usual metric on R. In general, let d : Rn Rn R be defined by

d(x, y) = (x1 y1 )2 + . . . + (xn yn )2 ,

where x = (x1 , . . . , xn ) and similarly for y. Then d is a metric on Rn called the usual or
Euclidean metric on Rn . Also, if d : C C R is defined by d(z, w) = |z w| then d is a
metric on C called the usual metric on C.

Example 1.1.4 In this example we give two other metrics on Rn . The square metric on Rn is
the metric : Rn Rn R given by (x, y) = max{|x1 y1 |, |x2 y2 |, . . . , |xn yn |}, where, as
before, x = (x1 , x2 , . . . , xn ) and y = (y1 , y2 , . . . , yn ). The taxicab metric on Rn is the metric
given by (x, y) = |x1 y1 | + |x2 y2 | + . . . + |xn yn |.

Example 1.1.5 Let (X, d) be a metric space and S X. Define dS : S S R by


dS (a, b) = d(a, b) for all a, b S. In other words, dS is the restriction of d to S S. Then
dS is a metric on S called the subspace metric. The metric space (S, dS ) is called a subspace
of (X, d). This example shows that every nonempty subset of a metric space inherits a metric
19 MAT3711/1

from the containing metric space. So, for instance, if we say consider [0, 1) with the usual me-
tric" what we mean is that equip [0, 1) with the metric d for which d(x, y) = |x y|. Henceforth
we shall adopt the convention that if no metric is specified on a subset A of a metric space, then
we assume A has the subspace metric.

Exercise 1.1.6

(a) Prove that each of the functions claimed to be a metric in the examples above is indeed a
metric.

(b) Let (X, d) and (Y, ) be metric spaces. Define by

((a, u), (b, v)) = max{d(a, b), (u, v)}.

Determine if is a metric on X Y .

(c) Let (X, d) be a metric space and define

d(x, y)
: X X R by (x, y) = .
1 + d(x, y)

Prove that (X, d) is a metric space. (Hint: To prove the triangle inequality, it might help
t
first to show that the function f (t) = 1+t is increasing for t 0.)

(d) On R let d(x, y) = (x y)4 . Show that d is not a metric on R.

(e) For x, y R define

d1 (x, y) = (x y)2 , d2 (x, y) = |x 2y|, d3 (x, y) = |x2 y2 | and d4 (x, y) = |x y|.

Determine, for each of these, whether it is a metric on R or not.

1.2 Open sets and interior


One of the key notions in the study of metric spaces is that of open sets and their complements
which are called closed sets. In this section we define these and explore some of their basic
properties.

Definition 1.2.1 Let (X, d) be a metric space, a X and r be a real number with r > 0. The
ball with centre a and radius r, denoted B(a, r), is the set

B(a, r) = {x X | d(x, a) < r}.


20

Notice that a ball is defined in terms of the metric. Thus if a set X is equipped with more than
one metric, we denote the ball with centre a and radius r by Bd (x, r) if the metric we are using
is d. Indeed it is possible for dierent metrics d and on a set X to have Bd (a, r) = B (a, r) as
we show by means of examples below.

In words, the ball B(a, r) consists of all those elements x of X such that the distance between
them and a is less than r. The ball B(a, r) is also referred to as the open ball with centre a
and radius r. The justification for the adjective open" will be clear when we study open sets in
general. The reason we sometimes wish to call a ball an open ball is that at times it is necessary
to distinguish it from a closed ball which is defined as follows:

Definition 1.2.2 Let (X, d) be a metric space, a X and r > 0. The closed ball with centre
a and radius r, denoted B(a, r), is the set

B(a, r) = {x X | d(x, a) r}.

As before if it is necessary to avoid confusion, we write B d (a, r) for the closed ball to indicate
that the metric in question is d. Also, the adjective closed" will be justified shortly. Let us
emphasise that ball" without qualification always means open ball". One sees easily that
B(a, r) B(a, r).

Pictorially, an open ball and a closed ball look as indicated in the following diagrams.

(X, d )

r
a
B (a, r)

Figure 1.2.3
21 MAT3711/1

(X, d )

r
a
B (a, r)

Figure 1.2.4

Informally, as illustrated in Figure 1.2.3, the open ball consists of all the points inside the cir-
cle excluding the points on the circumference. The points on the circumference are excluded
because the distance between each such point and a is equal to r and not less than r. On the
other hand, the closed ball, as illustrated in Figure 1.2.4, consists of all the points inside the
circle together with the points on the circumference.

As a word of caution, let us mention that diagrams help visualise things, but do not prove
anything. Now we give examples of balls in various metric spaces.

Example 1.2.5 Consider R with the usual metric. Let a R and r > 0. Then

B (a, r) = {x R | |x a| < r} = (a r, a + r) ,

the open interval with endpoints a r and a + r. On the other hand,

B(a, r) = {x R | |x a| r} = [a r, a + r] .

Example 1.2.6

(a) Consider N with the usual metric (i.e. N as a subspace of R with the usual metric). Then

3 3
B 5, = n N | |n 5| < = {4, 5, 6} .
2 2

(b) Now consider N with the discrete metric . Then

3 3
B 5, = n N | (n, 5) < =N
2 2
since for each n N, (n, 5) = 0 or 1.
22

Example 1.2.7 Consider R2 with the usual metric. Then for any (x, y) R2 and > 0,
B((x, y), ) is the open disk with centre (x, y) and radius . In other words it is the set of all
points inside the circle with centre (x, y) and radius .

Example 1.2.8 Consider [0, 1] with the usual metric. Then


1 1 1 1 3
B , = x [0, 1] : x < = 0, .
4 2 4 2 4
Notice that
3 1 1 1 1 1 1
0, = [0, 1] , + = [0, 1] BR , ,
4 4 2 4 2 4 2
where by BR (a, ) we mean the ball in R. In general, if (X, d) is a metric space and S a subspace,
then for any x S and r > 0, BS (x, r) = S BX (x, r) ; where the subscript on B indicates the
metric space in which the ball is computed. Prove this.

Now we come to the definition of openness.

Definition 1.2.9 Let (X, d) be a metric space and S X. Then:

a point x S is called an interior point of S if there is a ball B (x, r) such that


B (x, r) S.

the interior of S, denoted int (S) or S 0 , is the set of all interior points of S.

S is said to be open if each point of S is an interior point of S.

Let us reiterate. A subset S of a metric space is open provided for each x S, there is a ball
with centre x which is contained in S. Thus a set T is not open provided for some t T there
is no ball with centre t which is contained in T .

Concerning the empty set, note that because the empty set has no points, it is true that there is
no point of the empty set which is not an interior point. Consequently, the empty set is open.

Note that openness of a subset S of a metric space depends on the metric in question. For that
reason, if S is an open subset of (X, d) we say S is open in (X, d). As an example of an open
set we show that every ball B(x, ) is open regardless of the metric.

Theorem 1.2.10 Let (X, d) be a metric space. Then every ball B(a, r) in (X, d) is open.
23 MAT3711/1

Proof : Let x B(a, r). We must show that there exists a real number > 0 such that
B(x, ) B(a, r). Since x B(a, r), d(x, a) < r. Put = r d(x, a) and note that > 0. We
claim that B(x, ) B(a, r). To see this let z B(x, ). Then d(z, x) < . Now

d(z, a) d(z, x) + d(x, a)

< + d(x, a)
= r d(x, a) + d(x, a)

= r.

Thus d(z, a) < r; which implies that z B(a, r). As z is an arbitrary element of B(x, ), it
follows that B(x, ) B(a, r). Therefore B(a, r) is open. 

The foregoing theorem justifies calling balls of the form B(a, r) open. Notice that for any a, b R
with a < b, the interval (a, b) = (x , x + ) with

a+b ba
x= and = > 0.
2 2

Thus the interval (a, b) is an open ball in R with the usual metric. It is precisely for this reason
that we have always been taught to call such intervals open intervals".

Let us observe that the interval [a, b) is not open in R with the usual metric. Indeed, for any
> 0, B(a, ) = (a , a + ) [a, b). Thus [a, b) has a point which is not its interior point.
Likewise, one shows that each of the intervals (a, b] and [a, b] is not open.

Example 1.2.11 Let (X, d) be a discrete metric space. For any x X and with 0 < 1
we have B(x, ) = {x}. (Why?) Consequently if S is any subset of X, then for each x S we
have B x, 12 = {x} S. Thus every element of S is an interior point of S. Hence S is open.

The above example says in a discrete metric space every subset is open. So, for instance, in R
equipped with the discrete metric, [0, 1) is open. Of course in R with the usual metric [0, 1) is
not open as observed above.

We showed above that in any metric space the empty set is open. Now let (X, d) be a metric
space. For any x X, B(x, r) X for any r > 0. Thus each point of X is an interior point of
X. This shows that X is open in itself.
24

Example 1.2.12 We showed above that in R with the usual metric, the set [a, b) (a, b R with
a < b) is not open in R. If we equip [a, b) with the subspace metric, then [a, b) is open in itself.
By this example we are trying to emphasise that openness of a subset has everything to do with
the containing metric space.

The next result tells us that arbitrary unions of open sets are open sets, and finite intersections
of open sets are open sets.

Theorem 1.2.13 Let (X, d) be a metric space. Then

(a) If { A | } is any collection of open subsets of X, then A is open in X.


m
(b) If A1 , A2 , ..., Am are finitely many open subsets of X, then Ai is open in X.
i=1

Proof:

(a) Let x A . Then there exists 0 such that x A0 . Since A0 is open, there

is a ball B(x, r) such that B(x, r) A0 . But A0 A , so B(x, r) A . Thus

A is open.

m
(b) Let x Ai . Then x Ai for each i = 1, . . . , m. Since each Ai is open, there are positive
i=1
numbers r1 , . . . , rm such that B(x, r1 ) A1 , . . . , B(x, rm ) Am . Now let

r = min{r1 , r2 , . . . , rm },

i.e. let r be the smallest of the numbers r1 , . . . , rm . Then B(x, r) B(x, ri ) for each
m m
i = 1, 2, . . . , m. In consequence, B(x, r) B(x, ri ) Ai . 
i=1 i=1

The second result in the preceding theorem cannot be extended to an arbitrary intersection.
Witness the following example.

1
Example 1.2.14 Consider R with the usual metric. For each n = 1, 2, . . . let An = 0, 1 + n .

Then each An is open in R. But now An = (0, 1] (prove this) which is not open in R.
n=1

So far in our discussion, if X is a metric space and A an open subset, we have consistently said
A is open in X. Henceforth, if there is only one metric on a set X under consideration, and if
25 MAT3711/1

A X is open in X with the metric in question, we shall simply say A is an open subset of
X; or, briefer still, we shall say A is open. Likewise, when dealing with Rn (for any n), if we
say A Rn is open we shall be meaning that A is open in Rn equipped with the usual metric.
In this language, Theorem 1.2.13 says a union of open sets is open, and a finite intersection of
open sets is open.

The next result shows how open subsets of a subspace can be obtained from those of the con-
taining metric space.

Theorem 1.2.15 Let (X, d) be a metric space and S X be equipped with the subspace metric.
Then a set A S is open in S if and only if A = S U for some set U X which is open in
X.

Proof: () Assume A S is open in S. If A = , then A = S , and the result follows since


is open in X. If A = , for each a A there is a ball BS (a, ra ) such that BS (a, ra ) A. Now
BS (a, ra ) = S BX (a, ra ). [Verify this.] Define U X by

U= BX (a, ra ).
aA

Then U is open in X since it is a union of open sets. Now we claim that S U = A. To prove
the claim, notice that

SU = S BX (a, ra )
aA

= (S BX (a, ra ))
aA

= BS (a, ra )
aA
= A,

the last equality holding because each BS (a, ra ) A, so that BS (a, ra ) A; and clearly
aA
A BS (a, ra ) since for each x A, x BS (x, rx ) BS (a, ra ).
aA aA

() Assume A = S U for some U open in X. To prove that A is open in S, let a A. Then


a U . Since U is open in X, there exists a ball BX (a, r) U . Then

BS (a, r) = BX (a, r) S U S = A.
26

So every point of A is an interior point (relative to the subspace metric) of A. This shows that
A is open in S. 

In the case of R we have a remarkable result which tells us that every open subset of R is
expressible as a union of countably many disjoint open intervals. Recall that an interval in R
is a subset of the from (, ), (, a), (, a], [a, ), (a, ), (a, b), (a, b], [a, b) and [a, b].
Open intervals are precisely the intervals (, ), (, a), (a, ), (a, b).

For purposes of this discussion, if A R is not bounded above we shall put sup A = , and
if B R is not bounded below we shall put inf B = . Given an open subset S of R, call
an interval I a component interval of S provided I S and for any open interval J with
I J S we have I = J. In other words, a component interval of S is an open interval con-
tained in S which is such that it is not a proper subinterval of any other open interval contained
in S. This does not mean that a component interval of S contains every interval contained in S.

Example 1.2.16 Let S = (, 0) (1, 7) (7, 23). Then each of (, 0), (1, 7) and (7, 23) is
a component interval of S. (1, 23) is not a component interval of S (why not?).

Lemma 1.2.17 Let S be a nonempty open subset of R. Then every point of S belongs to one
and only one component interval of S.

Proof: Let x S. Since S is open, there is an open interval (recall that balls in R are finite
intervals of the form (a, b) with a, b R, a < b) I such that x I S. Let

a = inf{c R | (c, x) S}

and let

b = sup{d R | (x, d) S}.

Put I = (a, b). We claim that I is a component interval of S containing x.

(i) I S: I = (a, x) {x} (x, b). Since x S, it suces to show that (a, x) S and
(x, b) S. We show (a, x) S and leave the other verification as an exercise. There are
two cases; namely a = and a R. In the former case we have (r, x) S for every real
number r < x. This clearly implies (, x) S. In the latter case, for any natural number
n, (a + n1 , x) S for otherwise inf{c R | (c, x) S} would be bigger than a + n1 , which

is false since this infimum is a. Thus a + n1 , x S. But a + n1 , x = (a, x).
n=1 n=1
27 MAT3711/1

(ii) Let J be an interval with I J C. We must show that I = J. As we have I J, it


suces to show that J I. So let y J. If y = x, then y I. If y < x, then there exists
z J such that z < y. This shows that y I. Similarly we can show (do it) that if y > x
then y I. We deduce therefore that J = I.

We have so far shown that I is a component interval of S containing x. To be done we still


need to show that I is the only such component interval. So let I be a component interval of
S containing x. Then I I is an open interval with I I I , I I I and I I S.
Therefore by the definition of component interval we have that I I = I = I . 

Theorem 1.2.18 Every nonempty open set S in R is the union of countably many disjoint
component intervals of S.

Proof: For each x S let Ix denote the component interval of S containing x. Then clearly
S= Ix . If two such component intervals, Iz and Iw say, have a point in common, then their
xS
union Iz Iw is an open interval contained in S and containing both Iz and Iw . So, as argued in
the proof of Lemma 1.2.17, it follows that Iz = Iw . This shows that any two distinct component
intervals of S are disjoint.

It now remains to show that the collection of all these component intervals is countable. To
this end write the set of rational numbers as Q = {r1 , r2 , r3 , . . .}, which is possible since Q
is countable. Each Ix contains infinitely many rationals rn (recall Theorem 0.3.4). Among
the rationals rn contained in Ix there is exactly one with smallest index k. Define a function
F : {Ix | x S} N by

F (Ix ) = min{k N | rk Ix }.

Because the intervals Ix (x S) are mutually disjoint, it follows that F is a one-to-one function.
We therefore have that {Ix | x S} is countable. 

Remark. (a) The foregoing theorem tells us about the structure of all open sets in R. They
are expressible as countable unions of disjoint open intervals.

(b) The expression of an open set S in R in this manner is unique. In fact if S is a union of
disjoint open intervals, then these intervals must be component intervals of S. Note that
this statement is a consequence of Lemma 1.2.17.
28

(c) In particular, if S is an open interval then S is its only component interval. Thus, an open
interval in R cannot be expressed as a union of two nonempty disjoint open sets. Prove
this!

Now we turn our attention to interiors of sets. Recall that if S is a subset of a metric space
(X, d), then the interior of S is the set of all interior points of S. (Definition 1.2.9.) We will
show that int(S) is the largest open set contained in S. Recall that in order to show two sets A
and B to be equal, one shows that A B and B A.

Theorem 1.2.19 Let S be a subset of a metric space (X, d). Then

int(S) = {U S | U is open in X}.

Proof: Let x int(S). Then there exists r > 0 such that B(x, r) S. Since balls B(x, r) are
open, it follows that x {U S | U is open in X}. We therefore have that

int(S) {U S | U is open in X}.

On the other hand let z {U S | U is open in X}. Then there exists an open set U in X
such that z U S. Since U is open, there is a ball B(z, ) U S. This shows that z is
an interior point of S; and therefore z int(S). So {U S | U is open in X} int(S), and
equality follows. 

Corollary 1.2.20 Let S be a subset of a metric space (X, d). Then


(a) int(S) is open.
(b) int(S) is the largest open set of X contained in S.

Proof: Exercise. 

Theorem 1.2.21 Let A and B be subsets of a metric space X. Then


(a) int(A B) = int(A) int(B)
(b) A B int(A) int(B)
(c) A is open A = int(A)
(d) int(A) int(B) int(A B)
(e) int() = , int(X) = X
(f ) int(int(A)) = int(A).
29 MAT3711/1

Proof: We prove (b) and leave the proofs of remaining parts as an exercise. If A B, then
int(A) A B. So int(A) is an open set contained in B. But int(B) is the largest open set
contained in B; therefore int(A) int(B). 

Remark. We have deliberately given a proof of Theorem 1.2.21(b) without reference to points.
One can also show this by showing that each point in int(A) lies in int(B).

Example 1.2.22 We give an example that shows that " cannot be replaced by =" in Theo-
rem 1.2.21(d). Consider R with the usual metric. Let us compute the interiors of Q, R and R\Q.
We immediately have int(R) = R (why?). Let q Q. For any > 0, B(q, ) = (q , q + )
contains an irrational number. So B(q, ) Q. Thus there is no rational number which is an
interior point of Q. Similarly, for any irrational s and > 0, B(s, ) = (s , s + ) contains
a rational number, and is therefore not contained in R\Q. Therefore int(R\Q) = int(Q) = .
However int(Q R\Q) = int(R) = R. So we have sets A and B such that

int(A B) = int(A) int(B).

Example 1.2.23 Let a, b be real numbers with a < b. Then in R,

int([a, b)) = int((a, b]) = int([a, b]) = (a, b).

Also, int((, b]) = (, b), int([a, )) = (a, ). Prove these statements.

Exercise 1.2.24

(a) Let x, y be distinct elements of a metric space. Prove that there are disjoint open sets U
and V such that x U and y V .

(b) Let (X, d) be a metric space and let r be a real number with r > 0. Define : X X R
by (x, y) = rd(x, y).

(i) Prove that is a metric on X.

(ii) Show that a subset of X is open in (X, d) if and only if it is open in (X, ).

(iii) Does it follow that for each A X, intd (A) = int (A)?
30

1.3 Closed sets and closure


Having studied open sets we now pay attention to their complements. A number of results here
will be obtainable from their counterparts in the open-sets-section by appealing to De Morgans
laws, namely:
For any set X and collection {S | } of subsets of X, we have

(i) X\ S = (X\S ), and


(ii) X\ S = (X\S ).

Definition 1.3.1 A subset S of a metric space (X, d) is closed if its complement X\S is open.

Example 1.3.2 In any metric space (X, d), every closed ball B(x, r) is closed. Recall that
B(x, r) = {y X | d(x, y) r}. To prove that B(x, r) is closed we must show that X\B(x, r)
is open. Let z X\B(x, r). Then d(z, x) > r. Put = d(z, x) r and note that > 0. We
will show that B(z, ) X\B(x, r) which will complete the proof. So let w B(z, ). Then
d(w, z) < . Now d(x, w) + d(w, z) d(x, z). Hence d(x, w) d(x, z) d(w, z) > d(x, z) = r.
This shows that d(w, x) > r; whence we deduce that w B(x, r). Consequently w X\B(x, r).
So every element of X\B(x, r) is an interior point. Thus X\B(x, r) is open and therefore B(x, r)
is closed. So, as in the case of the open ball, we are justified to call B(x, r) a closed ball.

Example 1.3.3 Let a, b R with a < b. Then each of the intervals [a, b], (, b] and [a, ) is
closed. Indeed, [a, b] = R\(, a) (b, ); (, b] = R\(b, ) and [a, ) = R\(, a).

Example 1.3.4 In a discrete metric space every subset is closed. Why is this so?

Theorem 1.3.5 Let (X, d) be a metric space and x0 be any element of X. Then {x0 } is closed.

Proof: Let z X\{x0 }. This means that z = x0 . So r = d(z, x0 ) > 0. Since d(x0 , z) is not
less than r, it follows that x0 B(z, r). Hence B(z, r) X\{x0 }. So X\{x0 } is open, and the
theorem is proved. 

The theorem above tells us that singletons (i.e. sets consisting of a single point) are closed in
any metric space. Informally, we say points are closed" in a metric space. We will see that in
fact every finite set is closed.
31 MAT3711/1

The following result gives properties of closed sets which are akin to those of open sets given in
Theorem 1.2.13.
Theorem 1.3.6 Let X be a metric space. Then
(a) X and are closed sets.
m
(b) If A1 , A2 , . . . , Am are finitely many closed sets, then Ai is closed.
k=1
(c) If {A | } is a collection of closed sets, then A is closed.

Proof: (a) X = X\ and as is open we have that X is closed. Next, = X\X, and therefore
is closed since X is open.

(b) and (c) are left as exercises. Apply the de Morgans law to Theorem 1.2.13. 

Corollary 1.3.7 In any metric space, every finite subset is closed.

Proof: Let S be a finite subset of a metric space. Say S = {s1 , . . . , sm }. Then

S = {s1 } {s2 },

a finite union of closed sets (recall that singletons are closed Theorem 1.3.5). So by Theorem
1.3.6(b) we have that S is closed. 

When we were studying open sets we encountered the concept of the interior of a set. We
saw that the interior was defined in terms of points in that it is the set of all interior points;
and showed it to be expressible in terms of open sets in that it is the largest open subset of
any given set. Likewise in the discussion on closed sets we will introduce the concept of closure.
This we will define in terms of points and later prove that it is expressible in terms of closed sets.

Definition 1.3.8 Let (X, d) be a metric space and x X. By a neighbourhood (to be abbre-
viated nbd) of x we mean an open set containing x.

For instance in R2 , B ((0, 0) , 1) is a nbd of the point 12 , 13 . We caution that in certain books
by a nbd of a point is meant any open set (not necessarily a ball) containing the point. All
results stated in terms of neighbourhoods as defined here are true with "nbd" meaning any open
set containing the point, and vice versa. With this nomenclature, note (actually prove) that a
set is open if and only if it contains a nbd of each of its points.
32

Definition 1.3.9 Let (X, d) be a metric space, S be a subset of X and z X. We say z is an


adherent point of S if every nbd of z contains a point of S. We say z is an accumulation
point of S if every nbd of z contains a point of S dierent from z. The closure of S, denoted
by S or cl(S), is the set of all adherent points of S. The derived set of S, denoted by S , is the
set of all accumulation points of S.

Let us pause a bit and digest these concepts. For an adherent point p of a set A we require
that every nbd of p must contain a point of A. Such a point may be p itself. However, for p to
be an accumulation point of A, every nbd of p must contain a point of A dierent from p. In
other words, every nbd of p must contain a point of A\{p}. It clearly then follows that every
accumulation point of a set B is an adherent point of B. In symbols, B B. Is the converse
true? Before answering this let us notice that for any subset S of a metric space, if z S then
every nbd of z contains a point of S (z itself), and so every point of S is an adherent point
of S. That is, S S. The example below shows that not every adherent point of a set is an
accumulation point of the set.

Example 1.3.10 Consider N as a subset of R with the usual metric. For any m N, m is an
adherent point of N. Now B(m, 12 ) = {m} is a nbd of m which contains no element of N dierent
from m. Thus m is not an accumulation point of N. We have actually shown that N = .

In the discussion preceding Example 1.3.10, we saw that for any subset B of a metric space,
B B and B B. Hence B B B. In fact we have the following result.

Theorem 1.3.11 Let S be a subset of a metric space. Then S = S S .

Proof: We have already observed that S S S. So it remains to show that S S S . Let


x S. If x S, then given a nbd N of x we have that N contains a point p of S since x is an
adherent point of S. As x S it follows that p = x. Thus N contains a point of S dierent
from x. Since N is an arbitrary nbd of x, we conclude that every nbd of x contains a point of
S\{x}. This proves that x is an accumulation point of S, i.e. x S . All in all we have shown
that S S S . Therefore S = S S . 

In words the foregoing result says: (a) the closure of S consists of S together with all the
accumulation points of S and (b) p S every nbd of p has nonempty intersection with S. It
is important to note that an accumulation point of a set A need not be an element of A. As an
33 MAT3711/1

easy example, note that every nbd of 1 contains an element of the interval (0, 1). Thus 1 is an
accumulation point of (0, 1) which does not belong to (0, 1). On the other hand every point of
(0, 1) is an accumulation point of (0, 1).
From the definition, if p is an accumulation point of a set A, then we are guaranteed that every
nbd of p contains some point of A dierent from p. We show below that every nbd of an accu-
mulation point p of a set A contains infinitely many points of A.

Theorem 1.3.12 Let (X, d) be a metric space, A be a subset of X and p be an accumulation


point of A. Then every nbd of p contains infinitely many points of A.

Proof: Suppose, by way of contradiction, that p has a nbd N that contains only finitely
many points of A. Let a1 , a2 , . . . , am be those points of N A that are dierent from p. Let
r = min{d(p, a1 ), d(p, a2 ), . . . , d(p, am )}. Then r > 0 since d(p, ai ) > 0 for each i = 1, 2, . . . , m.
Since p N and N is open, there exists > 0 such that B(p, ) N . Now let = min{r, }.
Then > 0 and B(p, ) contains no point of A\{p}. To see this, note that B(p, ) N , so if it
did contain a point of A\{p} that point would be ak for some k = 1, . . . , m. But if ak B(p, ),
then d(p, ak ) < r d(p, ak ), i.e. d(p, ak ) < d(p, ak ); a contradiction. 

Corollary 1.3.13 A finite subset of a metric space has no accumulation points.

Proof: This follows immediately from Theorem 1.3.12. 

We saw above that an accumulation point of a set need not belong to the set. In the case of
closed sets this cannot happen as the next result shows.

Theorem 1.3.14 The following statements about a subset S of a metric space (X, d) are equi-
valent.
(a) S is closed.
(b) S contains all its adherent points.
(c) S contains all is accumulation points.

Proof: (a) (b): Let p be an adherent point of S. Since S is closed, X\S is open. So if p were
an element of X\S there would exist r > 0 such that B(p, r) X\S. But then that would mean
that B(p, r) is a nbd of p which contains no point of S; a contradiction since p is an adherent
34

point of S. Thus p X\S, i.e. p S.

(b) (c): Every accumulation point is an adherent point. So if (b) holds, then S contains all
its accumulation points.
(c) (a): We must show that X\S is open. So let p X\S. Then p is not an accumulation
of point S, for if it were then it would belong to S by the hypothesis. So p has a nbd N which
contains no point of S\{p}. This means that N (S\{p}) = . Since p S anyway, it follows
that N S = , and therefore N X\S. So p is an interior point of X\S, and hence X\S is
open as required. 

Remark 1.3.15. The statement (a) (c) in the above theorem says a set is closed if and
only if it contains all its accumulation points. Now if a set has no accumulation point, some
people might have qualms with the statement that it contains all its accumulation points".
This should however not cause us anxiety since (a) (c) of the foregoing theorem can also be
written as: S is closed S S. Now if S has no accumulation point, then S = S; so
that this case is in fact included in S contains all its accumulation points". If you are still not
comfortable with this, restate (a) (c) as: A subset of a metric space is closed if and only if it
has no accumulation points or it contains all its accumulation points if it has any.

Corollary 1.3.16 Let A be a subset of a metric space (X, d). Then A is closed if and only if
A = A.

Proof: () Let A be closed. Then A A by Theorem 1.3.14. Since A A for every set A, we
conclude that A = A.

() Let A = A. We show that X\A is open. Let p X\A = X\A. Then p A = A A .


Therefore p is neither a point of A nor an accumulation point of A. Hence p has a nbd N which
does not intersect A. Then N X\A. So p is an interior point of X\A, and as p was chosen
arbitrarily, we conclude that X\A is open, whence the result follows. 

Theorem 1.3.17 Let S be a subset of a metric space (X, d). Then

S= {F X | F is closed and F S}.

Proof: Let F be a closed subset of X with S F . We want to show that S F . So let x S.


Then every nbd of x has nonempty intersection with S; and therefore with F . This shows that
35 MAT3711/1

x F = F , since F is closed. We therefore have that S F . As F is an arbitrary closed subset


of X that contains S, we conclude that S {F X | F is closed and F S}. To show the
reverse inclusion let z {F X | F is closed and F S}. If we assume that z S, then
there is a nbd N of z with N S = . Thus S X\N . But now X\N is a closed set containing
S; therefore z X\N , which is a contradiction since z N as N is a nbd of z. So z S, and
hence {F X | F is closed and F S} S. This completes the proof. 

Corollary 1.3.18 The closure of any subset of a metric space is closed.

Proof: Intersections of closed sets are closed. 

We state the next result about closure and juxtapose it with its interior counterpart, Theorem
1.2.21. We deliberately include other results already stated. We shall use the notation A0 for
int(A).

Theorem 1.3.19 Let A and B be subsets of a metric space X. Then


(a) A is closed A = A (a ) A is open A = A0
0
(b) A =A (b ) A0 = A0
(c) = and X = X (c ) ()0 = and X 0 = X
(d) A B A B (d ) A B A0 B 0
(e) A B = A B (e ) (A B)0 = A0 B 0
(f ) A is the smallest closed set containing A (f ) A0 is the largest open set contained in A.
(g) A B A B (g ) (A B)0 A0 B 0

Proof: We give a proof of (e) only and leave the proofs of other statements not proved so far as
an exercise. Note that since A A and B B, we have A B A B. But now A B is
a finite union of closed sets, so it is a closed set containing A B. Thus, by Theorem 1.3.17,
A B A B. So it remains to show that A B A B. Let x A B. Then x A
or x B. Suppose x A. Let N be a nbd of x. Then N A = . Since A A B, it
follows that N (A B) = . Thus every nbd of x has a nonempty intersection with A B, so
x A B. Similarly, if x B we have that x A B. Thus A B A B, and consequently
A B = A B. 

Example 1.3.20 Consider R with the usual metric. Then Q = R and (R\Q) = R. That is,
every x in R is an accumulation point of Q and also of R\Q. Verify this. Thus Q = Q Q = R.
36

Similarly, R\Q = R. So Q R\Q = R. On the other hand, Q (R\Q) = . So " in Theorem


1.3.19 (g) and (g ) cannot be replaced by =". In R, any interval of the form (a, b] where a, b R
with a < b is neither open nor closed. So it is possible for a set to be neither open nor closed.
Now in any metric space X, each of and X is both open and closed. Is it possible for a proper
subset (i.e. one which is neither empty nor the whole set) to be both open and closed? Witness
the following example.

Example 1.3.21 Let (X, d) be any discrete metric space. We saw in Example 1.2.11 that every
subset of X is open. Thus every subset of X is closed (as its complement is open).

Example 1.3.22 In this example we examine the relationship between the closed ball B(x, r)
and the closure B(x, r) of the open ball B(x, r). If y B(x, r), then d(y, x) < r; and so
d(y, x) r, implying that B(x, r) B(x, r). Now B(x, r) is a closed set as was shown in
Example 1.3.2. Because the closure of any set is the smallest closed set containing the set, we
conclude that B(x, r) B(x, r).

Example 1.3.23 The preceding example shows that B(x, r) B(x, r). We now give examples
where B(x, r) = B(x, r) and where B(x, r) = B(x, r). Consider R with the usual metric. Then
B(0, 1) = (1, 1) = [1, 1]. Also B(0, 1) = [1, 1]. Now consider R with the discrete metric.
Then B(0, 1) = {0} = {0}. On the other hand B(0, 1) = R.

Definition 1.3.24 Let A be a subset of a metric space (X, d). A point p X is called a boun-
dary point of A in case p A X\A. The boundary of A, denoted bd(A), is the set of all
boundary points of A. In other words, bd(A) = A X\A.

It is clear that x bd(A) if and only if every nbd of x contains a point of A as well as a point
of the complement of A.

Example 1.3.25 Let A be the subset of R2 given by A = {(x, y) R2 | x2 + y 2 > 1}. Then
bd(A) = {(x, y) R2 | x2 + y 2 = 1}. Verify this.

Example 1.3.26 In R with the usual metric, bd(Q) = Q R\Q = R. On the other hand,
bd(N) = N.
37 MAT3711/1

Example 1.3.27 Let A be a subset of a metric space which is both open and closed. Therefore
its complement is also both open and closed. Now bd (A) = A X\A = A (X\A) since A = A
and X\A = X\A. Thus bd(A) = . In the exercise we ask that you prove that the converse is
also true.

Definition 1.3.28 A subset A of a metric space (X, d) is said to be dense in X (or simply
dense) if A = X. It is nowhere dense if int(A) = .

Example 1.3.29 The rationals are dense in the reals. That is, Q as a subset of R with the
usual metric is dense in R. Indeed Q = R.

Theorem 1.3.30 Let A be a subset of a metric space (X, d). Then A is dense in X if and only
if A intersects every nonempty open set.

Proof: () Let A be dense in X and U X be a nonempty open set. We must show that
A U = . If A U were empty, then we would have A X\U . But X\U is closed since U is
open. This would mean A X\U . But A = X since A is dense, so we would have X X\U ,
which is impossible as U = .
() We show that every x X is an element of A. Let x X and N be a nbd of x. Then N is
a nonempty open set. Therefore N A = , by the hypothesis. We have thus shown that every
nbd of x contains a point of A. So x A and therefore X A X. 

Exercise 1.3.31

(a) Let A be a subset of a metric space with bd(A) = . Show that A is both open and closed.

(b) Let A = {(x, y) R2 | y 0}. Find bd(A).

(c) Let A and B be subsets of a metric space X. Prove that X\(A B) = (X\A) (X\B).

(d) Let A be a subset of a metric space X. Prove that:

(i) A0 , (X\A)0 and bd(A) are pairwise disjoint sets whose union is X.
(ii) bd(A) is closed.
(iii) A = A0 bd(A).

(e) Let A be a subset of a metric space X. Prove that A is dense in X if and only if
int(X\A) = .
38

(f) Let A be a subset of a metric space X. Prove that

(i) X\A = X\A0 .

(ii) (X\A)0 = X\A.

(g) Let (X, d) be a metric space, x X and and be positive numbers with < . Prove
that B(x, ) B(x, ).

(h) Let A be a subset of R which is bounded above. Show that sup A A.

(i) Let A be a subset of a metric space. Prove that:

(i) A is closed.

(ii) A and A have the same accumulation points.

(j) Do A and A always have the same accumulation points? Prove your assertion.

(k) Let S be a subspace of the metric space X. Prove that a subset T of S is closed in S if
and only if T = S K for some K which is closed in X.

(l) Let A be the subset of R given by A = { n1 1


m | m, n N}. Find all the accumulation
points of A.

(m) Let A be a subset of a metric space. Prove that int(bd(A)) = if A is open or if A is


closed.

(n) Let U be an open subset of (X, d) and A X. Show that U A = U A = .

1.4 Bounded sets, diameters and distances


Throughout this section we shall adopt the convention that if A is a nonempty subset of R which
is not bounded above, then we set sup A = .
We define boundedness of a subset of a metric space in such a way that in the case of R it
reduces to boundedness as per Definition 0.2.4. Recall that A R is bounded if there exist real
numbers c and d such that c x d for all x A. Thus A is bounded if and only if there are
real numbers and such that < x < for all x A. In other words A is bounded if and
only if A is contained in some ball in R.
39 MAT3711/1

Definition 1.4.1 Let A be a subset of a metric space (X, d). We say A is bounded if A B(x, r)
for some r > 0 and some x X.

Thus, a bounded subset of a metric space is one which is contained in some ball. Now let A be
a bounded subset of a metric space (X, d). Find x X and r > 0 such that A B(x, r). We
intend to show that the set {d(a, b) | a, b A} is bounded above. Of course it is bounded below
since 0 d(a, b) for all a, b A. Let a, b A. Then d(a, b) d(a, x) + d(x, b) r + r = 2r
since d(a, x) r and d(x, b) r, as a, b A B(x, r). Therefore 2r is an upper bound for
{d(a, b) | a, b A}. Thus, A is bounded if and only if there exists M > 0 such that d(x, y) M
for all x, y A.

Definition 1.4.2 Let A be a subset of a metric space (X, d). If A is bounded, we define the
diameter of A, denoted diam(A), by

diam(A) = sup{d(a, b) | a, b A}.

If A is not bounded we set diam(A) = . We also set diam() = 0.

Example 1.4.3 Let (X, d) be a metric space, x X and r > 0. Then diam(B(x, r)) 2r.
Similarly, diam(B(x, r)) 2r. No doubt you are wondering why we are not saying the di-
ameter of the closed ball with radius r is equal to 2r. Afterall the closed ball includes its
boundary points; the points on the circumference", so to speak. The flaw in this line of
thinking is that you are pushing your geometric intuition about balls too far. Indeed in
Rn , diam(B(x, r)) = diam(B(x, r)) = 2r.
However in our old friend, the discrete space, things arent what they seem to be. Take any
set X with at least two elements and equipped with the discrete metric. For any x X,
diam(B(x, 12 )) = diam({x}) = 0. Also, diam(B(x, 1)) = 1 and diam(B(x, 2)) = 1. Note that
the last two statements are false if X has only one element.

Definition 1.4.4 Let (X, d) be a metric space, A and B be nonempty subsets of X and z X.
The distance from z to A, denoted d(z, A), is defined by

d(z, A) = inf{d(z, a) | a A}.

The distance between A and B, denoted d(A, B), is defined by

d(A, B) = inf{d(a, b) | a A, b B}.


40

So if we wish to compute the distance between z and A we must compute d(z, a) for all a A
and find the infimum of the set of all these numbers. Similarly for d(A, B).

Example 1.4.5 In R with the usual metric, d(3, (2, 8]) = 5. Note that there is no point of

(2, 8] which is closest to 3. On the other hand, d( 2, Q) = 0 since, although 2 Q, no

matter how small > 0 is, there is a rational number q such that d( 2, q) < .

Theorem 1.4.6 Let (X, d) be a metric space and A X. For any z X, z A if and only if
d(z, A) = 0.

Proof: () Suppose z A and let > 0. Then, as every nbd of z contains a point of A, there
exists a A such that a B(z, ). Thus d(z, A) < . So any positive number cannot be a lower
bound for {d(z, x) | x A}. But of course 0 is a lower bound for this set, therefore it is the
greatest lower bound for this set; that is 0 = inf{d(z, x) | x A} = d(z, A).

() Suppose d(z, A) = 0. We must show that z A. If we assume that z A, then there exists
some > 0 such that B(z, ) A = . This would mean that for each x A, x B(z, ); that
is, d(z, x) . Consequently would be a lower bound for {d(z, x) | x A}, and hence we
would have d(z, A) > 0, contradicting the hypothesis. 

Theorem 1.4.7 Let A be a bounded subset of a metric space (X, d). Then A is bounded and
diam(A) = diam(A).

Proof: Since A is bounded, there exists M > 0 such that d(x, y) M for all x and y A. We
will show that d(a, b) M + 1 for all a, b A. Let a, b A. Pick x, y A such that x B(a, 12 )
and y B(b, 12 ). Then

d(a, b) d(a, x) + d(x, b)

d(a, x) + d(x, y) + d(y, b)


1 1
+M +
2 2
= M + 1.

Therefore A is bounded.
To show that diam(A) = diam(A), first note that if B is a bounded set and C B, then C is
bounded and diam(C) diam(B). Now let > 0. As before, given a, b A, find x, y A such

that d(a, x) < 2 and d(b, y) < 2 . Then d(a, b) diam(A) + , arguing as above and noting that
41 MAT3711/1

d(x, y) diam(A). We therefore have that diam(A)+ is an upper bound for {d(a, b) | a, b A}.
Hence diam(A) diam(A) + . As this is true for every > 0, we deduce from Theorem 0.2.3
that diam(A) diam(A). But diam(A) diam(A) since A A; so diam(A) = diam(A). 
Exercise 1.4.8

(a) Let (X, d) be a metric space. Prove that a subset A of X is closed if and only if for each
x X\A, d(x, A) = 0.

(b) Let A and B be nonempty subsets of a metric space (X, d). Show that

d(A, B) = sup{d(a, B) | a A} = sup{d(b, A) | b B}.

(c) Give an example of a metric space (X, d) and nonempty subsets A and B of X such that
A B = but d(A, B) = 0.

(d) Let A be a subset of a metric space such that A B(x, r) for some x X and r > 0.
Prove that diam(A) 2r.

1.5 The Bolzano-Weierstrass Theorem


Our goal in this section is to show that every infinite bounded subset of Rm has an accumulation
point. This result does not hold in all metric spaces. Indeed consider N with the discrete metric.
Then A = N is an infinite subset of N. Since d(x, y) 1 for all x, y A, it follows that A is
bounded. Now A has no accumulation point since, for each n N, B(n, 1) = {n} and therefore
does not contain a point of A\{n}.

In order to prove the Bolzano-Weierstrass Theorem for Rm (for the duration of this discussion
Rm will have the usual metric), we shall need some preliminary results.

Theorem 1.5.1 For each n N let In = [an , bn ], (an < bn ) and suppose that I1 I2 . . .;

that is, In In+1 for each n N. Then In = .
n=1

Proof: For each n N, In I1 ; and therefore an b1 . The set {an | n N} is therefore


bounded above. Let denote its supremum. Thus an for each n. If we can show that

bn for each n, then we shall be done since then we will have In . Suppose, by way
n=1
of contradiction, that there is some k N such that bk < . Since = sup{an | n N} and bk
is strictly smaller than , bk is not an upper bound for {an | n N}. There is therefore some
N such that bk < a . Let p be the larger between k and ; that is let p = max{k, }. Since
42

a1 a2 . . . and b1 b2 . . . we conclude that bp bk < a ap . This means that bp < ap ;


a contradiction. Therefore an bn for each n N. 

Remark. (a) The above result fails for intervals which are not closed. Take for instance

Jn = (0, 12 ), n N. Then J1 J2 J3 . . ., but Jn = .
n=1

(b) What the theorem says is that In is not empty. It does not tell us how many elements
n=1
the intersection contains. In fact if we let = inf{bn | n N} then one can show that

[, ] In .
n=1

A closed cell in Rm is a set of the form I = {(x1 , . . . , xm ) Rm | ai xi bi for i = 1, 2, . . . , m}


where ai , bi are real numbers with ai bi . It is clear that I = [a1 , b1 ] . . . [an , bn ].

Corollary 1.5.2 Let I1 , I2 , . . . be closed cells in Rm with I1 I2 . . .. Then there is a point


in Rm which belongs to all the cells.

Proof: For each k N write Ik = {(x1 , . . . , xm ) | ak xi bk , i = 1, . . . , m} where ak and bk


are real numbers with ak bk for i = 1, . . . , m. Then note that

[a11 , b11 ] [a21 , b21 ] . . . [an1 , bn1 ] . . .


[a12 , b12 ] [a22 , b22 ] . . . [an2 , bn2 ] . . .
...

[a1m , b1m ] [a2m , b2m ] . . . [anm , bnm ] . . .



By Theorem 1.5.1 find points y1 [ak1 , bk1 ], . . . , ym [akm , bkm ]. Then the point
k=1 k=1
y = (y1 , . . . , ym ) belongs to all the cells. 

Momentarily, for x = (x1 , x2 , . . . , xm ) Rm , let us write ||x|| = x21 + . . . + x2m . Recalling that
addition and subtraction in Rm are defined componentwise; i.e. x y = (x1 y1 , . . . , xm ym )
where x = (x1 , . . . , xm ) and y = (y1 , . . . , ym ), we note that with this notation the usual distance
between x and y is ||x y||.


Lemma 1.5.3 Let x = (x1 , . . . , xm ) Rm . Then |xi | ||x|| m max{|x1 |, . . . , |xm |} for each
i = 1, . . . , m.
43 MAT3711/1

Proof: Since ||x||2 = x21 + . . . + x2m , it is clear that |xi | ||x|| for each i. Likewise, if we put

b = max{|x1 |, . . . , |xn |}, then ||x||2 mb, whence ||x|| mb. 

Theorem 1.5.4 (Bolzano-Weierstrass Theorem). Every bounded infinite subset of Rm has an


accumulation point.

Proof: Let B be a bounded subset of Rm that has infinitely many points. Because B is bounded
there is a closed cell I1 which contains B. We divide I1 into 2m closed subcells by bisecting
each of its sides. Since I1 contains infinitely many points of B, at least one part obtained in this
subdivision will contain infinitely many points of B. Let I2 be one such part. Next we divide I2
as we did I1 to obtain a subcell I3 of I2 which contains infinitely many points of B. Continuing
this process we obtain closed cells I1 I2 I3 . . .. By Corollary 1.5.2 there is a point y
which belongs to all the cells I1 , I2 , . . .. We show that y is an accumulation point of B.
First note that if I1 = [a1 , b1 ] . . . [am , bm ] with ai < bi and if we set

(I1 ) = max{b1 a1 , . . . , bm am },

then (I1 ) > 0, and in fact (I1 ) is the length (or diameter in the language of metric spaces) of
the largest side of I1 . According to the above construction of the cells In , we have

1
0 < (In ) < (I1 ),
2n1

for each n N. Let U be a nbd of y and pick r > 0 such that B(y, r) U . Let k be a natural
number such that
1
k > 1 + log2 .
m r (I1 )
Then
m
(I1 ) < r.
2k1
For such k, if w Ik then, by Lemma 1.5.3, we have that

m
||y w|| m (Ik ) = k1 (I1 ) < r;
2

showing that Ik U . Since Ik contains infinitely many points of B, so does U . So U does


contain a point of B\{y}. Therefore y is an accumulation point of B. 

Exercise 1.5.5 Prove the validity of the statements made in the Remark following Theorem
1.5.1.
44
45 MAT3711/1

Chapter 2

Convergence and Completeness

2.1 Sequences and Subsequences


In the theory of metric spaces sequences play a very important role. The aim of this chapter is
to study convergence of sequences in metric spaces.

Definition 2.1.1 Let X be a set (not necessarily a metric space). A sequence in X is a func-
tion f : N X. For each n N let xn denote f (n). The elements xn of X are called the terms
of the sequence.

In practice we normally specify a sequence by its terms. We shall thus frequently speak of a
sequence {xn } in X" in place of the more tedious a sequence f : N X in X for which
f (n) = xn ". It is crucial always to keep in mind the dierence between a sequence {xn } and
the set {xn | n N} whose elements are the terms of the sequence {xn }. Also important is that
even if terms of a sequence are repeated, when one writes them down, one does not remove the
repetition as one would do in the case of the set whose elements are the terms of a sequence.
For instance the sequence {an } in R for which

1, if n is odd
an =
n, if n is even

has terms 1, 2, 1, 4, 1, 6, . . .. The set of terms is of course {1, 2, 4, 6, . . .}.

Definition 2.1.2 Let {xn } be a sequence in a set X. Let n1 < n2 < . . . be a sequence of natural
numbers which is strictly increasing. The sequence xn1 , xn2 , xn3 , . . . , xnk , xnk+1 , . . . is called a
subsequence of the sequence {xn }.

The thrust of the definition of a subsequence {xnk } is that the indices n1 , n2 , . . . are strictly
increasing; that is n1 < n2 < . . ..
46

Example 2.1.3 Let {an } be the sequence for which an = 1 if n is odd and an = n if n is even.
Then {a2n } is the subsequence a2 , a4 , a6 , . . . whose terms are 2, 4, 6, . . .; and the subsequence
{a2n+1 } is the subsequence a1 , a3 , a5 , . . . whose terms are 1, 1, 1, . . ..

In terms of the definition of a sequence as a map from N into a set, notice that if f : N X
is a sequence and : N N is a strictly increasing function (i.e. (m) < (n) for all m < n
in N), then (f )(n) = f ((n)) = x(n) . Conversely, if {xnk } is a subsequence of the sequence
{xn }, then there is a function : N N which is strictly increasing, such that xnk = x(k) for
each k N.

An observation which will be useful later is that if n1 < n2 < . . . is a sequence of natural num-
bers which is strictly increasing, then given any natural number N , there exists k N such that
n > N for each k.

Definition 2.1.4 Let {xn } be a sequence in a metric space (X, d), and let p X. We say
the sequence converges to p if for every > 0 there exists a natural number n0 such that
d(xn , a) < whenever n n0 . In this case we say p is the limit of the sequence. We use the
notation xn p or lim xn = p or lim xn = p to indicate that the sequence {xn } converges to p.
n n

Note that if we say a sequence in a metric space converges, we mean there is a point p in the
metric space such that xn p.

1
Example 2.1.5 Let {xn } be the sequence given by xn = n for each n N in R with the usual
metric. One checks easily that xn 0. So the sequence converges in R with limit 0. Now let
1
(X, d) denote the set R\{0} with the usual metric, and let xn = n as before. In this case the
sequence does not converge in (X, d) because there is no point of X which is the limit of the
sequence.

Theorem 2.1.6 Let {xn } be a sequence in a metric space (X, d) and suppose p, q X are such
that xn p and xn q. Then p = q.

r
Proof : If not, then d(p, q) > 0. Let r = d(p, q). Since xn p and 2 > 0, there exists n0 N
r
such that d(xn , p) < 2 whenever n n0 . Similarly, since xn q, there exists n1 N such that
47 MAT3711/1

r
d(xn , q) < 2 whenever n n1 . Now let k = max{n1 , n2 }. Then k n1 and k n2 . Thus

r = d(p, q) d(p, xk ) + d(xk , q)


r r
< +
2 2
= r

We therefore have r < r; which is absurd. 

This theorem tells us that in a metric space a sequence cannot converge to more than one point.
In other words, limits are unique. That is why in fact in Example 2.1.5 we could say with cer-
tainty that there is no point p in R\{0} such that the sequence given there converges to p. If there
were such a point then this point p would be dierent from 0, and as we are using the same metric
in R\{0} as in R we would then have had xn 0 and xn p in R, contradicting Theorem 2.1.6.

Given a set A X and a sequence {xn } in X, we say {xn } is eventually in A if there is a


natural number n0 such that for all n n0 we have xn A. By a tail of a sequence {xn }
we mean a set of the form {xn | n n0 } for some n0 N. For instance each of the sets
{x31 , x32 , x33 , . . .}, {x98 , x99 , x100 , . . .} is a tail of the sequence {xn }. Note that terms of a sub-
sequence do not necessarily form a tail of a sequence because starting from xn1 to xn2 to xn3 ,
etc. some terms of the sequence may have been left out. A tail starts with xn0 and includes all
terms thereafter. Thus, a sequence {xn } is eventually in A if and only if A contains a tail of the
sequence.
We now characterise convergence in terms of neighbourhoods.

Theorem 2.1.7 Let {xn } be a sequence in a metric space (X, d). Then xn p if and only if
for every nbd U of p, {xn } is eventually in U . Put dierently; a sequence converges to p if and
only if every nbd of p contains a tail of the sequence.

Proof : () Let xn p and U be a nbd of p. Then there exists > 0 such that B(p, ) U .
Since xn p, we can find n0 N such that d(xn , p) < for all n n0 . This means that
xn B(p; ) U for every n n0 . Thus {xn } is eventually in U .

() We want to show that xn p. So let > 0. Then B(p, ) is a nbd of p; therefore {xn }
is eventually in B(p, ). But this means that there is a natural number n0 such that if n n0
then xn B(p, ); that is, such that d(xn , p) < whenever n n0 . So xn p. 
48

Theorem 2.1.8 Let (X, d) be a metric space and A be a nonempty subset of X. For any p X,
p A if and only if there is a sequence {xn } whose terms are points of A such that xn p.

Proof : () Let p A. By Theorem 1.3.11, every nbd of p contains a point of A. For each
n N choose a point xn in A such that xn B(p, n1 ); i.e. such that d(xn , p) < 1
n. Given any
> 0, there exist n0 N such that n0 > 1 . Now if n n0 , then d(xn , p) < 1
n 1
n0 < . Thus
xn p.

() If xn p where each xn is in A; then every nbd of p contains a tail of {xn }. Thus it


contains a point of A and therefore p A. 

We note that it is possible for a sequence which does not converge to have a subsequence that
converges. For instance the sequence 1, 12 , 1, 13 , 1, . . . does not converge in R but each of the
subsequences 1, 1, 1, . . . and 12 , 13 , 14 , . . . converges. The limits of subsequences of a sequence {xn }
are called the subsequential limits of {xn }. They form a closed set as we will show below.
First we prove that if a sequence converges then so does each of its subsequences. Furthermore,
if xn p, then each subsequence of {xn } converges to p.

Theorem 2.1.9 A sequence {xn } in a metric space (X, d) converges to p if and only if every
subsequence of {xn } converges to p.

Proof : () If every subsequence of {xn } converges to p, then {xn } converges to p because every
sequence is a subsequence of itself.

() Let xn p and {xnk } be a subsequence of {xn }. Given > 0, find m N such that
d(xn , p) < whenever n m. Choose N such that n m. Then for k we have
nk n m, and therefore d(xnk , p) < . Thus xnk p as k . 

Theorem 2.1.10 The subsequential limits of any sequence in a metric space form a closed set.

Proof : Let {xn } be a sequence in a metric space (X, d) and E be the set of all subsequential
limits of {xn }. Let q be an accumulation point of E. The idea of the proof is to construct a
subsequence of {xn } which converges to q. So choose n1 N such that xn1 = q. (If there is no
such n1 , then xn q and therefore, by Theorem 2.1.9, E = {q} which is a closed set; and hence
49 MAT3711/1

there is nothing to prove). Put = d(xn1 , q). Then > 0. Since q is an accumulation point of
1
E, there exists x E such that d(x, q) < 22
. Likewise, since x E, there exists n2 > n1 such
1
that d(x, xn2 ) < 22
. Thus

1
d(q, xn2 ) d(q, x) + d(x, xn2 ) < .
2

Continuing, by induction, suppose n1 > n2 > . . . > nk1 have been chosen as above. Choose
nk > nk1 such that d(q, xnk ) < 21k . We thus obtain a subsequence {xnk } which converges
to q. Therefore q E, and hence E is closed. 

Definition 2.1.11 Let {xn } be a sequence in a metric space (X, d). A point y of X is a cluster
point of {xn } if for every > 0 and every n N, there exists m > n such that d(xm , y) < .

Note that there is a dierence between a cluster point and a limit of a sequence. If y is a cluster
point, then given any > 0 and any index k, we can find an index > k such that d(x , y) < .
In the case of y being a limit, given > 0 we can find an index k so that for all indices > k,
d(x , y) < .

Quite clearly, a limit of a sequence is a cluster point, but not conversely. The next result shows
that y is a cluster point of {xn } if and only if some subsequence of {xn } converges to y. In
consequence, cluster point" is another name for subsequential limit".

Theorem 2.1.12 Let {xn } be a sequence in a metric space (X, d). Then a point y of X is a
cluster point of {xn } if and only if some subsequence of {xn } converges to y.

Proof : () Let y be a cluster point of {xn }. Choose an index n1 such that d(xn1 , y) < 1. Now
1
suppose indices n1 > n2 > . . . > nk have been chosen such that d(xni , y) < i for i = 1, 2, . . . , k.
1
Then choose an index nk+1 > nk such that d(xnk+1 , y) < k+1 . We thus have a subsequence
1
{xnk } with d(xnk , y) < k for each k N. Therefore lim xnk = y.
k

() Exercise. 

Exercise 2.1.13

(a) Let {xn } be a sequence in a metric space (X, d) which converges to p. Put S = {xn | n N}.
50

Show that

(i) S is a bounded subset of X,

(ii) p is an adherent point of S.

(b) Let {an } be a sequence of complex numbers, and let p be a complex number. Show that
lim an = p if and only if lim Re(an ) = Re(p) and lim Im(an = Im(p), where Re(z) and
n n n
Im(z) denote, respectively, the real part and the imaginary part of z.

(c) Let {an } be a sequence in a metric space (X, d). Show that an a lim d(an , a) = 0.
n

2.2 Cauchy sequences and completeness


When dealing with sequences it is sometimes helpful to be able to determine that a sequence
converges without knowing the point it converges to. In this section we discuss, among other
things, a necessary condition for a sequence in a metric space to converge.

Definition 2.2.1 A sequence {xn } in a metric space (X, d) is called a Cauchy sequence if for
every > 0 there exists a natural number n0 such that if m, n n0 then d(xm , xn ) < .

Theorem 2.2.2 Every convergent sequence is a Cauchy sequence.

Proof : Let {xn } be a convergent sequence in a metric space (X, d). Put p = lim xn . Let > 0.

Since xn p and 2 > 0, there exists an index n0 such that d(xk , p) < 2 whenever k n0 . Now
for any m, n n0 we have

d(xm , xn ) d(xm , p) + d(p, xn ) < + = . 
2 2
The converse of the above result does not hold. That is, there are Cauchy sequences which do
not converge.

1
Examples 2.2.3 Let X be R\{0} with the usual metric. Put xn = n for n = 1, 2, . . .. Then
1
{xn } is a Cauchy sequence. Indeed, given > 0, choose n0 N so that n0 < 2. Then for
1
m, n n0 we have d(xm , xn ) = | m n1 | 1
m + n1 < . The sequence however does not converge
in X.

There are however metric spaces in which all Cauchy sequences converge.
51 MAT3711/1

Definition 2.2.4 A metric space (X, d) is said to be complete if every Cauchy sequence in X
converges to a point of X.

Note that to a point of X" in the above definition is a tautology because when we say a se-
quence converges in a metric space we mean it converges to a point in the space. Complete
metric spaces abound as we shall see. Let us agree to say a sequence {xn } is bounded to mean
the set {xn | n N} of its terms is bounded. Then we have the following result.

Theorem 2.2.5 Every Cauchy sequence is bounded.

Proof : Let {xn } be a Cauchy sequence in a metric space (X, d). Choose n0 N such that
d(xn , xm ) < 1 for all m, n n0 . Thus for n n0 we have d(xn , xn0 ) < 1. Put

r = max{d(x1 , xn0 ), . . . , d(xn0 1 , xn0 )}.

One checks easily that the ball B(xn0 , r + 1) contains all the terms of the sequence. So the set
{xn | n N} is bounded, as required. 

We saw in the previous section that it is possible for a sequence which does not converge to have
a convergent subsequence. This phenomenon is not possible with Cauchy sequences.

Theorem 2.2.6 If a Cauchy sequence has a convergent subsequence then it is itself convergent
to the limit of that subsequence.

Proof : Let {xn } be a Cauchy sequence in a metric space (X, d) and let {xnk } be a subsequence

of {xn } with lim xnk = p, say. Given > 0 find n0 N such that d(xn , xm ) < 2 whenever
k

m, n n0 . Since xnk p, we can find N such that n > n0 and d(xnc , p) < . Then if
2
n n0 we have
d(xn , p) d(xn , xnc ) + d(xnc , p) < . 

Theorem 2.2.7 Let {xn } be a Cauchy sequence in a metric space (X, d). If the range of the
sequence, i.e. {xn | n N}, is finite, then {xn } converges.

Proof : Since {xn | n N} is finite, there is x X such that xn = x for infinitely many n N.
If x is the only member of the range, then clearly xn x. If the range has other elements, put
= min{d(a, b) | a and b are in the range of {xn } and a = b}. Then > 0 because it is the
52

minimum of a finite set of positive real numbers. Since {xn } is a Cauchy sequence, there is an
index n0 such that d(xn , xm ) < whenever m, n n0 . As xn and xn0 are in the range of {xn }
and d(xn , xn0 ) < , we deduce that xn = xn0 for all n n0 . But xk = x for infinitely many
integers k; so it follows that xm = x for all m n0 . Thus xn x. 

Theorem 2.2.8 The Euclidean space Rm is complete for each m N. Also, C with the usual
metric is complete.

Proof : Let {xn } be a Cauchy sequence in Rm and let S = {xn | n N} be its range. If S is
finite then by Theorem 2.2.7 we have that {xn } converges. If S is infinite, then, by Theorem
2.2.5, S is a bounded infinite subset of Rm . So by the Weierstrass-Bolzano Theorem we have
that S has an accumulation point, say p. But clearly p is then a cluster point of the sequence
{xn }. Thus, by Theorem 2.1.12, {xn } has a subsequence that converges to p. So by Theorem
2.2.6 it follows that xn p. We leave it as an exercise to show that C is complete using the
fact that R is complete. 

We saw in Example 2.2.3 that R\{0}, although being a subspace of the complete metric space R,
is not complete. In the next result we state a theorem which tells us precisely when a subspace
of a complete metric space is complete. First some terminology.

Definition 2.2.9 Let (X, d) be a metric space and S be a subset of X. We say S is complete
if the metric space (S, dS ) is complete.

Thus a subset S of a metric space (X, d) is complete if and only if whenever {an } is a Cauchy
sequence of points in S, then there exists p S such that an p.

Theorem 2.2.10 Let (X, d) be a complete metric space and A X. Then A is complete if and
only if A is closed.

Proof : () Let p A. We must show that p A. By Theorem 2.1.8 there is a sequence {xn }
in X whose terms are points of A and such that xn p. But then {xn } is a Cauchy sequence
with terms in A. So by the hypothesis there exists q A such that xn A q, where A means
convergence in (A, dA ). But clearly xn A q implies xn q, where the latter convergence is in
(X, d). Now the uniqueness of limits (Theorem 2.1.6) yields p = q. So p A.
53 MAT3711/1

() Let {xn } be a Cauchy sequence in (A, dA ). Then {xn } is a Cauchy sequence in (X, d) and
therefore converges in X. Thus there is a point p X such that xn p. By Theorem 2.1.8
we have that p A. But A is closed, so A = A. Therefore {xn } converges in (A, dA ). So A is
complete. 

Theorem 2.2.11 A metric space (X, d) is complete if and only if for every decreasing sequence

F1 F2 . . . of nonempty closed subsets of X with lim diam(Fn ) = 0, the intersection Fn
n n=1
is nonempty.

Proof : () Let (X, d) be a complete metric space and {Fn } be a sequence of closed subsets of
X such that
Fn+1 Fn for each n and lim diam(Fn ) = 0.
n

For each n N choose a point xn Fn . The sequence {xn } of these points is a Cauchy sequence
for if > 0 is given, then choosing n0 N such that diam(Fn0 ) < we obtain the following: For
n, m n0 , xn and xm are in Fn0 since Fk+1 Fk for every k. Thus d(xn , xm ) diam(Fn0 ) < .
Since (X, d) is complete there is a point p X such that xn p. For each k N the sequence
xk , xk+1 , . . . is a sequence of points of Fk which converges to p. Thus p F k = Fk since Fk is

closed. So p Fn .
n=1

() Let (X, d) be a metric space which satisfies the stated condition. Let {xn } be a Cauchy
sequence in (X, d). For each n N put Fn = {xn , xn+1 , . . .}. Then each Fn is closed, Fn+1 Fn
for each n and diam(Fn ) 0 as n (verify each of these assertions, recalling Theorem 1.4.7,

among other results). Thus there is a point x Fn . Check that x = lim xn . 
n+1 n


Remark. Note that, with regard to the above result, Fn actually contains one point. Indeed
n=1
if x = y were both in the intersection then we could not have diam(Fn ) 0.

An important property of complete metric spaces is that countable unions of nowhere dense sets
have empty interiors. This property is often formulated in a dierent but equivalent form and
is known as the Baire Category Theorem.

Theorem 2.2.12 (Baire Category Theorem) Let {Gn | n N} be a collection of dense open

subsets of a complete metric space (X, d). Then Gn is also dense.
n=1
54


Proof : Put H = Gn . By Theorem 1.3.30 it suces to show that H intersects every nonempty
n=1
open set. So let x X and r > 0. Since G1 is dense there exists y1 B(x, r) G1 . Because G1
is open there exists a real number with 0 < < 1 such that

y1 B(y1 , ) G1 B(x, r).


Put r1 = 2 and note that 0 < r1 < 1 and

B(y1 , r1 ) {z X | d(z, y1 ) r1 } G1 B(x, r).

Now use the same argument, with B(x, r) replaced by B(y1 , r1 ), to get a point y2 in G2 B(y1 , r1 )
1
and a real number r2 such that 0 < r2 < 2 and

B(y2 , r2 ) G2 B(y2 r1 ).

Proceeding this way we obtain a sequence of point y1 , y2 . . . of X, and a sequence of positive


1
real numbers r1 , r2 , . . . such that 0 < rn < n and

B(yn+1 , rn+1 ) Gn+1 B(yn , rn ) for each n 1.

We claim that {yn } is a Cauchy sequence. To see this let > 0 be given and choose n0 N
1
such that n0 < . If m, n n0 (say m n), then B(ym , rm ) B(yn , rn ) and so ym B(yn , rn )
which implies that
1 1

d(ym , yn ) < rn < < .
n n0
Since (X, d) is complete, there exists y X such that yn y. Next we show that y belongs to
all the sets G1 , G2 , . . ., which will complete the proof. Let k N. For any integer m k we
have ym B(yk , rk ). So the sequence yk , yk+1 , . . . is a sequence in B(yk , rk ) that converges to

y. Thus, by Theorem 2.1.8, y B(yk , rk ) Gk . So y Gn . 
n=1

Remark. The Baire Category Theorem can be used to prove the rather startling fact that:
There is a continuous function f : [0, 1] R which is dierentiable at no point of [0, 1].

Exercise 2.2.13

(a) Let {an } and {bn } be Cauchy sequences in a metric space (X, d). For each n N let
sn = d(an , bn ). Show that {sn } converges. Hint: Use

d(an , bn ) d(an , am ) + d(am , bm ) + d(bm , bn )

to show that |sn sm | is small for large m and n.


55 MAT3711/1

(b) Let (X, d) be any discrete metric space and let {xn } be a Cauchy sequence in (X, d). Show
that there exists n0 N such that for each n n0 , xn = xn0 . Deduce that every discrete
metric space is complete.

(c) Let {an } be a sequence of real numbers. Suppose that


1
|an | < 2 and |an+2 an+1 | |a2n+1 a2n | for all n 1.
8
Prove that {an } converges.

(d) Let (X, d) be a metric space and A be a dense subset of X such that every sequence in A
has a limit in X. Prove that (X, d) is complete.

(e) Let (X, d) be a metric space. Define d : X X R by d(x, y) = min{d(x, y), 1}. Show
that:

(i) d is a metric on X, and

(ii) (X, d) is complete if and only if (X, d) is complete

(f) This exercise shows that if the condition lim diam(Fn ) = 0 is dropped in the statement of
n
Theorem 2.2.11, then the result might be false. For each n N let Fn = [n, ). Verify
that

(i) Each Fn is closed

(ii) Fn+1 Fn for every n 1



(iii) Fn = .
n=1

2.3 Sequences with terms in R


Having seen above that R is complete, it is now possible to conclude that certain sequences
converge even if one has no idea what the limit is. This is accomplished by showing a given
sequence to be a Cauchy sequence.

n1
Example 2.3.1 For each n N let xn = 1 12 + 13 . . .+ (1)n . We claim that {xn } converges.
Of course you know from MAT2613 that {xn } converges because xn is the nth partial sum of
an alternating series with terms approaching zero. Let us however use the Cauchy condition to
1
establish this. Let > 0. Choose n0 N such that n0 < . For m, n n0 ; say m > n, we have

1 1 1 1
|xm xn | = | + . . . | < (why?)
n+1 n+2 m n
56

So {xn } is a Cauchy sequence in the complete metric space R, so {xn } converges.

Example 2.3.2 Let an be a sequence of real numbers such that |an+2 an+1 | 12 |an+1 an |
for all n 1. We show that {an } converges by showing that it is a Cauchy sequence. For each
n 1 let bn = |an+1 an |. Then 0 bn+1 bn /2 and so, by induction, bn+1 b1 /2n . So if
m > n we have
m1
am an = (ak+1 ak );
k=n

hence
m1
|am an | bk
k=n
1 1
bn (1 + + . . . + m1n )
2 2
< 2bn
1
b1
2n1

Thus given > 0, we can find n0 N such that if m, n n0 we have |am an | < .

We now discuss the important notions of limit superior and limit inferior for a sequence of real
numbers. By the extended real line we mean the set R = R {, +}, consisting of R
and two symbols and +, satisfying the following properties:

ER1 : < x < + for all x R.

ER2 : x + (+) = + and x + () = , for all x R.

ER3 : x (+) = and x () = +, for all x R.

ER4 : x/(+) = x/() = 0, for all x R.

ER5 : x (+) = + and x () = , for all x R with x > 0.

ER6 : x (+) = and x () = +, for all x R with x < 0.

ER7 : (+) + (+) = (+) (+) = () () = +, and


() + () = (+) () = .

Note that we do not define


0 (), + (+), , + (+).

57 MAT3711/1

It should be emphasized that the principal reason for introducing the symbols and + is
one of convenience. For instance if a subset A of R is not bounded above, we write sup A = +
when working within R . Similarly, we write inf A = if A is a subset of R which is not
bounded below. In this way every nonempty subset of R has a sup and an inf in R .

Definition 2.3.3 Let {an } be a sequence of real numbers. We say {an } diverges to +, and
write an + or lim an = + or simply lim an = +, if for every real number M > 0
n n
there exists n0 N such that an > M for all n n0 . We say {an } diverges to , and write
an or limn an = or simply lim an = , if for every real number M < 0 there
n
exists n0 N such that an < M for all n n0 .

Exercise: Show that lim an = lim(an ) = +.


n n

Definition 2.3.4 Let {an } be a sequence of real numbers. Let

E = {x R | ank x for some subsequence {ank }}.

Note that E contains all subsequential limits of {an } and possibly and +. Then sup E
(in R ) is called the limit superior of {an }, and inf E (in R ) is called the limit inferior of
{an }. The limits superior and inferior of a sequence {an } are respectively denoted by lim sup an
n
and lim inf an .
n

1
Example 2.3.5 Let an = n for n odd and an = n+1 for n even. Let us compute lim sup an and
n
lim inf an . Writing out the terms of the sequence yields 1, 13 , 3, 15 , 5, 17 , 7, 19 , . . . . So {an } has only
n
two subsequences that converge" in R ; namely, {a2n1 } and {a2n } with a2n1 + and
a2n 0. Therefore lim sup an = + and lim inf an = 0.
n n

In what follows we shall say a sequence is bounded above, bounded below, or bounded according
whether its range has the corresponding property. We leave the proof of the following theorem
as an exercise.

Theorem 2.3.6 Let {an } be a sequence of real numbers. Then

(a) lim supan = + {an } is not bounded above.


n

(b) lim inf an = {an } is not bounded below.


n

(c) lim inf an lim supan .


n n
58

Let us note the following about sequences of real numbers. If {xn } is a sequence with a finite
range, then there is an integer k N such that xn = xk for infinitely many indices n. Convince
yourself that this is true. It should be obvious though, for if the range is {xm1 , . . . , xmp }, say,
and for each mi there were only finitely many integers n with xn = xmi , then for n large enough
we would have xn {xm1 , . . . , xmp }, a contradiction since {xm1 , . . . , xmp } is the range of the
sequence.

Thus if a sequence (in any metric space) has a finite range then it has a convergent subsequence.
[Compare this with Theorem 2.2.7.] On the other hand, if {xn } is a bounded sequence of real
numbers with an infinite range, then, by the Weierstrass-Bolzano Theorem, the sequence has a
cluster point, and hence a convergent subsequence. This discussion actually proves that:

Theorem 2.3.7 Every bounded sequence of real numbers has both the limit superior and the
limit inferior.

In fact, as will be apparent later, every sequence of real numbers (whether bounded or not),
has both the limit superior and the limit inferior. We now give a characterisation of the limit
superior. In the exercises you will give a similar characterisation for the limit inferior.

Theorem 2.3.8 Let {an } be a sequence of real numbers which is bounded. Then lim sup an =
n
(where R) if and only if satisfies the following properties:

(a) For every > , an < for all but a finite number of integers n. (That is, n0 N such
that n n0 an < .)

(b) For every < , an > for infinitely many integers n. (Thai is, m N, n > m such
that an > .)

Proof : () Let E = {x R | ank x for some subsequence {ank }}. Note that, since the
sequence is bounded, all subsequential limits are real numbers. Let s be an upper bound for the
sequence; i.e. an s for every n. Now if (a) were false, then there would exist > such that
an s for infinitely many integers n. But then by the discussion preceding Theorem 2.3.7
we would have that {an } has a subsequential limit in the interval [, s]. This would contradict
the fact that lim sup an = sup E since < . So (a) is true. Next, if (b) were false then there
n
would exist < such that an for all but a finite number of integers n. The sequence
59 MAT3711/1

would thus have no subsequential limit bigger than . Thus we would have = sup E < ;
an absurdity.
() It is clear from (a) that no number strictly bigger than can be a subsequential limit of
{an }. Therefore lim sup . If we assume that lim sup < , then a contradiction arises as
n n
follows: Say lim sup an = . Then let r be such that < r < . Now in the interval [r, ] there
n
are terms an for infinitely many integers n by (b). So {an } has a subsequential limit in [r, ];
i.e. a subsequential limit p with lim sup an < p. This is impossible. 
n

Theorem 2.3.9 Let {an } be a sequence of real numbers and put lim sup an = . Then there is
n
a subsequence {ank } such that ank .

Proof : If = + then {an } is not bounded above; hence the result follows. If = then
{an } has no subsequential limit. So, for any real number M , an M for at most a finite number
of integers n. Thus there is a natural number n0 such that an < M for all n n0 . So an .
If R, then {an } has at least one subsequential limit. Let E be the set of all subsequential
limits of {an }. Then E is not empty. But by Theorem 2.1.10 we have that E is closed, so that
E = E. One checks easily that for any subset S of R which is bounded above, sup S S. So
E, and we are done. 

Theorem 2.3.10 Let {an } be a sequence of real numbers. Then we have:

(a) lim inf = lim sup (an )


n n

(b) {an } converges if and only if lim supan and lim inf an are both real numbers and are equal.
n n
In that case lim an = lim inf an = lim supan .
n n n

(c) an + if and only if lim inf an = lim supan = +.


n n

(d) an if and only if lim supan = .


n

Proof : Exercise. 

Exercise 2.3.11

(a) Formulate (and prove) a characterisation of lim inf an which is similar (actually dual) to
n
that of lim sup an given in Theorem 2.3.8.
n
60

(b) Let {an } and {bn } be sequences of real numbers. Suppose there exists n0 N such that
an bn for all n n0 . Prove that lim inf an lim inf bn and lim inf an lim inf an .
n n n n

(c) Let {an } and {bn } be sequences of real numbers that are bounded below. Show that
lim sup(an + bn ) lim sup an + lim sup bn .
n n n

(d) Given a sequence {an } of real numbers which is bounded above, let un = sup{ak | k n}.
Show that:

(i) {un } is a decreasing sequence of real numbers.

(ii) lim sup an = lim un .


n n

(e) Formulate (and prove) a result for lim inf an similar to that in (d).
n

(f) Find lim sup an and lim inf an if:


n n
n 1
(i) an = (1) (1 + n ),
(ii) an = n2 sin2 ( n
2 ),
n n
(iii) 3 [ 3 ],
where [x] denotes the greatest integer x.
61 MAT3711/1

Chapter 3

Compactness

3.1 Compact sets


Let (X, d) be a metric space and A X. A collection C of subsets of X is a cover of A if
V
A C. If C is a cover of A we also say C covers A. Given a cover C of A, we say D C
is a subcover if D also covers A. An open cover of A is a cover C such that each set in C
is open (in X). A finite subcover of C is a subcover which consists of finitely many sets from C.

Definition 3.1.1 Let (X, d) be a metric space and K X. We say K is compact if every
open cover of K has a finite subcover.

Thus a set which is not compact is one which has an open cover that has no finite subcover.

Example 3.1.2 Consider R with the usual metric. The set (0, 1] is not compact. To prove this
we must produce an open cover of (0, 1] which has no finite subcover. Let
  
1
C= , 2 | n = 1, 2, . . . .
n
Then:

(i) C covers (0, 1]. To see this let x (0, 1]. Choose m N such that m > x1 . Then 1
m < x < 2,
which implies that
  ^  
1 1
x ,2 ,2 .
m n=1
n

(ii) C is an open cover since each set in C is open.

(iii) C has no finite subcover; for if it had then there would be finitely many natural numbers
n1 , n2 , . . . , nk such that
   
1 1
(0, 1] ,2 ,2 .
n1 nk
1 
Letting m = max{n1 , . . . , nk } would yield (0, 1] m , 2 , which is false as, for instance,
1
 1

m+1 is in (0, 1] but not in m , 2 .
62

Example 3.1.3 R is not compact. The open cover {(, n) | n N} of R has no finite
subcover.

Example 3.1.4 In any metric space every finite set is compact. Indeed, let F be a finite subset
of a metric space (X, d) and let C be an open cover of F . Say F = {x1 , . . . , xk }. For each
i = 1, . . . , k choose Ci C such that xi Ci . Then F C1 Ck ; so that {C1 , . . . , Ck } is
a finite subcover.

Example 3.1.5 Let (X, d) be a discrete metric space. A subset of X is compact if and only
if it is finite. The one implication follows from the foregoing example. Conversely, let K X
be compact. Then C = {{x} | x K} is an open cover of K since in a discrete metric space
singletons are open. By compactness C has a finite subcover, i.e. there are points x1 , . . . , xm
such that K {x1 } {xm } = {x1 , . . . , xm }. So K is finite.

Notice that we defined compactness for subsets of metric spaces. If (X, d) is a metric space and
X (as a subset of X) is compact, we say (X, d) is a compact metric space. Thus (X, d) is
compact if and only if every open cover of X has a finite subcover. As we saw above, R is not
a compact metric space.

Recall from Exercise 1.2.24(a) that if x and y are distinct points in a metric space (X, d), then
there are disjoint open sets U and V such that x U and y V . In fact, for r = d(x, y) > 0 the
   
balls B x, 2r and B y, 2r are open and disjoint. So we can say: in a metric space, distinct points
can be separated by open sets. We will show below that if K is a compact set and p a point not in
K, then we can separate p and K by open sets. That is, we can find disjoint open sets U and V
such that p U , K V . From this result we will be able to deduce that compact sets are closed.

Theorem 3.1.6 Let (X, d) be a metric space, K X be compact and p X\K. Then there
exist disjoint open sets U and V such that K U and p V .

Proof : For each x K, x = p. So choose (for each x K) disjoint open sets Ux and Vx such
that x Ux and p Vx . Then C = {Ux | x K} is an open cover of K. Since K is compact
there is a finite subcollection {Ux1 , Ux2 , . . . , Uxm } of C which covers K. Since Uxi Vxi = for
each i = 1, 2, . . . , m, we have that
63 MAT3711/1

m
^ m
_
U= Uxi and V = Vxi
i=1 i=1

are disjoint open sets with K U and p V . 

Theorem 3.1.7 Every compact subset of a metric space is closed and bounded.

Proof : Let K be a compact subset of a metric space (X, d). We first show that K is closed. We
do this by showing that X\K is open. So let p X\K. By Theorem 3.1.6 find disjoint open
sets U and V such that K V and p U . Since U V = , we have that U X\V . Since
K V , we have that X\V X\K. Thus U is an open set with p U X\K. This shows
that every point of X\K is an interior point; so X\K is open and hence K is closed.

Next, to prove that K is bounded, let p be any point in K. The balls B(p, 1), B(p, 2), . . . form
an open cover of K, for if x K and m is a natural number with m > d(p, x), then x B(p, m).
By compactness a finite number of these balls also cover K. So K is contained in the one with
the largest radius. 

The converse of Theorem 3.1.7 does not hold. That is, if a subset of a metric space is closed
and bounded, it does not follow that it is compact. Take for instance any infinite set with the
discrete metric. In such a metric space all subsets are closed and bounded, but the compact
ones are the finite ones as we saw above.

Theorem 3.1.8 Closed subsets of compact metric spaces are compact.

Proof : Let K be a closed subset of a compact metric space (X, d). Let C be an open cover of K.
V
Since K is closed, X\K is open. Since X = K (X\K) and K C, we have that C {X\K}
is an open cover of X. Since X is compact, there are finitely many sets C1 , C2 , . . . , Cm in C such
that
X = C1 C2 Cm (X\K).

Since K (X\K) = , it follows that K C1 Cm . 

Theorem 3.1.9 Let K be a compact subset of a metric space (X, d). Then every infinite subset
of K has an accumulation point in K.
64

Proof : Suppose, by way of contradiction, that T is an infinite subset of K and that no point of
K is an accumulation point of T . Then for each x K there is a real number rx > 0 such that
the ball B(x, rx ) does not contain a point of T \{x}. Now {B(x, rx ) | x K} is an open cover
of K; so by compactness there are points x1 , . . . , xm in K such that

K B(x1 , rx1 ) B(xm , rxm ).

Thus, also
T B(x1 , rx1 ) B(xm , rxm );

so that m
^
T = (T B(xi , rxi )) .
i=1
But each of the sets
T B(x1 , rx1 ), . . . , T B(xm , rxm )

is finite (in fact each T B(xi , rxi ) is either empty if xi T, or only {xi } if xi T ); thus T
is finite, contradicting the fact that T is infinite. 

Theorem 3.1.10 Let A be a collection of compact subsets of a metric space (X, d) such that
W
the intersection of every finite subcollection of A is nonempty. Then A is nonempty.

Proof : Note first that each set in A is nonempty. Fix C0 A and put B = {X\A | A A}.
Then each member of B is open since compact sets are closed. Now assume that no point of
C0 belongs to every member of A. So for each x C0 , there exists A A such that x A.
Thus x X\A. This shows that B covers C0 . Since C0 is compact, there are finitely many sets
A1 , A2 , . . . , Am in A such that C0 (X\A1 ) (X\Am ). But

(X\A1 ) (X\Am ) = X\(A1 Am ).

So we have that C0 A1 Am = , which contradicts the hypothesis that the intersection


of any finitely many sets in A is nonempty. 

Corollary 3.1.11 Let K1 , K2 , . . . be nonempty compact subsets of a metric space such that
W

K1 K2 . . .. Then Kn is nonempty.
n=1

Proof : This follows immediately from Theorem 3.1.10 because if Kn1 , Kn2 , . . . Knp are finitely
many sets from the collection {Kn }, then Kn1 Knp = Km where m = max{n1 , . . . , np }
and of course Km = . 
65 MAT3711/1

Remark. The foregoing result should be compared with that in Theorem 2.2.11.

Exercise 3.1.12

(a) Show that the intersection of an arbitrary collection of compact sets is compact.

(b) Show that the union of a finite number of compact sets is compact.

(c) Let (X, d) be a metric space and S M X. Show that S is compact in (X, d) if and
only if it is compact in (M, dM ).

(d) Show that if K is compact and C is closed, then K C is compact.

3.2 Characterisations of compactness


The main goal of this section is to give various characterisations of compactness. Some of these
are, in certain instances, easier to use to determine compactness than the definition.

Let us note the following fact which we could also have raised when we were discussing counta-
bility. Let A be an infinite set. We argue that A contains a countably infinite subset. To
construct such a set, let x1 be any element of A. Then A\{x1 } is not empty, so we can choose
x2 A\{x1 }. Thus x1 = x2 . Now suppose elements x1 , x2 , . . . , xk in A which are mutually
distinct have been chosen. Then since A is infinite, A\{x1 , x2 , . . . , xk } is not empty. So we can
choose an element xk+1 in A\{x1 , x2 , . . . xk }. Thus, by induction, we have a countably infinite
set {x1 , x2 , . . .} A.

The foregoing discussion also shows that in every infinite set there is a sequence whose range
is infinite. We observed earlier that a point p in a metric space is an accumulation point of a
set {x1 , x2 , . . .} if and only if the sequence {xn } has a subsequence converging to p. To be sure,
this we did not express explicitly, but it certainly follows from results pertaining accumulation
points of sets and cluster points of sequences. Better yet: prove the claim.

Definition 3.2.1 Let (X, d) be a metric space. We say:

(a) (X, d) has the Bolzano-Weierstrass property if every infinite subset of X has an ac-
cumulation point.

(b) (X, d) is sequentially compact if every sequence in X has a convergent subsequence.

We are going to show that each of the notions defined above is equivalent to compactness.
66

Theorem 3.2.2 A metric space is sequentially compact if and only if it has the Bolzano-
Weierstrass property.

Proof : () This follows from the discussion preceding Definition 3.2.1.


() Let (X, d) be a metric space with the Bolzano-Weierstrass property. Let {xn } be a sequence
in X. If the sequence has a term which is infinitely repeated, then it has a constant subsequence,
and this subsequence clearly converges. If no term of the sequence is infinitely repeated, then
the set {xn | n = 1, 2, . . .} is infinite. So by the hypothesis this set has an accumulation point.
This accumulation point is a subsequential limit of {xn }. 

Note that, in the terminology of this section, Theorem 3.1.9 says that if (X, d) is a compact
metric space, then (X, d) has the Bolzano-Weierstrass property.

An alert student will at this juncture have noticed what, at first glance, might appear to be a
discrepancy between the discussion of compactness in Section 3.1 and that of sequential com-
pactness defined above. In Section 3.1 we defined (and consistently spoke about) compactness
of a subset of a metric space. Here we have defined subsequential compactness of a metric space.
Keeping in mind that we are aiming to prove that these concepts are equivalent, and that (in
view of Exercise 3.1.12(c)) a subset S of a metric space (X, d) is compact, as a subset of (X, d),
if and only if the metric space (S, dS ) is compact, we see that in fact there is no discrepancy in
the two discussions. However if you insist on pedantry, you can rewrite Section 3.2 along the
lines of Section 3.1 by defining sequential compactness and the Bolzano-Weierstrass property for
subsets of metric spaces as follows:

Definition 3.2.1 A subset S of a metric space (X, d) is said to:

(a) have the Bolzano-Weierstrass property if every infinite subset of S has an accumula-
tion point which is an element of S.

(b) be sequentially compact if every sequence in (X, d) whose terms are elements of S has
a subsequence which converges to a point of S.

Now that we have tied loose ends, let us continue with the task at hand; namely, that of proving
that compactness sequential compactness the Bolzano-Weierstrass property.
67 MAT3711/1

So far we have achieved the following:

(i) sequential compactness having the Bolzano-Weierstrass property, and

(ii) compactness sequential compactness.

We are thus left with showing that compactness is implied by sequential compactness. This
we shall do in stages, introducing two important notions in the process; that of the Lebesgue
number and that of total boundedness.

Definition 3.2.3 Let C be an open cover of a metric space (X, d). A real number > 0 is called
a Lebesgue number for the cover if every subset of X with diamter less that is contained in
one of the open sets in C. That is, for every A X with diam(A) < , there exists C C such
that A C.

Theorem 3.2.4 Let (X, d) be a sequentially compact metric space. Then every open cover of
X has a Lebesgue number.

Proof : Let C be an open cover of X. Let us call a subset of X big if it not contained in any of
the sets in C. Notice that if X has no big subsets, then any r > 0 is a Lebesgue number for C.
So let us assume that X has some big sets. Let

= inf{diam(B) | B is a big subset of X}.

Then clearly, 0 +. Notice that if = +, then each big subset of X is unbounded.


So for any real number r > 0, if diam(S) < r, then S is bounded and hence not big. So S is
contained in some member of C. Thus any r > 0 is a Lebesgue number for C. So we investigate
the case 0 < +. If we can show that > 0, then a real number r with 0 < r <
will be a Lebesgue number for C because then any set with diameter less than r is not big.
We prove by contradiction that > 0. Suppose, by way of contradiction, that = 0. Since
singletons have diameter 0, we have that every big set has more than one element. Since = 0
1
we have that for each n N, there is a big set Bn such that 0 < diam(Bn ) < n. For each
n = 1, 2, . . ., choose a point xn Bn . Since (X, d) is sequentially compact, the sequence {xn }
has a subsequence {xnk } which converges to some point x X. Since C covers X, there exists
a set C C such that x C. Since C is open, there exists a real number > 0 such that
2
B(x, ) C. Since xnk x, we can find N such that n > and such that if k then
d(xnk , x) < 2 . Now xnc Bnc and diam(Bnc ) < 1
nc < 2 ; so Bnc B(x, ), for if z Bnc then
68


d(z, x) d(z, xnc ) + d(xnc , x) < 2 + 2 . But now this implies that Bnc C since B(x, ) C;
which contradicts the fact that Bnc is big. 

Definition 3.2.5 A metric space (X, d) is totally bounded if for every real number r > 0, the
cover {B(x, r) | x X} always has a finite subcover.

What is the relationship between boundedness and total boundedness? We show that the latter
implies the former; but not conversely.

Theorem 3.2.6 Every totally bounded metric space is bounded.

Proof : Let (X, d) be a totally bounded metric space. Then the cover {B(x, 1) | x X} has
a finite subcover, say, B(x1 , 1), . . . , B(xm , 1). Let r = max{d(xi , xj ) | i, j = 1, . . . , m}. Let
x, y X. Then x B(xi , 1) and y B(xj , 1) for some i, j {1, . . . , m}. Therefore

d(x, y) d(x, xi ) + d(xi , xj ) + d(xj , y) < r + 2.

So r + 2 is a real number such that d(x, y) r + 2 for all x, y in X. 

Example 3.2.7 Let X be a set of real numbers with the metric d(x, y) = min{|x y|, 1}. Then
(X, d) is bounded since d(x, y) 1 for all x, y X. We show that the cover {B(x, 12 ) | x X}
has no finite subcover. For any finite set F X, let p be the largest element of F (recall that
V  1
X = R). If q F , then d(p + 1, q) = 1, so that p + 1 B x, 2 . Hence X is not totally
xF
bounded.

Theorem 3.2.8 Every sequentially compact metric space is totally bounded and complete.

Proof : Let (X, d) be a sequentially compact metric space. To show completeness, let {xn } be a
Cauchy sequence in X. By sequential compactness, {xn } has a convergent subsequence. Thus,
by Theorem 2.2.6, {xn } converges. Next we show that (X, d) is totally bounded. Suppose this
were not the case. Then there is a real number r > 0 such that the cover {B(x, r) | x X} has no
finite subcover. Let x1 be any point of X. Since X B(x1 , r) there is a point x2 X\B(x1 , r).
Also, {B(x1 , r), B(x2 , r)} does not cover X, so there is a point x3 X\(B(x1 , r) B(x2 , r)).
Continuing, by induction, we obtain a sequence {xn } in X such that

d(xi , xj ) r for all i = j.


69 MAT3711/1

So {xn } cannot have a convergent subsequence; contrary to the hypothesis. 


We have finally arrived at a stage where we can complete the loop.

Theorem 3.2.9 Every sequentially compact metric space is compact.

Proof : Let (X, d) be a sequentially compact metric space and let C be an open cover of X.
By Theorem 3.2.4, C has a Lebesgue number . Put = 3 ; and by Theorem 3.2.8 find points
x1 , x2 , . . . , xm in X such that

X = B(x1 , ) B(xm , ).

Now, for each k = 1, 2, . . . , m we have diam(B(xk , )) 2 < . So by the definition of a


Lebesgue number, for each k = 1, . . . , m there is a set Ck C such that B(xk , ) Ck . Thus
{C1 , C2 , . . . , Cm } is a finite subcover of C. Hence (X, d) is compact. 

From Theorem 3.2.8 we deduce that every compact metric space is totally bounded and com-
plete. The converse is also true.

Theorem 3.2.10 A metric space is compact if and only if it is totally bounded and complete.

Proof : The one implication has already been observed. We show the converse by proving that if
(X, d) is totally bounded and complete then it has the Bolzano-Weierstrass property. Of course
if X is a finite set there is nothing to prove. Let A be an infinite subset of X. For each n N
use total boundedness to find a finite set Fn X such that
^  
1
X= B x, .
n
xFn

Since A is infinite and the finitely many balls B(x, 1) for x F1 cover A1 , there is a point
 
x1 F1 , such that A B(x1 , 1) is infinite. Next, since the finitely many balls B y, 12 for y F2
cover the infinite set A B(x1 , 1), there is a point x2 F2 such that
 
1
A B(x1 , 1) B x2 ,
2
 
is infinite. Inductively, we choose xn Fn for all n N such that A B(x1 , 1) B xn , n1
1
is infinite for each n N. This implies that d(xm , xn ) < m + n1 < n2 for all m, n N with m > n
 1
  
since there is a point y B xm , m B xn , n1 . Therefore {xn } is a Cauchy sequence in (X, d)
and hence converges to some point x X. For each n N we have that d(xn , x) < n2 ; so
   
1 3
B xn , B x, .
n n
70

 
So B x, n3 contains infinitely many points of A. Since 3
n 0 as n , it follows that x is an
accumulation point of A. 

We now introduce another kind of compactness which is also equivalent to compactness for met-
ric space.

Definition 3.2.11 A metric space is countably compact if every countable cover has a finite
subcover. A subset S of a metric space is countably compact if it is countably compact as
a subspace. That is, if every countable open cover of S (by sets open in the containing metric
space) has a finite subcover for S.

Theorem 3.2.12 A metric space is compact if and only if it is countably compact.

Proof : If every cover has a finite subcover, then every countable cover has a finite subcover. So
compactness clearly implies countable compactness.
Conversely, let (X, d) be a countably compact metric space. We show that (X, d) has the
Bolzano-Weierstrass property. Let A be an infinite subset of X. Then A contains a countably
infinite subset B = {xn | n = 1, 2, . . .}. We may assume that xm = xn for m = n. Suppose B
has no accumulation point. Then for each n N, Cn = {xi | i n} is a closed set (why?). For
each n N, put Dn = X\Cn . Note that if Cn1 , . . . , Cnk are finitely many of the sets Cn , then
Cn1 Cnk = since this intersection is simply Cm for m = max{n1 , n2 , . . . , nk }. This
means that we cannot find finitely many of the sets Dn that cover X. So {Dn | n N} is not
a cover of X, for if it were it would have a finite subcover. So there is a point z in X such that
z Dn for each n = 1, 2, . . .. Thus z Cn for each n = 1, 2, . . .. As Cn B for each n, this
means that z = xk for some k N. But xk Ck+1 since Ck+1 = {xk+1 , xk+2 , . . .}. We therefore
have a contradiction. So the assumption that B has no accumulation point is false; hence B has
an accumulation point. As B A, it follows that A also has an accumulation point. Therefore
(X, d) is compact. 

Remark. In case you could not work out why each of the sets Cn in the proof above is closed;
here is the reason. If B has no accumulation point, then Cn (being a subset of B) also has no
accumulation point. But we showed in Chapter 1 that a set which has no accumulation point is
closed.
71 MAT3711/1

Now let us collect all the characterisations of compactness into a single theorem stated for arbi-
trary subsets of a metric space. All the concepts, but total boundedness, in question (namely:
compactness, countable compactness, sequential compactness, having the Bolzano-Weierstrass
property and completeness) have been defined for arbitrary subsets of metric spaces. In exactly
the same way, if S is a subset of a metric space (X, d), we say S is totally bounded if the
metric space (S, dS ) is totally bounded. This is so if and only if for every > 0 there is a finite
T X (yes, T subset of X) such that for each a S there exists t T with d(a, t) < . You
will prove this in the exercise below. Our characterisation theorem states that:

Theorem 3.2.13 Let S be a subset of a metric space. Then the following statements are
equivalent:
(a) S is compact.
(b) S is countably compact.
(c) S is sequentially compact.
(d) S has the Bolzano-Weierstrass property.
(e) S is totally bounded and complete.

We have seen when we were studying completeness that the Euclidean space Rm (m N) seems
to have special properties which are not enjoyed by all metric space. It should therefore come as
no surprise that there is a very transparent characterisation of compactness for subsets of Rm .
By transparent we mean expressible in terms of basic properties.

Let us elaborate. It is no doubt easier to intuit or get a geometric feel of boundedness than
that of total boundedness. In other words it is easier to determine boundedness than total
boundedness. For instance we can tell immediately that the interval (3; 22], as a subset of R,
is bounded. But is it totally bounded?

We showed that in any metric space total boundedness always implies boundedness, but not
conversely. In the Euclidean space Rm it turns out that boundedness is equivalent to total
boundedness. Now this is a very nice result because if we want to check whether a subset of
Rm is compact or not we need to determine whether it is bounded and complete. But wait: is
completeness of a subset of Rm easy to check? Indeed. Since Rm is complete (Theorem 2.2.8)
we know that a subset of Rm is complete if and only if it is closed (Theorem 2.2.10). So to check
compactness for subsets of Rm we need only check boundedness and closedness. Nothing could
be easier.
72

Well, let us prove these assertions. Recall that a closed cell in Rm is a set of the form
I = [a1 , b1 ] . . . [am , bm ] where the ai and bi are real numbers with ai bi for all i. By
a cube of edge ( > 0) we mean a cell I as above where bi ai = for all i = 1, 2, . . . , m. A
cube in R is an interval, a cube in R2 of edge
is simply a square with sides each of length .

Now convince yourself that in Rm , a cube of edge has diameter equal to m. Revisit Lemma
1.5.3.

Lemma 3.2.14 Let S be a subset of a metric space (X, d). Then the following conditions are
equivalent.
(a) S is totally bounded, i.e. the metric space (S, ds ) is totally bounded as per Definition 3.2.5.
(b) For every > 0, there exists a finite T X such that for each x S there is y T with
d(x, y) < .
(c) For every > 0, there is a finite cover of S by subsets of X each with diameter less than .
(d) For every > 0, there is a finite partition of A consisting of sets each of diameter less than .

Proof : Exercise. 

While studying the proof of the next result, it is recommended that you draw a diagram in R2
to help you visualise the argument.

Theorem 3.2.15 A subset of Rm is totally bounded if and only if it is bounded.

Proof : Let B Rm be bounded and choose any point b = (s1 , . . . , sm ) in B. Since B is bounded
we can find a real number r > 0 such that B B(b, r). Then, as one checks easily, B is also
contained in the cube
C = [s1 r, s1 + r] [sm r, sm + r]

with edge 2r. If each of the intervals [si r, si + r], i = 1, 2, . . . , m, is partitioned into k
subintervals of equal length, then the various products of all these subintervals (one from each
partition) provide a partition of C into k m subcubes, each of edge 2r/k, and therefore diameter

2r m/k. Thus, given > 0, if we choose k so large that 2r m/k < , then the partition of C
mentioned above provides a finite cover for B satisfying condition (c) of Lemma 3.2.14. 
73 MAT3711/1

Corollary 3.2.16 (Heine-Borel Theorem) A subset of Rm is compact if and only if it is closed


and bounded.

We end this section by stating, without proof, the following characterisation of total bounded-
ness.

Theorem 3.2.17 A subset S of a metric space is totally bounded if and only if every infinite
sequence in S has a Cauchy subsequence.

Exercise 3.2.18

(a) Prove Lemma 3.2.14.

(b) Let A be a subset of a metric space (X, d). Suppose A is not compact.  Showthat there
W
are closed sets F1 F2 F3 . . . such that Fk A = for all k and Fk A = .
k=1

(c) A collection A of subsets of a set is said to have the finite intersection property provided
W
that if B is a finite subcollection of A then B is nonempty. Prove that a metric space
(X, d) is compact if and only if every countable collection of closed subsets of X with the
finite intersection property has a nonempty intersection.

(d) A metric space is called separable if it has a countable dense subset. For instance R is
separable since Q is a countable dense subset of R.

(i) Given an example of a metric space which is not separable.

(ii) Prove that every totally bounded metric space is separable.

(e) Show that a closed subset of a complete metric space is compact if and only if it is totally
bounded.

(f) Let A be a subset of a metric space. Show that A is totally bounded if and only if A is
totally bounded.
74
75 MAT3711/1

Chapter 4

Connectedness

4.1 Connected Sets


From an intuitive point of view, a connected metric space (or subset of a metric space) is one
which consists of a single piece. The main goal of this section is to make this notion precise.

Definition 4.1.1 A metric space (X, d) is disconnected if there are two nonempty, disjoint
open sets A, B X such that X = A B. A connected metric space is one which is not
disconnected. A subset S of a metric space (X, d) is connected (respectively, disconnected) if the
metric space (S, dS ) is connected (respectively, disconnected).

Recall that a subset A of a set X is a proper subset in case A = and A = X. Recall also that
in any metric space, the (improper) subsets and the whole space are both open and closed.
The following easy result characterises connectedness in terms of proper subsets that are both
open and closed.

Theorem 4.1.2 A metric space is connected if and only if it has no proper subset which is both
open and closed. Equivalently, a metric space is disconnected if and only if it has a proper subset
which is both open and closed.

Proof : Let (X, d) be a metric space which has a proper subset S, say, which is both open and
closed. Put T = X\S. Since S is a proper subset of X, T is nonempty. Thus S and T are
nonempty, disjoint open subsets of X with X = S T . Thus (X, d) is disconnected.
Conversely, suppose (X, d) is a disconnected metric space. Find two nonempty, disjoint open
sets A and B such that X = A B. Now, since A B = X and A B = , it follows that
B = X\A. Because A is open, we have that B is closed. So B is both open and closed. Since
A is a proper subset of X and B = X\A, we have that B is a proper subset of X. Thus B is a
proper subset of X which is both open and closed. 
76

Now let us examine a little closer what it means for a subset of a metric space to be connected
(or disconnected, if you please). Recall that if (X, d) is a metric space and S X, then a subset
A of S is open in S if and only if A = S U for some set U which is open in X.

Theorem 4.1.3 Let (X, d) be a metric space and S X. Then S is disconnected if and only if
there are open subsets U and V of X (i.e. U and V open in X) such that S U V , S U = ,
S V = and S U V = .

Proof : () Suppose S is disconnected. This means that the metric space (S, dS ) is discon-
nected. So there are two disjoint, nonempty open subsets A and B (i.e. A and B open in S) of
S such that S = A B. So by Theorem 1.2.15, we can find open subsets U and V of X such
that A = S U and B = S V . It is easy to check that U and V satisfy the properties stated
in the theorem.

() Suppose S, U and V are as stated. Put G = S U and H = S V . Then clearly, G and H


are two nonempty, disjoint open subsets of S with S = G H. So (S, dS ) is disconnected. 

Another result which provides a useful criterion for connected subsets of metric spaces is facili-
tated by the following definition.

Definition 4.1.4 Let (X, d) be a metric space and let A and B be subsets of X. We say A and
B are separated in X if A B = B A = . If A and B are separated in X and X = A B,
we say A and B form a separation of X.

Theorem 4.1.5 A metric space is connected if and only if it cannot be expressed as a union of
two nonempty sets that are separated in it.

Proof : Let (X, d) be a disconnected metric space. Then there are nonempty, disjoint open sets
U and V such that X = U V . Then U and V are also closed, so U V = U V = and
V U = V U = . Therefore U and V are separated in X. This proves that if a metric
space cannot be expressed as a union of two nonempty sets that are separated in it, then it is
connected.
Conversely, suppose (X, d) is a metric space such that X = A B for some nonempty subsets
A and B with A B = B A = . Since X = A B and A B = , it follows that A A.
77 MAT3711/1

Hence A = A, showing that A is closed. Similarly, B is closed. But then from X = A B and
A B = A B = , we deduce that A = X\B, and therefore A and B are also open. Thus X
is not connected. 

It follows from the proof of Theorem 4.1.5 that if A and B form a separation of a metric space
(X, d), then A and B are both open and closed.

Theorem 4.1.6 Let A be a subset of a metric space (X, d). Then A is connected if and only
if it cannot be expressed as the union of two nonempty subsets of X that are separated in X.
Equivalently, A is disconnected if and only if A can be expressed as the union of two nonempty
subsets of X that are separated in X.

Proof : () Let us first note that if U A, then clA U , the closure of U in the metric space
(A, dA ) is U A, where U denotes the closure of U in (X, d). That is, clA U = U A. Prove this.
Now suppose A = U V where U V = V U = , and U and V are nonempty. We must show
that A is disconnected; that is, we must show that the metric space (A, dA ) is disconnected.
Note that U and V are subsets of A. Now

clA (U ) V = (A U ) V = A (U V ) =

since U V . Similarly, clA (V ) U = . So by Theorem 4.1.5 we have that A is disconnected.

() Let A be a disconnected subset of (X, d). We must express A as a union of two nonempty
subsets of X that are separated in X. By Theorem 4.1.3 there are nonempty, open subsets U
and V of X such that

A U V, A U = , A V = , and A U V = .

From A U V we deduce that A = A (U V ) = (A U ) (A V ). If we can show that


A U and A V are separated in X, we shall be done. We claim that A U (A V ) = .
If not, then there is a point p such that p A U and p A V . But p A U means that
every nbd of p has a nonempty intersection with A U . From p A V we have that p V .
As V is open, V contains a ball with centre p. But this ball is then a nbd of p, and hence has
a nonempty intersection with A U . So also V (A U ) = ; which is a contradiction. This
proves the claim. Similarly, A V (A U ) = . Thus A U and A V are separated in X.

78

Remark. Note that in proving that A U (A V ) = in the foregoing theorem, we have in


a way proved Exercise 1.3.29(n).

The next result shows that if A and B form a separation of a metric space (X, d), then every
connected subset of X is contained either in A or in B.

Theorem 4.1.7 Let A and B be a separation of a metric space (X, d). If H is any connected
subset of X, then H A or H B.

Proof : Since A and B are open in X, both H A and H B are open in (H, dH ). If both
H A and H B were nonempty, then H would be disconnected. Therefore either H A =
or H B = . Thus H A or H B. 

Theorem 4.1.8 Let A be a connected subset of a metric space (X, d). If B is a subset of X such
that A B A, then B is connected. So, in particular, closures of connected sets are connected.

Proof : If B is not connected, there are two open subsets U and V of X such that

B U V, B U = , B V = and B U V = .

Since A is connected and A U V , A U V = , it follows that A U = or A V = . Say


A U = . Then A V . Since U is open and A U = , we have that A U = . [Recall Ex-
ercise 1.3.29(n)]. As B A, we then have B U = , contradicting the fact that B U = . 

We end by showing that connected subsets of R have a particularly simple structure, namely:
they are intervals. Recall that intervals are subsets of R of the form

(, ), (, b), (, b], (a, ), [a, ), (a, b), [a, b), (a, b], [a, b], where a, b R with a b.

We note that the interval [a, b] with a = b is the singleton {a}. Clearly all singletons (in any
metric space) are connected.

Notice (actually, prove) that a subset S of R is not an interval if and only if there are real
numbers a, b, c such that a < b < c and a, c S, b S.

Theorem 4.1.9 A subset of R is connected if and only if it is an interval.


79 MAT3711/1

Proof : We shall show that a subset of R is disconnected if and only if it is not an interval. So
let S be a subset of R which is not an interval. Find real numbers a, b, c such that a < b < c,
a, c S and b S. It is easy to check that S (, b) and S (b, ) are separated in R and S
equals, their union. So S is disconnected.
Conversely, let S be a disconnected subset of R. Then there are nonempty subsets A and B of
R which are separated in R and such that S = A B. Pick x A and y B. Then x = y since
AB = [separated sets are disjoint]. Then x < y or y < x. Say x < y. Put = sup(A[x, y]).
Then A, and therefore B since A B = . As x A [x, y] we have that x . So
x y. But = y as y B, so x < y. Now if A, then x < < y and A,
then B. Hence there is a real number z such that < z < y and z B. So x < z < y and
z S. So again we have that S is not an interval. 

Exercise 4.1.10

(a) Give an example of a disconnected set whose closure is connected.

(b) Show that a metric space is connected if and only if every nonempty proper subset has
nonempty boundary.

(c) Show that a metric space (X, d) is connected if and only if for every two points in X there
is some connected subset of X which contains both the points.
80
81 MAT3711/1

Chapter 5

Continuity

5.1 Continuous functions


Since defining metric spaces, we have thus far studied only properties of metric spaces and sub-
sets of metric spaces. In this chapter we study maps between metric spaces, and also properties
of metric spaces that are preserved or reflected by continuous functions.

In elementary calculus you learnt that a function f : R R is continuous at x0 R if for every


> 0 there is a > 0 such that

|f (x) f (x0 )| < whenever |x x0 | < .

Now recalling that in R |w z| is the distance between w and z it is clear that the notion of
continuity can be defined for functions that map metric spaces into metric spaces in such a way
that in the case of real-valued functions of a real variable, the definition reduces to the one above.

Definition 5.1.1 Let (X, d) and (Y, ) be metric spaces. A function f : X Y is said to be
continuous at a point x0 X if for every > 0, there exists > 0 such that (f (x), f (x0 )) <
whenever d(x, x0 ) < . We say f is continuous on a set S X if it is continuous at every
point of S. In particular, if we simply say f is continuous, we mean it is continuous at every
point of its domain X.

So a function f : (X, d) (Y, ) is continuous at x0 X if

> 0 > 0 such that d(x, x0 ) < (f (x), f (x0 )) < ,

that is, if whenever we are given > 0 we can find > 0 (this depends on x0 and ) such
that for each x X with d(x, x0 ) < , we have that (f (x), f (x0 )) < . This is equivalent to:
For every > 0 there exists > 0 such that if x B(x0 , ) then f (x) B(f (x0 ), ), that is
f [B(x, )] B(f (x0 ), ).
82

Continuity at a point is characterisable in terms of sequences as stated in the following theorem:

Theorem 5.1.2 Let (X, d) and (Y, ) be metric spaces and f : X Y be a function. Then f
is continuous at x0 X if and only if for every sequence {xn } in X which converges to x0 , the
sequence {f (xn )} in Y converges to f (x0 ).

Proof : () Let f be continuous at x0 and {xn } be a sequence in X with xn x0 . We must


show that f (xn ) f (x0 ). Let > 0 be given. Since f is continuous at x0 , we can find > 0
such that if x X with d(x, x0 ) < , then (f (x), f (x0 )) < . Since xn x0 and > 0, we can
find n0 N such that d(xn , x0 ) < whenever n n0 . Now if n n0 then d(xn , x0 ) < , which
implies that (f (xn ), f (x0 )) < . This proves that {f (xn )} converges to f (x0 ) in (Y, ).

() Now let f have the stated property. We must prove that f is continuous at x0 . We do it
by contradiction. So suppose f is not continuous at x0 . Then there exists > 0 for which no
1
> 0 works. So for each k N there is at least one point xk of X for which = fails.
k
This means that
1
d(xk , x0 ) < but (f (xk ), f (x0 )) .
k
1
Now the sequence {xn } has the property that d(xn , x0 ) < for every n; hence xn x0 . But
n
clearly the sequence {f (xn )} does not converge to f (x0 ). This violates the hypothesis. 

The foregoing result is helpful in showing that a function is not continuous at some point p. All
we need do is find a sequence {xn } with xn p but for which f (xn ) f (p).

The following result characterises continuity in terms of open sets.

Theorem 5.1.3 Let (X, d) and (Y, ) be metric spaces and let f : X Y be a function. Then
f is continuous if and only if f 1 [G] is open in X whenever G is open in Y . That is, a function
is continuous if and only if pre-images of open sets are open sets.

Proof : () Let f be continuous and G Y be open. We must show that f 1 [G] is open. If
f 1 [G] = , there is nothing to prove. So assume f 1 [G] = . Let z f 1 [G]. Then f (z) G,
and so there exists > 0 such that B(f (z), ) G since G is open. Since f is continuous
at z, there exists > 0 such that (f (x), f (z)) < whenever x X with d(x, z) < . Thus
f [B(z, )] B(f (z), ), whence we get B(z, ) f 1 [B(f (z), )] f 1 [G]. Therefore f 1 [G] is
open.
83 MAT3711/1

() Let f have the stated property. We must show that f is continuous. Let p be an arbitrary
point of X and > 0. The ball B(f (p), ) is open in Y , so by the hypothesis we have that
f 1 [B(f (p), )] is open in X. Since f (p) B(f (p), ), we have that p f 1 [B(f (p), )]. So,
since f 1 [B(f (p), )] is open, there exists > 0 such that B(p, ) f 1 [B(f (p), )]. Then
f [B(p, )] B(f (p), ); that is if d(x, p) < then (f (x), f (p)) < . So f is continuous at p,
and hence on X. 

Corollary 5.1.4 A function f : (X, d) (Y, ) is continuous if and only if f 1 [C] is closed in
X whenever C is closed in Y .

Proof : Recall that f 1 [X\C] = Y \f 1 [C]. 

Next we show that the composite of continuous functions is continuous.

Theorem 5.1.5 Let (X1 , d1 ), (X2 , d2 ) and (X3 , d3 ) be metric spaces. Let f : X1 X2 and
g : f [X1 ] X3 be functions. If f is continuous at a point p X1 and g is continuous at f (p),
then the composite function g f is continuous at p.

Proof : Let > 0 be given. Since g is continuous at f (p), there exists > 0 such that

d3 (g(y), g(f (p))) < if d2 (y, f (p)) < and y f [X1 ].

Since f is continuous at p, there exists > 0 such that

d2 (f (x), f (p)) < if d1 (x, p) < .

It then follows that


d3 ((g f )(x), (g f )(p)) <

if d1 (x, p) < . So g f is continuous at p. 

Notice that continuity of g f follows readily from Theorem 5.1.3 since, if f and g are continuous
on their domains and G is an open set, then

(g f )1 [G] = f 1 [g 1 [G]].

Example 5.1.6 Let X be any metric space and Y be any discrete metric space. Any function
f : Y X is continuous. To show this, let G be an open subset of X. Then f 1 [G] is open in
Y since every subset of Y is open. So f is continuous.
84

Example 5.1.7 Let X be the reals with the usual metric and Y be the reals with the discrete
metric. Let f : Y X be given by f (x) = x. Then of course, as seen above, f is continuous.
Now let g : X Y be given by g(x) = x. Notice that, as functions from the set R, g is the same
function as f . But now g is not continuous since, for instance, {3} is open in Y but g 1 [{3}]
is not open in X. In fact there is no single point of X where g is continuous. Indeed, for any
p X, there is no > 0 such that g[B(p, )] B p, 12 .

By now it should be clear that the discrete metric spaces are a source of all kinds of bizarre"
examples.

Exercise 5.1.8

(a) Let (X, d) be a metric space, and let f, g : X R be continuous functions.

(i) Prove that the function h : X R defined by h(x) = f (x) + g(x) is continuous.

(ii) Prove that the function h : X R defined by h(x) = f (x) g(x) is continuous.

(iii) Prove that the function h : X R defined by h(x) = max{f (x), g(x)} is continuous.

(iv) Prove that the function h : X R defined by h(x) = min{f (x), g(x)} is continuous.

(v) Let A = {x X | g(x) = 0}. Prove that the function h : X\A R defined by
h(x) = f (x)/g(x) is continuous.

(b) Let (X, d) and (Y, ) be metric spaces, f and g be continuous functions mapping X into
Y and A be a subset of X such that f (a) = g(a) for all a A. Show that f (x) = g(x) for
all x A.

(c) Let f : R R be a continuous function and define g : R R2 by g(x) = (x, f (x)). Prove
that g is continuous.

(d) Let f : [a, b] R be a continuous function and suppose f (x) = 0 for each x Q [a, b].
Show that f (x) = 0 for all x [a, b].

m
(e) Recall that every rational number x in [0, 1] can be written as x = n where m, n N and
have no common divisor = 1. Now let f : [0, 1] R be defined by

x, if x is irrational
f (x) =
1 , if x = m , where m and n have no common divisor = 1.
n n

Show that f is continuous only at irrational numbers in [0, 1].


85 MAT3711/1

(f) Let (X, d) and (Y, ) be metric spaces. Prove that for a function f : X Y the following
are equivalent:

(i) f is continuous

(ii) f (A) f [A] for every A X

(iii) f 1 [int(B)] int(f 1 [B]) for every B Y .

(g) Let f be a function mapping a metric space (X, d) into a metric space (Y, ). Prove that
f is continuous if and only if f is continuous on every compact subset of X. [Hint: show
that if xn p in X, then {p, x1 , x2 , . . .} is a compact subset of X].

5.2 Uniform continuity


If a function is continuous on a subset S of a metric space, then for each p S, if > 0 is given
we can find > 0 which depends on and p such that if the distance between a point x of S
and p is less than , then the distance between f (x) and f (p) is less than . In general it is not
possible to find a which depends only on that serves equally well for every point of S. When
this happens, the function is continuous in a special way which is the subject of this section.

Definition 5.2.1 Let f : (X, d) (Y, ) be a function between metric spaces. We say f is
uniformly continuous on a subset A of X if for every > 0 there exists > 0 such that if x
and w are any two points of A with d(x, w) < , then (f (x), f (w)) < .

Let us reiterate the dierence between continuity and uniform continuity. If a function is con-
tinuous on a set S, then given > 0 and a point p in S, there is a > 0 that depends on and
p that satisfies the condition of the definition. On the other hand, given > 0, there is a > 0
that depends only on such that works" for all points of S.

Let us also point out that whereas we can talk of continuity at a point, it is meaningless to ask
whether a given function is uniformly continuous at a point. Uniform continuity is a property
of a function on a set.

Clearly, every uniformly continuous function is continuous. Below we give an example of a con-
tinuous function which fails to be uniformly continuous on some set. As in the case of continuity,
if we say a function is uniformly continuous without stating where, we mean it is uniformly
86

continuous on its domain.

1
Example 5.2.2 Let f : (0, 1] R given by f (x) = . Then f is not uniformly continuous (on
x
(0, 1]). To show this, let = 2, and suppose we can find > 0 that satisfies the condition of the
definition. We may assume 0 < < 1. The reason is that if > 0 works, then any < will
3
work as well. Now let x = and w = . Then x, w (0, 1] and |x w| = < . But now
4 4
4 1 3
|f (x) f (w)| = = > 3.

So for these points, |f (x) f (w)| < .
This function is however continuous on (0, 1]. Indeed, let p (0, 1] and > 0 be given. Now
choose a real number r such that 0 < p r < p + r < 1. For any x (0, 1] with |x p| < r we
have r < x p < r, whence 0 < p r < x. Now for such x we have
1 1
|f (x) f (p)| =
x p
1
= |x p|
xp
1 1 1
|x p| since < .
p(p r) x pr
So,
|f (x) f (p)| < if |x p| < p(p r).

Thus choosing = min{r, p(p r)} will ensure that |x p| < |f (x) f (p)| < .


Example 5.2.3 Let f : [1, ) R be given by f (x) = x. This function is uniformly
continuous. To see this, observe that for any two points x, y in [1, ) we have

|f (x) f (y)| = x y
|x y|
=
x+ y
1
|x y|, since x 1 and y 1.
2
Thus |f (x) f (y)| < if 12 |x y| < , i.e. if |x y| < 2. Therefore given > 0, choose = 2.

We now state a result which is useful in showing a given function not to be uniformly continuous.

Theorem 5.2.4 A function f : (X, d) (Y, ) is uniformly continuous if and only if for any
pair of sequences {an }, {bn } in X,

lim d(an , bn ) = 0 lim (f (an ), f (bn )) = 0.


n n
87 MAT3711/1

Proof : () Let f be uniformly continuous. Suppose, by way of contradiction, that the stated
property does not hold. So find two sequences {an } and {bn } in X such that lim d(an , bn ) = 0
n
but lim (f (an ), f (bn )) = 0. Now observe that if a sequence {xn } of nonnegative real numbers
n
does not converge to 0 then there exists > 0 and a subsequence {xnk } such that xnk for
each k = 1, 2, . . ..
So then there are subsequences {ank } and {bnk } of {an } and {bn } respectively such that for
some > 0
(f (ank ), f (bnk )) for each k = 1, 2, . . .

Since lim d(an , bn ) = 0, we have that lim d(ank , bnk ) = 0. Now, by uniform continuity, find
n k
> 0 such that for the above we have

(f (z), f (w)) <

whenever z, w X and d(z, w) < . Thus, from the fact that lim d(ank , bnk ) = 0, we can find
k
N such that
d(anc , bnc ) < .

So then we get
(f (anc ), f (bnc )) < ,

contradicting what we have above.

() Suppose the two-sequences property holds. We must show that f is uniformly continuous.
1
Suppose not. Then there exists > 0 such that for every = (n = 1, 2, . . .) there are points
n
an and bn in X with
1
d(an , bn ) < but (f (an ), f (bn )) .
n
As lim d(an , bn ) = 0 and lim (f (an ), f (bn )) = 0, we have a contradiction. 
n n

Example 5.2.5 The function f : R R given by f (x) = x2 is not uniformly continuous. For
1 1
each n N let an = n and bn = n + . Then lim |an bn | = lim = 0. However
n n n n

2
1
|f (an ) f (bn )| = n2 n +
n
1
= 2+ ,
n2
88

which does not tend to 0 and n . Thus by Theorem 5.2.4 we have that f is not uniformly
continuous.

Exercise 5.2.6

(a) Find an example of a continuous function f and a Cauchy sequence {xn } such that {f (xn )}
is not a Cauchy sequence. Now suppose g : (X, d) (Y, ) is a uniformly continuous
function. Show that if {xn } is a Cauchy sequence in X, then {g(xn )} is a Cauchy sequence
in Y .

(b) Recall that if E is a nonempty subset of a metric space (X, d), then the distance from x
to E, where x X, is defined by d(x, E) = inf{d(x, z) | z E}. Now let f : (X, d) R
be defined by f (x) = d(x, E). Show that f is uniformly continuous.
[Hint: d(x, E) d(x, z) d(x, y) + d(y, z), so that d(x, E) d(x, y) + d(y, E) for all
x, y X. Use this to show that |d(x, E) d(y, E) | d(x, y).]

1
(c) Let f : (0, ) R be given by f (x) = . Use Theorem 5.2.4 to show that f is not
x
uniformly continuous.

(d) Is it true that if a function is uniformly continuous on a set S, then it is uniformly contin-
uous on every subset of S that has at least two points?

5.3 Continuity and Compactness


We saw above that continuity on a set does not necessarily imply uniform continuity on that set.
If however the set on which a function is continuous is compact, then necessarily the function is
uniformly continuous on the set as we prove below.

Theorem 5.3.1 Let f : (X, d) (Y, ) be a function mapping one metric space into another.
Let A be a compact subset of X. If f is continuous on A, then f is uniformly continuous on A.

Proof : Let > 0 be given. By continuity, for each point a A there is a real number ra > 0

such that if x A and d(x, a) < ra , then (f (x), f (a)) < . Let us rewrite this in terms of
2
balls. It says

(f (x), f (a)) < whenever x A B(a, ra ).
2
ra
Now C = {B a, | a A} is an open cover of A. So by compactness there are points
2
89 MAT3711/1

a1 , . . . , am in A and corresponding positive numbers r1 , . . . , rm (with ri = rai ) such that


r1 rm
A B(a1 , ) B(am , ).
2 2
Put = min{ r21 , . . . , r2m }. We shall show that this works. So let z and w be any two points of
A with d(z, w) < . Then there is some ball B(ak , r2k ), say, that contains z. So z AB(ak , r2k ),
and therefore

(f (z), f (ak ) < .
2
Now d(w, ak ) d(w, z)+d(z, ak ) < + r2k rk
2 + r2k = rk . Thus w AB(ak , rk ), and therefore

(f (w), f (ak )) < .
2
Combining () and () yields

(f (z), f (w)) (f (z), f (ak )) + (f (ak ), f (w)) < . 



Example 5.3.2 Let f : [0, ) R be given by f (x) = x. Then f is continuous, and therefore
continuous on [0, 1]. Since [0, 1] is a compact subset of R (and therefore of [0, )), f is uniformly
continuous on [0, 1]. Try to prove this uniform continuity directly from the definition.

Next we show that the image of a compact set under a continuous function is compact.

Theorem 5.3.3 Let f : (X, d) (Y, ) be a function mapping one metric space into another.
If f is continuous on a compact subset K of X, then the image f [K] of K under f is a compact
subset of Y .

Proof : We shall show that every sequence in f [K] has a convergent subsequence. The result
will then follow by Theorem 3.2.13. Let {yn } be a sequence in f [K]. For each n choose xn K
such that yn = f (xn ). We then have a sequence {xn } in K. Since K is compact, there is
a subsequence {xnk } of {xn } and a point x K such that xnk x. Since f is continuous,
Theorem 5.1.2 gives f (xnk ) f (x). But now f (x) f [K]; so the subsequence {ynk } of {yn }
converges to a point of f [K]. This completes the proof. 

In Example 5.1.7, the function f is one-to-one onto and continuous, but f 1 (called g in that
example) is not continuous. As a corollary to Theorem 5.3.3, we show that if the domain of a
one-to-one onto continuous function is compact, then the inverse of the function is also contin-
uous.
90

Corollary 5.3.4 Let (X, d) be a compact metric space and f : (X, d) (Y, ) be a continuous
one-to-one function mapping (X, d) onto a metric space (Y, ). Then f 1 : (Y, ) (X, d) is
continuous.
Proof : We apply Theorem 5.1.3. Let U be an open subset of X. So we must show that
(f 1 )1 [U ] is open in Y . Note that (f 1 )1 [U ] = f [U ]. The set X\U is closed in X, and
therefore compact by Theorem 3.1.8. Thus by Theorem 5.3.3 we have that f [X\U ] is a com-
pact subset of Y , and is therefore closed by Theorem 3.1.7. Since f is one-to-one and onto,
f [U ] = Y \f [X\U ] (verify this). Thus f [U ] is open, as required. 

In the case of real-valued (i.e. mapping into R) functions we can say more.

Theorem 5.3.5 Let f : X R be a continuous real-valued function on a compact metric space


X. Then
(a) f [X] is a closed and bounded subset of R.
(b) There exist points p and q in X such that

f (p) = sup{f (x) | x X} and f (q) = inf{f (x) | x X}.

Proof :

(a) By Corollary 5.3.4 we have that f [X] is compact. So by the Heine-Borel Theorem (Corol-
lary 3.2.16) we have that f [X] is closed and bounded.

(b) If A is a bounded subset of R, then sup A A and inf A A, as you should verify. If
fact we have used one of these facts earlier. So if A is also closed, then inf A A and
sup A A. So then, inf f [X] f [X], which means that f (q) = inf f [X] for some q X.
Similarly for sup f [X]. 

Notice that without compactness, the conclusion above does not always hold. Indeed, if
f : [0, 1) R is the function given by f (x) = x, then sup{f (x) | x [0, 1)} = 1, and there is no
point p [0, 1) such that f (p) = 1.

Exercise 5.3.6

(a) Prove Theorem 5.3.3 using the definition.


91 MAT3711/1

(b) Let K R2 be compact and put A = {x R | there exists y R such that (x, y) K}.
Prove that A is compact. [Suggestion: define f : R2 R by f (x, y) = x. Show f to be
continuous and A = f [K].

(c) Find a continuous function f and a compact set K such that f 1 [K] is not compact. [Hint:
try constant functions.]

(d) Prove that there is no continuous function mapping [0, 1] onto [0, 1).

(e) Let (X, d) be a compact metric space and f : X X be a function such that

d(f (x), f (y)) = d(x, y) for all x, y X.

Prove that:

(i) f is continuous. Is f uniformly continuous?

(ii) f is one-to-one.

(iii) f is onto. [Hint: If w X\f [X], show that there is a real number r > 0 such that
d(w, z) r for all z f [X]. Then use the sequence w, f (w), f (f (w)), . . . to contradict
the compactness of X.]

5.4 Continuity and connected


We start with some bit of propaganda. That Real Analysis is a beautiful piece of Mathematics is
by now clear to you if you have studied and understood the discussion so far, as well as done (or
at least attempted) all the exercises. The results that go counter to intuition, and the ingenuity
required to prove facts which are so obviously true" from a heuristic point of view all add up
to this beauty.

Take for instance the following example. If f is a continuous function that maps [a, b] into R
and the graph of f lies above the x-axis at a and below the x-axis at b, then surely the graph
must cut the x-axis at some point because, being the graph of a continuous function, it consists
of one continuous piece. Who can dispute this? To prove it rigorously takes some doing and
will, among other things, be the goal of this section.

Theorem 5.4.1 Let f : (X, d) (Y, ) be a continuous function mapping one metric space into
another. Let A be a connected subset of X. Then f [A] is connected.
92

Proof : Suppose, by way of contradiction, that f [A] is disconnected. Then, by Theorem 4.1.6,
f [A] = G H where G and H are nonempty subsets of Y that are separated in Y . Put
U = A f 1 [G] and V = A f 1 [H]. Then A = U V , and neither U nor V is empty. Since
U f 1 [G], it is immediate that U f 1 [G]. Since f is continuous and G is closed, we deduce
from U f 1 [G] that U f 1 [G]. Consequently f [U ] G. Now

f [V ] = f [A f 1 [H]]

= f [A] f [f 1 [H]]
= (G H) f [f 1 [H]]

(G H) H since f [f 1 [H]] H

= H.

Therefore

f [U V ] = f [U ] f [V ]

GH
= .

Hence f [U V ] = , which implies that U V = . A similar argument shows that V U = .


So U and V are separated in X. From f [A] = G H we have

A f 1 [G] f 1 [H]

which implies that

U V = (A f 1 [G]) (A f 1 [H])

= A (f 1 [G] f 1 [H])

= A.

By Theorem 4.1.6 this means that A is disconnected. This contradiction completes the proof. 

Theorem 5.4.2 (Intermediate-value Theorem) Let f : S R R be a function mapping a


subset of R into R. If [a, b] is an interval contained in S and f is continuous on [a, b], then for
every real number c between f (a) and f (b), there is a point p in [a, b] such that f (p) = c.

Proof : If f (a) = f (b) there is nothing to prove. So suppose f (a) = f (b). Then either f (a) < f (b)
or f (b) < f (a). Say f (a) < f (b). The other case is similar. We must show that for any c with
93 MAT3711/1

f (a) < c < f (b), there is a point p in [a, b] such that f (p) = c. By Theorem 4.1.9, [a, b] is
connected. So, by Theorem 5.4.1, the set {f (x) | a x b} is connected; and is therefore an
interval containing f (a) and f (b). If we assume that c {f (x) | a x b}, then this set will
be disconnected. Thus c = f (p) for some p [a, b]. 

Corollary 5.4.3 Let f : [a, b] R be a continuous function. If f (a) and f (b) have dierent
signs (i.e. one positive and the other negative), then f (c) = 0 for some c with a < c < b.

Proof : Say f (a) < 0 and f (b) > 0. Then f (a) < 0 < f (b). The result then follows immediately
from Theorem 5.4.2. 

This latter result proves what we stated in the second paragraph of the introductory comments.

The Intermediate-value Theorem, in conjunction with Theorem 5.3.5(b), actually tells us more
about images of closed and bounded intervals under real-valued continuous functions. Let
f : S R R be a function which is continuous on the interval [a, b] S. Put

= inf{f (x) | a x b} and = sup{f (x) | a x b}.

Then and are real numbers since the set {f (x) | a x b} is a compact subset of R and
is therefore bounded (and of course closed). Then clearly we have

{f (x) | a x b} [, ].

But by Theorem 5.3.5(b), , {f (x) | a x b} and by the Intermediate-value Theorem


every real number between and is the image of some point in [a, b]. So we therefore have

[, ] {f (x) | a x b}.

All in all this shows that the image of the closed and bounded interval [a, b] is the closed and
bounded interval [, ].

Let us end by characterising connectedness of arbitrary metric spaces by means of special kinds
of continuous function. Let us denote by D the metric space whose underlying set is {0, 1} and
whose metric is the discrete metric. Recall that every subset of D is open.

Theorem 5.4.4 A metric space (X, d) is connected if and only if every continuous function
f : X D is constant.
94

Proof : () Let (X, d) be connected and f : X D be continuous. We must show that f is


constant. Let A = f 1 [{0}] and B = f 1 [{1}]. Then A and B are open subsets of X with
A B = and X = A B. Since X is connected, we must have A = or B = . Say A = .
Then X = B, i.e. f (x) = 1 for each x X, showing that f is a constant function.

() Let (X, d) be such that every continuous function f : X D is constant. We must show
that X is connected. Suppose not. Then there are two nonempty, disjoint open subsets A and
B of X such that X = A B. Define a function g : X D by

0, if x A
g (x) =
1, if x B.

Since A = and B = , g is not a constant function. The open subsets of D are , {0}, {1}
and {0, 1} whose pre-images under g are , A, B and X respectively. Thus g is continuous. We
therefore have a contradiction. 

Exercise 5.4.5

(a) Prove Theorem 5.4.1 using Theorem 5.4.4 [Hint: Let g : f [A] D be continuous and
define h : A D by h(x) = g(f (x)).]

(b) Let f : [0, 1] [0, 1] be a continuous function. Show that there is a point p [0, 1] such
that f (p) = p. [Hint: Consider the continuous function f (x) x and use the Intermediate-
value Theorem.]

(c) Let A, B R, and suppose A B is connected in R2 . Prove that A is connected. [Sug-


gestion: Show first that the map f : R2 R given by f (x, y) = x is continuous.]

(d) Let {K | } be a collection of nonempty connected subsets of a metric space such


that K = . Use Theorem 5.4.4 to show that K is connected.

5.5 Contraction mappings


If X is a set (not necessarily a metric space) and f : X X is a function mapping X into itself,
we say an element x0 X is a fixed point of f in case f (x0 ) = x0 . In Exercise 5.4.5(b) you
proved (did you?) that every continuous function mapping [0, 1] into itself has a fixed point.
The purpose of this section is to show that certain continuous functions always have fixed points.
95 MAT3711/1

Definition 5.5.1 Let (X, d) be a metric space. A function f : X X is called a contraction


of (X, d) if there is a real number with 0 < < 1 such that d(f (x), f (y)) d(x, y) for all
x, y X.

It is obvious that every contraction is uniformly continuous. For a function f : X X mapping


a set X into itself, let us define f 2 , f 3 , . . . , f n , . . . by f 2 (x) = f (f (x)), f 3 (x) = f (f 2 (x)), etc.
Thus f 2 is the composite of f with itself, and so on.

Theorem 5.5.2 (Banachs Fixed Point Theorem). Let (X, d) be a complete metric space and
T : X X be a contraction. Then T has a unique fixed point.

Proof : Let p be any point of X. We define a sequence {xn } of points of X as follows:

x0 = p, x1 = T (x0 ), x2 = T (x1 ) = T 2 (x0 ), . . . , xn+1 = T (xn ) = T n+1 (x0 ).

We claim that {xn } is a Cauchy sequence. To show this, let , with 0 < < 1, be such that
d(T (x), T (y)) d(x, y) for all x, y X. Note that for any n N we have

d(xn+1 , xn ) = d(T (xn ), T (xn1 ))

d(xn , xn1 )
= d(T (xn1 ), T (xn2 ))

2 d(xn1 , xn2 )

...

n d(T (x0 ), x0 )
= n d(x1 , x0 ).

Thus for m, n N with m > n, say, we have

d(xm , xn ) d(xn , xn+1 ) + d(xn+1 , xn+2 ) + + d(xm1 , xm )

(n + n+1 + + m1 )d(x1 , x0 )

= n (1 + + + mn1 )d(x1 , x0 )
n
< d(x1 , x0 )
1
1
since r = as 0 < < 1. Because
r=0 1
n
d(x1 , x0 ) 0 as n (since 0 < < 1),
1
96

n0
it follows that given > 0 we can find n0 N such that d(x1 , x0 ) < . Thus if m, n n0
1
we have d(xm , xn ) < , as required. Since (X, d) is complete there is a point q X such that
xn q. Because T is continuous, T (q) = T (lim xn ) = lim xn+1 = q. So q is a fixed point of T .
n n
We are left with showing that q is the only fixed point of T . To this end, let z be a fixed point of
T . We must show that z = q. Now T (z) = z and T (q) = q; so d(z, q) = d(T (z), T (q)) d(z, q).
If d(z, q) = 0 we would get 1 , a contradiction. Hence d(z, q) = 0, whence z = q. 

Example 5.5.3 We show that the condition < 1 is essential to ensure that a fixed point exists
and that it is unique. That is, = 1 will not do.

(a) Let f : R R be given by f (x) = x. Then |f (x) f (y)| = |x y| for all x, y R. Note
that every point of R is a fixed point of f .

(b) Let g : R R be given by g(x) = x + 1. Then |g(x) g(y)| = |x y| for all x, y R.


However, g has no fixed point since there is no p R such that p = p + 1.

1 2
Example 5.5.4 Let T (x) = x+ .
2 x
(a) Verify that T maps [1, ) into [1, ).

(b) Show that T is a contraction on [1, )


(c) Find the fixed point of T .

Solution.

(a) We must show that if x 1, then T (x) 1. Note that for any x R

(x 1)2 + 1 0 x2 2x + 2 0
x2 2x + 2
0 if x 1
x
x2 + 2
20
x
2
x+ 2
x
1 2
x+ 1.
2 x

So indeed for x 1 we have T (x) 1. How did we know that starting with (x1)2 +1 0
would work? Actually we started with T (x) 1 and worked backwards.
97 MAT3711/1

(b) By the Mean Value Theorem (from elementary calculus) we have

|T (x) T (y)| = |T (c)||x y|

where c is some point between x and y. Now if x and y are points in [1, ) and c is
between x and y, then

1 2
|T (c)| = 1 2
2 c
1

2
2
since 1 c2 1 for every c in [1, ). Thus, if x, y [1, ) we have

1
|T (x) T (y)| |x y|.
2

So T is a contraction.

(c) To find the fixed point of T we solve the equation

1 2
T (x) = x, i.e. x+ =x
2 x

to obtain x = 2 or 2. Since we want a point in [1, ), we conclude that the fixed

point of T is 2. Note that we know beforehand that T has a fixed point in [1, ) since
[1, ) is a closed subset of a complete metric space R and is therefore itself complete. 

Exercise 5.5.5

(a) Let (X, d) be a complete metric space and T : X X be a function such that T 2 is a
contraction. Show that T has a unique fixed point in X.
+x
(b) Let be a real number with 1 < < 2. Put f (x) = .
1+x

(i) Show that f maps [1, ) into [1, ).

(ii) Show that f is a contraction on [1, ) and find its fixed point.
98
99 MAT3711/1

Chapter 6

Function Spaces
As the title suggests, in this chapter we shall be concerned with spaces the elements of which
are functions. In the first section we show how a special metric can be defined on a set whose
elements are bounded functions. We then explore briefly the completeness of such a space and
indicate how a completion of a metric space can be constructed.

In subsequent sections we concentrate on sequences and series of functions.

6.1 Space of bounded functions


Recall that a subset S of a metric space (X, d) is bounded if there is a real number M such that
d(x, y) M for all x, y S. Now let f : T X be a function mapping a set T (T does not have
to be a metric space) into a metric space (X, d). We shall say f is bounded if the range f [T ]
is a bounded subset of X. If V is a subset of T , we also say f is bounded on V if f [V ] is bounded.

Definition 6.1.1 Let Z be a nonempty set and (X, d) be a metric space. We define the set
B(Z, X) by
B(Z, X) = {f : Z X | f is bounded}.

Thus B(Z, X) is the collection of all bounded functions that map the set Z into the metric space
(X, d). We show how a metric, called the supremum metric, is defined on B(Z, X).

Theorem 6.1.2 Let Z be a nonempty set and (X, d) be a metric space. Define

d : B(Z, X) B(Z, X) R by d (f, g) = sup{d(f (z), g(z)) | z Z}.

Then d is a metric on B(Z, X) called the supremum metric.

Proof : First we must verify that indeed d maps into R; that is, d (f, g) is a real number for
each pair f, g of bounded functions mapping Z into X. To achieve this it suces to show that
the set
A = {d(f (z), g(z)) | z Z}
100

is bounded above. To this end, let z0 be any point of Z. Given any arbitrary point z in Z we
have
d(f (z), g(z)) d(f (z), f (z0 )) + d(f (z0 ), g(z0 )) + d(g(z0 ), g(z)).

Thus if M and N are real numbers with d(f (x), f (y)) M and d(g(s), g(t)) N for all x, y, s
and t in Z; then
d(f (z), g(z)) M + N + d(f (z0 ), g(z0 ))

for all z Z. Thus A is bounded above.


Now check that d (f, g) = 0 if and only if f = g; and that d (f, g) 0. Since

d (f, g) = d (g, f )

easily, it remains to show that the triangle inequality holds.


So let f, g, h B(Z, X). For any z Z we have

d(f (z), g(z)) d(f (z), h(z)) + d(h(z), g(z))

d (f, h) + d (h, g).

Taking suprema over all z Z we obtain

d (f, g) d (f, h) + d (h, g). 

Now we specialise somewhat and look at B(Z, X) where Z is not merely a set, but in fact a
metric space. Among bounded functions mapping Z to X are continuous ones. We shall denote
them by C(Z, X); that is

C(Z, X) = {f : Z X | f is continuous and bounded}.

Just to whet your appetite, in the next chapter we shall consider C(Z, X) where X will either
be R or C. This will allow us to define addition and multiplication on C(Z, X).

Coming back to the discussion at hand, we show that C(Z, X) is a closed subset of B(Z, X),
where the latter is endowed with the supremum metric.

Theorem 6.1.3 Let (Z, ) and (X, d) be metric spaces. Then C(Z, X) is a closed subset of
B(Z, X).
101 MAT3711/1

Proof : Let f B(Z, X) be in the closure of C(Z, X). We show that f C(Z, X), which will
establish the result. Let z0 be an arbitrary element of Z. Since f C(Z, X), given > 0, there
is a function g C(Z, X) such that

d (f, g) < .
3
Consequently,

d(f (z), g(z)) <
3
for every z Z. Since g is continuous at z0 , there exists > 0 such that


d(g(z), g(z0 )) < whenever (z, z0 ) < .
3

Thus, if z is an element of Z with (z, z0 ) < , then

d(f (z), f (z0 )) d(f (z), g(z)) + d(g(z), g(z0 )) + d(g(z0 ), f (z0 ))

< + +
3 3 3
= .

So f is continuous at z0 , and hence on Z. 

Next we show that if (X, d) is a complete metric space, then B(Z, X) is complete with respect
to the supremum metric.

Theorem 6.1.4 Let Z be any nonempty set and (X, d) be a complete metric space. Then
B(Z, X) is complete.

Proof : Let {fn } be a Cauchy sequence in B(Z, X). We must show that there is a function in
B(Z, X) to which this sequence converges. Let > 0 be given. Then there exists n0 N such
that d (fn , fm ) < whenever m, n n0 . If z Z, then

d(fn (z), fm (z)) d (fn , fm ) <

for all m, n n0 . This shows that {fn (z)} is a Cauchy sequence in X. Since (X, d) is complete,
the sequence {fn (z)} converges to some point of X which we denote by xz . As limits are unique,
the relation
f : Z X given by f (z) = xz

defines a function. We thus have a function f : Z X such that lim fn (z) = f (z) for all z Z.
n
We will show that f B(Z, X) and that fn f in B(Z, X).
102

To prove that fn f , let > 0. Choose n0 N such that



m, n n0 d (fn , fm ) < .
4
This is possible since {fn } is a Cauchy sequence. Now take k n0 and z Z. Since lim fn (z) =
n
f (z), we can find m n0 such that

d(fm (z), f (z)) < .
4
We then have

d(fk (z), f (z)) d(fk (z), fm (z)) + d(fm (z), f (z))

d (fk , fm ) + d(fm (z), f (z))



< +
4 4

=
2
As z Z was chosen arbitrarily we conclude that

d(fk (w), f (w)) < for all w Z.
2
Consequently

d (fk , f ) = sup{d(fk (w), f (w)) | w Z} < .
2
It therefore remains to show that f is bounded. With = 1 in the preceding argument, choose
k N such that
d (fk , f ) < 1.

Since fk is bounded, there is a real number M such that

d(fk (s), fk (t)) M for all s, t Z.

Now let p, q Z. Then

d(f (p), f (q)) d(f (p), fk (p)) + d(fk (p), fk (q)) + d(fk (q), f (q))
< 1+M +1
= M + 2.

So f B(Z, X) and {fn } converges to f . So B(Z, X) is complete. 

Corollary 6.1.5 Let (Z, ) and (X, d) be metric spaces. The set C(Z, X) of all bounded con-
tinuous functions mapping Z to X is complete with respect to the supremum metric.
103 MAT3711/1

Proof : Closed subsets of complete metric spaces are complete. 

The result in the foregoing corollary will resurface under a dierent guise in the next chapter.

Let us now turn our attention to the completion. We start with a definition which isolates
special kinds of uniformly continuous functions.

Definition 6.1.6 A function f : (X, d) (Y, ) is said to be isometric if it preserves distances;


that is, if (f (x), f (w)) = d(x, w) for all pairs of points x, w in X. An isometric function which
is onto is called an isometry. If there is an isometry f : X Y we say (X, d) is isometric
to (Y, ).

One checks easily that an isometric function is uniformly continuous and one-to-one. Thus if
f : (X, d) (Y, ) is an isometry, then it is one-to-one onto. In this case it can be checked
easily that f 1 : Y X is also an isometry. Consequently X is isometric to Y if and only if Y
is isometric to X. We can thus speak of two metric spaces that are isometric.

Let us examine closer what it means for two metric spaces to be isometric. If X is isometric
to Y , then the points of these metric spaces can be put into a one-to-one correspondence in
such a way that the distances between pairs of corresponding points are the same. The spaces
therefore dier only in nature (or names) of the points; and, from the point of view of them
being metric spaces, this is often not important. We can thus think of isometric metric spaces
as being identical. Now suppose f : X Y is an isometric function which is not necessarily
onto. Then f [X] is a (proper) subspace of Y , and f viewed as a function f : X f [X] is an
isometry. Thus X is isometric to a subspace of Y . In this case we say X has been imbedded
isometrically into Y . So, to all intents and purposes, we can think of X as being a subspace of Y .

Theorem 6.1.7 Every metric space can be imbedded isometrically into a complete metric space.

Proof : Let (X, d) be a metric space and, as above, B(X, R) denote the set of all bounded
functions mapping X into R. Choose and fix a point x0 X. Given a X, define Ta : X R
by

Ta (x) = d(x, a) d(x, x0 ).


104

We claim that Ta is bounded. From the following inequalities

d(x, a) d(x, x0 ) + d(a, x0 )


d(x, x0 ) d(x, a) + d(a, x0 )

we obtain
d(a, x0 ) d(x, a) d(x, x0 ) d(a, x0 )

which implies that


|Ta (x)| d(a, x0 ) for all x X.

So indeed Ta is bounded. Now define a function g : X B(X, R) by

g(a) = Ta for each a X.

We shall be done if we can show that g is isometric because B(X, R) is complete by Theorem
6.1.4 since R is complete. So let a, b be any two points in X. We must show that

d (Ta , Tb ) = d(a, b).

We have

d (Ta , Tb ) = sup{|Ta (x) Tb (x) | : x X}


= sup{|d(x, a) d(x, b) | : x X}

But now
|d(x, a) d(x, b)| d(a, b)

as can be seen by applying the triangle inequality in the same way we did when showing Ta to
be bounded. As a consequence of all this we have

d (Ta , Tb ) d(a, b).

If we put x = a in |d(x, a) d(x, b)| we get d(a, b). This shows that d(a, b) is an element of
{|d(x, a) d(x, b)| : x X}, which is also an upper bound for this set. Hence d(a, b) is the
supremum of this set; that is, d(a, b) = d (Ta , Tb ) as required. 

Let (X, d) be a metric space and g : X B(X, R) be as in the proof of the above theorem. Then
g[X] is complete being a closed subset of a complete metric space. We call g[X] the completion
of (X, d). In this course we do not pursue the discussion on completions beyond what we have
so far.
105 MAT3711/1

Exercise 6.1.8

(a) Show that an isometric function is uniformly continuous and one-to-one.

(b) Let f : X Y be an isometry. Show that f 1 is also an isometry.

(c) Show that composites of isometries are isometries.

6.2 Pointwise and uniform convergence


In certain branches of Mathematics (for instance in Complex Analysis) it is often desirable to
know when a function defined as a series of functions has certain properties. In Complex Ana-
lysis for instance, one of the main results in the study of analytic functions is that a function is
analytic if and only if it is locally representable as a power series. In this section we consider,
among other things, questions of this nature.

Definition 6.2.1 Let A be a nonempty set, (X, d) be a metric space and, for each n N,
fn : A X be a function mapping A to X. We say the sequence {fn } converges pointwise
to a function f : A X if, for each x A, the sequence {fn (x)} converges to f (x). We then
write fn f pointwise on A, and say f is the pointwise limit of {fn } on A.

So if we have a sequence {fn } of functions and we want to check if fn f pointwise on A, we


must check if for each given a A, lim fn (a) = f (a).
n

Example 6.2.2 For each n N let fn : R R be defined by

x2 + nx
fn (x) = .
n

Determine if {fn } converges pointwise to some function.

Solution: Let a R. Then

a2 + na
lim fn (a) = lim
n n n
a2
= lim a +
n n
= a.

Thus if we define f : R R by f (x) = x, we see that fn f pointwise on R.


106

Example 6.2.3 For each n N define fn : [0, 1] R by



0, if x n1
fn (x) =
nx + 1, if 0 x 1 .
n

Let us determine if {fn } converges pointwise on [0, 1] and if so let us compute the pointwise limit.

Solution: The following diagram might assist to see how we should tackle the problem of com-
puting lim fn (x) for a given x [0, 1].
n

(0,1)

f
2
f1

f3

(1,0)

If x = 0, then fn (x) = fn (0) = 1 for each n. Thus fn (0) 1. If x = 0, then for n0 large enough
we have fn0 (x) = 0. Thus for all n n0 we have fn (x) = 0, and hence fn (x) 0. So {fn } does
converge pointwise on [0, 1] and the pointwise limit is the function f : [0, 1] R defined by

0, if x = 0
f (x) =
1, if x = 0.

Notice that in Example 6.2.3 each function fn is continuous on [0, 1], but the pointwise limit is
not continuous on [0, 1]. Thus pointwise convergence has a deficiency from the point of view
of preserving continuity. This deficiency is remedied by a stronger notion of convergence which
we define next.

Definition 6.2.4 A sequence {fn } of functions mapping a set A to a metric space (X, d) con-
verges uniformly to a function f : A X on A if for every > 0 there exists n0 N such
that n n0 implies
d(fn (x), f (x)) < for every x A.
107 MAT3711/1

In order to see easily the dierence between pointwise and uniform convergence, let us restate
the definition of pointwise convergence in the n0 language. A sequence {fn } converges point-
wise to f on A if and only if given > 0 and given x A, there exists a natural number n0 ,
depending on and x, such that d(fn (x), f (x)) < if n n0 . So, given , each point of A has its
own n0 that works. In the case of uniform convergence, given > 0 we can find n0 that works
for all the points of A. This is reminiscent of the dierence between continuity and uniform
continuity. Clearly, uniform convergence implies pointwise convergence.

A basic property of uniform convergence is its connection with continuity as illustrated in the
following theorem.

Theorem 6.2.5 Let (X, d) and (Y, ) be metric spaces. Let {fn } be a sequence of functions
mapping X to Y . If each function fn is continuous on A X and {fn } converges uniformly on
A to a function f : X Y , then f is continuous on A.

Proof : We shall show that f is continuous at each point of A. So let p A. Given > 0, we
can find n0 N such that if n n0 then

(fn (x), f (x)) <
3
for every x A. This is so since {fn } converges uniformly to f on A. Now fn0 is continuous at
p by the hypothesis. Thus there exists > 0 such that if x A and d(x, p) < , then

(fn0 (x), f (p)) < .
3
Consequently, if x A and d(x, p) < , then

(f (x), f (p)) (f (x), fn0 (x)) + (fn0 (x), fn0 (p)) + (fn0 (p), f (p))

< + +
3 3 3
= .

As p is an arbitrary point of A, we have that f is continuous on A. 

Caution: If the pointwise limit of a sequence of continuous functions is continuous, it does not
follow that the convergence is uniform. However if the pointwise limit of a sequence of continu-
ous functions is not continuous, then the convergence is certainly not uniform.
108

Example 6.2.6 The sequence of functions in Example 6.2.3 does not converge uniformly be-
cause each function in the sequence is continuous but the pointwise limit is not continuous.

Example 6.2.7 Consider the sequence given in Example 6.2.2. Clearly fn is continuous and the
pointwise limit f is also continuous. We show that the convergence is not uniform by exhibiting
an > 0 for which there is no n0 that works. Take = 12 . Suppose there does exist n0 N such
1
that |fk (x) f (x)| < 2 for every k n0 and every x R. Choosing k = n0 and x = n0 we get
n20 + n0 n0
|fn0 (n0 ) f (n0 )| = n0
n0
= n0
1
< .
2
Showing that a sequence {fn } does not converge pointwise to a function f is quite easy. Simply
produce a point x0 for which f (x0 ) = lim fn (x0 ). On the other hand, showing that a sequence
n
of functions does not converge uniformly involves a little more sweat. The following criterion,
which is simply the negation of the statement in the definition, is frequently useful.

Theorem 6.2.8 A sequence {fn } does not converge uniformly on a set E to f if and only if for
some 0 > 0 there are integers n1 < n2 < . . . and points x1 , x2 , . . . in E such that

d(fnk (xk ), f (xk )) 0 for every k = 1, 2, . . .

Proof : Exercise. 

Example 6.2.9 Define fn : (0, 1) R by fn (x) = xn . We show that {fn } converges pointwise
but not uniformly. Indeed, for any x with 0 < x < 1 we have xn 0 as n . Thus fn f
pointwise where f (x) = 0 for all x (0, 1). Now if the sequence did converge uniformly then it
would converge to the pointwise limit. We use Theorem 6.2.8 to show that this is not the case.
1
1
For each k N let nk = k and xk = 2
k
. Then
1
|fnk (xk ) f (xk )| = for each k.
2
Thus {fn } does not converge uniformly on (0, 1).

If a sequence of functions is such that the co-domain of the functions is a complete metric space,
then there is a criterion for uniform convergence which does not require that we know the limit
function. In order to prove this let us note first the following fact.
109 MAT3711/1

Lemma 6.2.10 Let {xn } be a sequence in a metric space (X, d) such that xn p. Suppose q
is a point in X and r is a real number such that for some n0 N, d(xn , q) r for all n n0 .
Then d(p, q) r.

Proof : The set U = {x X | d(x, q) > r} is open. (Verify this.) If we assume that d(p, q) > r,
then p U . Since U is open, B(p, ) U for some > 0. Since xn p, there exists an integer
n1 such that if n n1 then d(xn , p) < ; so that xn B(p, ) U and hence d(xn , q) > r . Now
let k = max{n0 , n1 }. Then k n0 and k n1 . Thus we have d(xk , q) r and d(xk , q) > r,
which is impossible. So the assumption that d(p, q) > r is untenable; hence d(p, q) r. 

Theorem 6.2.11 (Cauchy Criterion for Uniform Convergence) Let {fn } be a sequence of func-
tions mapping a set E into a complete metric space (X, d). Then the sequence {fn } converges
uniformly on E if and only if for every > 0 there is a natural number n0 such that whenever
m, n n0 then d(fn (x), fm (x)) < for all x E.

Proof : () Suppose {fn } converges uniformly on E, and let f be the limit function. Then,
given > 0, there exists n0 N such that if m, n n0 then


d(fm (x), fn (x)) <
2

for every x E. Consequently, if m, n n0 then

d(fn (x), fm (x)) d(fn (x), f (x)) + d(f (x), fm (x))



< +
2 2
= ,

for every x E.

() Suppose the stated condition holds. Given any point x E, the sequence {fn (x)} is a
Cauchy sequence in (X, d) and therefore converges since (X, d) is complete. Define f : E X
by
f (x) = lim fn (x)
n

for each x E. We claim that {fn } converges uniformly on E to f . To show this let > 0 be
given. By the hypothesis we can find n0 N such that


m, n n0 d(fm (x), fn (x)) <
4
110

for each x E. We will show that this n0 satisfies the requirements in the definition of uniform
convergence. So let k n0 and p E. We must show that d(fk (p), f (p)) < . Since


fn (p) f (p) and d(fm (p), fn0 (p)) <
4 3

for each m n0 , we deduce from Lemma 6.2.10 that


d(fn0 (p), f (p)) .
3

Now

d(fk (p), f (p)) d(fk (p), fn0 (p)) + d(fn0 (p), f (p))

+
4 3
< ;

so we are done. 

Because R and C are complete, the above criterion can be used on functions that map into R
or C.
In the case of bounded functions, we show that uniform convergence coincides with convergence
in the supremum metric.

Theorem 6.2.12 Let E be a nonempty set, (X, d) be a metric space and, for each n N,
fn : E X be a bounded function mapping E to X. Then the sequence {fn } converges uni-
formly on E if and only if fn f in B(E, X).

Proof : () Let > 0 be given. By uniform convergence we can find a positive integer n0 such
that

n n0 d(fn (x), f (x)) <
2
for each x E. Thus, for n n0 we have


sup{d(fn (x), f (x)) | x E} < ,
2

that is we have
d (fn , f ) < .

So fn f in B(E, X).
111 MAT3711/1

() Exercise. 

Let us note that if the functions map into R (or C) and they are bounded, then putting

Mn = sup{|fn (x) f (x)| : x E}

we have that {fn } converges uniformly on E to f if and only if Mn 0 as n .

Example 6.2.13 Consider again the sequence of function given in Example 6.2.2. We saw in
Example 6.2.7 that the sequence does not converge uniformly on R. Now let M be a real number
with M > 0. We show that the sequence converges uniformly on [0, M ]. Note that since each
fn is continuous on [0, M ] and [0, M ] is compact, then each fn is bounded on [0, M ]. Now

Mn = sup{|fn (x) f (x)| : 0 x M }


x2
= sup 0xM
n
M2
= 0 as n .
n
Thus {fn } converges uniformly on [0, M ].

x2
Example 6.2.14 Define fn : [0, ) R by fn (x) = . Let us determine whether {fn }
enx
x2
converges uniformly on [0, ). For a given x [0, ), lim nx = 0. So if {fn } is to converge
n e
uniformly, it will converge to the zero function; that is, the function f : [0, ) R given by
f (x) = 0 for all x 0. Let us check if the functions fn are bounded, so that if they are we may
use Theorem 6.2.12. First we note that fn (0) = 0, fn (x) 0 for each x 0 and fn (x) 0. So
the graph of fn (x) looks like this:

By elementary Calculus, fn is bounded and attains its maximum value at the point x0 for which
fn (x0 ) = 0. Put

Mn = sup{|fn (x) 0| : x 0}
= sup{fn (x) : x 0}.
112

To obtain Mn we dierentiate fn to obtain


x(2 nx)
fn (x) =
enx
2
= 0 i x = 0 or .
n
2 4e2
Now fn (0) = 0 and fn = .
n n2

4e2
Therefore Mn = , which tends to zero as n . Hence {fn } converges uniformly on
n2
[0, ) to 0.

Next we turn our attention to series of functions. Let F denote either R or C without specifying
which.

Definition 6.2.15 Let E be a nonempty set and, for each n N, gn : E F be a function


k
mapping E to F. For k N let sk : E F be the k-th partial sum defined by sk (x) = gi (x).
i=1

We say the series gn converges pointwise on E to a function g, and write gn = g
n=1 n=1

pointwise on E, in case the sequence {sn } converges pointwise to g. We say the series gn
n=1

converges uniformly on E to a function g, and write gn = g uniformly on E, in case the
n=1
sequence {sn } converges uniformly on E to g.

The Cauchy criterion for uniform convergence of sequences of function has an obvious extension
to series of functions. We state this result without proof and leave the proof as an exercise.


Theorem 6.2.16 (Cauchy Criterion for Uniform Convergence of Series) The series fn con-
n=1
verges uniformly on E if and only if for every > 0 there exists n0 N such that n n0
implies
n+p
fk (x) <
k=n+1

for each p = 1, 2, . . . and every x E.

Proof : Apply Theorem 6.2.11 to the sequence of partial sums. 

Next we prove a useful test for uniform convergence of series of functions.


113 MAT3711/1

Theorem 6.2.17 (Weierstrass M-test) Let {fn } be a sequence of functions mapping a nonempty
set E into F. Suppose that for each n N there is a nonnegative real number Mn such that

|fn (x)| Mn for every x E. If the series Mn converges, then the series fn converges
n=1 n=1
uniformly on E.

Proof : Let > 0. By Theorem 6.2.16 it suces to find n0 N such that n n0 implies
n+p
fk (x) <
k=n

for every x E. Since Mn converges, the sequence {bn } of partial sums

b1 = M1 , . . . , bn = M1 + . . . + Mn , . . . ,

converges, and is therefore a Cauchy sequence. So, for the given > 0, we can find n0 N such
that if m, n n0 then
|bm bn | < .

Thus if n n0 and p = 1, 2, . . . we have

|bn bn+p | < .

But now
|bn bn+p | = Mn+1 + Mn+2 + . . . + Mn+p .

So, if n n0 and x E we have


n+p n+p
fk (x) Mk < .
k=n+1 k=1

The result then follows from Theorem 6.2.16. 

Caution: A common error is that, given a series fn (x), one finds constants Mn 0 such
that |fn (x)| Mn for each x but Mn diverges, and then incorrectly concludes that the series
fn (x) does not converge uniformly. This is false. All the M-test says is that if we can find
bounds Mn and Mn < , then we know that fn (x) converges uniformly. The examples
below should clarify this further.

zn
Example 6.2.18 Consider the complex series 2
for z C with |z| 1. Let us check if the
n=1 n
series converges. For each n N, |z n | 1 for every z C with |z| 1. Thus
zn 1
2
n2 n
114

1
zn
for every z C with |z| 1. Since n2
< , the Weierstrass M-test applies. Thus 2
n=1 n
converges uniformly on {z C : |z| = 1}.

z n1
Example 6.2.19 Let us now consider the series . The Weierstrass does not apply on
n=1 n
{z C : |z| = 1}. If however r is a real number with 0 < r < 1 and D = {z C : |z| r}, then,
for each n N, we have
z n1
rn1
n
z n1
for every z D. Now r n1 converges. So converges uniformly on D. Incidentally,
n=1 n=1 n
z n1 zn
note that the series consists of derivatives of the terms of the series .
n n

We remark that there are other tests for uniform convergence of series apart from the Weier-
strass M-test. We shall however not discuss them.
In closing, we show that a uniformly convergent series of continuous functions defines a contin-
uous function.

Theorem 6.2.20 Let (X, d) be a metric space and {fn } be a sequence of functions mapping X
n
to F. (Recall: F = R or C). Suppose fn = f uniformly on E X. If each fn is continuous
n=1
on E, then f is continuous on E.

Proof : Clearly each partial sum sn is continuous on E. Thus {sn } is a sequence of continuous
functions that converges uniformly on E to the function f . Therefore by Theorem 6.2.5 the
result follows. 

Example 6.2.21 Every polynomial is continuous on C. Thus the function f given by


zn
f (x) = 2
is continuous on {z C : |z| 1}, in view of Example 6.2.18.
n=1 n

Exercise 6.2.22

(a) For each of the following functions, test whether the given sequences of functions are
pointwise convergent and if they are uniformly convergent on the indicated domains.

xn
(i) fn (x) = ; x 0.
n + xn
2
ex /n
(ii) fn (x) = ; x R.
n
115 MAT3711/1

1 2
(iii) fn (x) = x ; 0 x 1.
n
(iv) fn (x) = x xn ; 0 x 1.

(v) fn (x) = n3 xn (1 x); 0 x 1.


x2n
(vi) fn (x) = ; x R.
1 + x2n

(b) Test the following series for uniform convergence.


xn
(i) n
on [0, ).
n=1 n + x

(ii) an cos nx on R, given that an converges absolutely.
n=1

(c) Let (X, d) and (Y, ) be metric spaces and {fn } be a sequence of functions mapping X to
Y . Let E X and suppose each fn is continuous on E and that {fn } converges uniformly
on E to a function f . Let {pn } be a sequence in E and p be a point in E such that pn p.
Show that lim fn (pn ) = f (p).
n

(d) Let {fn } be a sequence of functions fn : E F. Suppose {fn } converges uniformly on


E to a function f . Show that if each function fn is bounded on E, then there is a real
number M > 0 such that |fk (x)| M for all k N and all x E.

(e) Let {fn } and {gn } be two sequences defined as follows:


1
fn (x) = x 1 +
n

1

n, if x = 0 or x is irrational
gn (x) = 1 a
b+ n, if x is rational, say x = with b > 0


b

and a and b having no common factor.

Let hn (x) = fn (x)gn (x). Show that

(i) both {fn } and {gn } converge uniformly on every bounded interval.

(ii) {hn } does not converge uniformly on any bounded interval.

6.3 Miscellaneous examples


Before we bid this chapter farewell, we give a few examples covering the scope of the chapter.

Example 6.3.1 Let C = {f : [0, 1] R | f is continuous}. Then of course C B([0, 1], R)


since continuous functions on a compact set are bounded. Now let C0 = {f C | f (x) > 0 for
116

all x [0, 1]}. We show that C0 is an open subset of C([0, 1], R) in the supremum metric. Let
f C0 . We will produce a ball B(f, ) in B([0, 1], R) such that B(f, ) C C0 . Since [0, 1] is
compact and f is continuous on [0, 1], f has a minimum value at some point of [0, 1]. That is,
there is a point p [0, 1] such that
f (x) f (p) > 0
f (p)
for all x [0, 1]. Put = . Now if g C([0, 1], R) is such that d (f, g) < , then for any
2
x [0, 1] we have
f (p)
|f (x) g(x)| < = .
2
Consequently
f (p) f (p)
< f (x) g(x) <
2 2
which implies that

f (p)
g(x) f (p)
2
f (p)
=
2
> 0.

So g C0 . This shows that B(f, ) C([0, 1], R) C, as required.

Example 6.3.2 With reference to Example 6.3.1, regard C as a metric space with metric d
given by
d (f, g) = sup{|f (x) g(x)| : x [0, 1]}.

In other words regard C as a subspace of B([0, 1], R). Now we show that the closure of

C0 = {f C | f (x) > 0}

in the space (C, d ) is


D = {f C | f (x) 0}.

This is how we will do it: First we shall show that D is a closed set in C, and secondly we will
show that if f is any point in D, then there is a sequence {fn } in C0 which converges to f . Now
why will this prove that C0 = D? Well, notice that if g C0 , then g(x) > 0 for each x [0, 1].
Hence g(x) 0 for each x [0, 1], implying that C0 D. Thus, if D is closed, then C0 D
since the closure of a set is the smallest closed set containing the set. On the other hand, if for
each f D there is a sequence in C0 converging to f , then D C0 .
117 MAT3711/1

Now to show that D is closed, we show that D contains all its adherent points. Let then f be
an adherent point of D. Find a sequence {fn } in D such that fn f (where this convergence
is with respect to the supremum metric). Now by Theorem 6.2.12 we have that {fn } converges
to f uniformly on [0, 1]. Hence {fn } converges to f pointwise on f . But fn (x) 0 for each n
and each x [0, 1]; so f (x) 0 for each x [0, 1]. This is so since a sequence of nonnegative
reals cannot converge to a negative number. Thus f D as required.
Next let g D. For each n N define fn : [0, 1] R by
1
fn (x) = f (x) + .
n
Then fn C0 for each n and {fn } converges to g uniformly on [0, 1]. We are therefore done.

Example 6.3.3 C is isometric to R2 . Recall that the usual metric on C is given by d(z, w) =
|z w|, where the mod of a complex number + i is | + i| = 2 + 2 . Also recall that
the usual metric on R2 is given by ((a, b), (c, d)) = (a c)2 + (b d)2 . So we must exhibit
an isometry
T : C R2

such that (T (z), T (w)) = d(z, w) for all z, w C. Define T by

T (z) = (Re(z), Im(z))

for each z C. It is routine to check that T is an isometry. So, as we remarked earlier, the
complex plane C and the Euclidean plane R2 are, as metric spaces, essentially the same.

Example 6.3.4 Let (X, d) be a compact metric space and {fn } be a sequence of continuous
functions fn : X R such that:

(a) fn (x) > 0 for each x X and each n.

(b) fn 0 pointwise on X.

(c) fk (x) fc (x) for each x X and all k (i.e. given x X, the sequence {fn (x)}
decreases).

Then fn 0 uniformly on X. To prove this let > 0 be given. We must produce n0 N such
that if n n0 and x X, then |fn (x)| < . For each a X, since fn (a) 0, there exists

a positive integer n(a) such that |fk (a)| < whenever k n(a). Since fm is continuous at a,
2
there is an open ball centered at a, call it Ba,m , such that
118


|fm (y) fm (a)| <
2
for all y Ba,m . The balls Ba,n(a) (for a X) form an open cover of X. Since X is compact,
there are finitely many of these balls, say centered at x1 , x2 , . . . , xc , such that

X = Bx1 ,n(x1 ) Bxc ,n(xc ) .

Put n0 = max{n(x1 ), . . . , n(xc )}. Now let x X and k n0 . Then x Bxp ,n(xp ) for some
p {1, . . . , }; so that

|fn(p) (x) fn(p) (xp )| < .
2
Consequently, using (c) in the hypothesis we obtain

0 fk (x) fn0 (x) fn(p) (x) = fn(p) (xp ) + [fn(p) (x) f (xp )] < + = .
2 2
Therefore |fk (x)| < for k n0 and x X. Thus we have uniform convergence.

Example 6.3.5 In this example we show that if fn f pointwise on [a, b] it does not follow
that
b b
lim fn (x)dx = f (x)dx
n a a

In other words we may have


b b
lim fn = lim fn .
n a a n

We will see in Chapter 8 that in the case of uniform convergence we always have that
b b
lim fn = lim fn .
n a a n

Define
fn : [0, 1] R by fn (x) = n2 x(1 x)n .

Then fn 0 pointwise on [0, 1]. So if f is the limit function we have


b
f (x)dx = 0.
a

But now
b 1
fn (x)dx = n2 x(1 x)n dx
a 0
1
= n2 (1 t)tn dt (substitution)
0
1 1
= n2 tn dt n2 tn+1 dt
0 0
119 MAT3711/1

n2 n2
=
n+1 n+2
n2
= .
(n + 1)(n + 2)

Therefore
1 1
lim fn (t)dx = 1 = lim fn (x)dx.
n 0 0 n
120
121 MAT3711/1

Chapter 7

Linear Analysis
Informally speaking, Linear Analysis is concerned with studying sets (and maps between these
sets) which are endowed with an algebraic structure and a metric. The algebraic structure in
question is one which makes the set a vector space, and the metric is one induced by a general-
isation of the notion of length of a vector in a vector space.

7.1 Basic inequalities


In this section we recall the definition of a vector space and prove some inequalities which will
be useful in the sequel. Throughout this chapter, F will denote either R or C.

Definition 7.1.1 A vector space over F is a quadruple (V, F, +, ) consisting of a nonempty


set V and two operations

+ : V V V (called addition) and : F V V (called scalar multiplication)

satisfying the following properties:

(1) x + y = y + x for all x, y V

(2) x + (y + z) = (x + y) + z for all x, y, z V

(3) there exists a unique element in V, denoted by 0 and called the zero element, such that
x + 0 = x for all x V.

(4) for each x V there corresponds a unique element in V , called the negative of x and
denoted by x, such that x + (x) = 0.

(5) (x + y) = x + y for all F and all x, y V.

(6) ( + ) x = x + x for all , F and all x V.

(7) () x = (x) for all , F and all x V.

(8) 1 x = x for all x X.


122

In practice we simply write V for the vector space (V, F, +, ). We also call elements of F scalars
in this case. A real vector space is a vector space over R, and a complex vector space is a
vector space over C. In the context of Mathematical Analysis, vector spaces are also frequently
referred to as linear spaces. Elements of a vector space are at times called vectors. So 0 can
be referred to as the zero element or the zero vector. We will not distinguish in notation been
the scalar 0 and the zero vector. It will however always be clear which zero the symbol 0 denotes.

The first inequality we wish to prove is called the Cauchy-Bunyakowski-Schwarz inequality. We


shall refer to it as the CBS inequality. We remark that in a number of books the CBS inequality
is also called the Cauchy-Schwarz inequality. We need to prepare some ground in order to be
able to state this inequality.

Definition 7.1.2 Let V be a vector space over F. An inner product on V is a scalar-valued


function , : V V F such that for all x, y, z V and all F:

IP1. x, x is a real number 0.

IP2. x, x = 0 x = 0.

IP3. x, x = y, x , where the bar denotes the complex conjugate.

IP4. x + y, z = x, z + y, z .

IP5. x, y = x, y .

An inner product space is a pair (V, , ) where V is a vector space and , an inner product
on V.

Notice that even if V is a vector space over C, we require that x, x be a real number which is
nonnegative. In general, if x = y are vectors in a complex vector space V , then x, y may be
a complex number with imaginary part unequal to zero. Also notice that if V is a real vector
space, then IP3 says x, y = y, x .

Observations: Let V be an inner product space. For x, y, z V and F we have:

(a) x, y + z = y + z, x = y, x + z, x = x, y + x, z .

(b) x, y = y, x = x, y = y, x = x, y
123 MAT3711/1

Theorem 7.1.3 (Cauchy-Bunyakowski-Schwarz inequality) Let V be an inner product space.


Then for all x, y V
| x, y | x, x y, y

Proof : If x = 0 or y = 0, then the result holds trivially. So assume x = 0 and y = 0. Now for
any F we have

0 x y, x y

= x, x y, x x, y + y, y
= x, x x, y [ y, x y, y ].

x, y
Choosing = yields
y, y

y, x x, y
0 x, x
y, y
x, y x, y
= x, x
y, y
| x, y |2
= x, x ,
y, y

which implies that


| x, y |2 x, x y, y ,

whence the result follows. 

Our next inequality deals with sequences of scalars. Recall that by a scalar we mean a real or
complex number. In order to prove it we shall need the following result.

1 1
Lemma 7.1.4 Let a, b, p and q be real numbers such that a 0, b 0, p > 1 and + = 1
p q
p
(so that q = , and hence q > 1). Then
p1

a b
ab + .
p q

Proof : If a = 0 or b = 0, then the result holds trivially since then the left-hand side equals 0.
So assume a > 0 and b > 0. Define a function f : (0, ) R by

tp tq
f (t) = + .
p q
124

We show that f has a minimum at t = 1. Dierentiating f we obtain

p1
f tq1

= 0 i t = 1.

Since f (t) as t 0+ and f (t) as t , it follows that f has a minimum at t = 1.


Thus, for all t > 0, we have
1 = f (1) f (t).

If we choose t = a1/q b1/p we obtain

ap/q b1 a1 bq/p
1 +
p p

which implies
a(p/q)+1 b(q/p)+1
ab + .
p q
But (p/q) + 1 = p and (q/p) + 1 = q, so the result follows. 

Definition 7.1.5 Let p be a real number with p 1. The sequence space p (R) is the set of

all sequences {xn } of real numbers such that |xk |p < . The sequence space p (C) is the set
k=1

of all sequences {zn } of complex numbers such that |zk |p < . By p we shall mean p (R)
k=1
or p (C) without specifying which.

Theorem 7.1.6 (Hlders inequality for sequences) Let p and q be real numbers with p > 1, q > 1
1 1
and + = 1. Let {xn } p and {yn } q . Then
p q
1 1
p q

|xk yk | |xk |p |yk |q


k=1 k=1 k=1


Proof : If |xk |p = 0 or |yk |q = 0 the result holds trivially. So assume that
k=1 k=1


|xk |p > 0 and |yk |q > 0.
k=1 k=1

Let i N, and in the lemma above put

|xi | |yi |
a= 1 and b = 1
p q
|xk |p |yk |q
k=1 k=1
125 MAT3711/1

to obtain
|xi | |yi | 1 |xi |p 1 |yi |q
1 1 + .
p q p q
|xk |p |yk |q |xi |p |yi |q
k=1 k=1
k=1 k=1
Now summing over all i N in this latter inequality we get

|xi yi |
i=1 1 1
1 1 + = 1,
p q p q
|xk |p |yk |q
k=1 k=1
which implies that
1 1
p q

|xi yi | = |xk yk | |xp |p |yk |q . 


i=1 k=1 k=1 k=1

Next we have the following inequality the proof of which uses Hlders inequality.

Theorem 7.1.7 (Minkowskis inequality for sequences) Let p be a real number with p > 1. Let
{xn } and {yn } be sequences in p. Then
1 1
p p
1
|xk + yk |p |xk |p + |yk |p .
p
k=1 k=1 k=1

p 1 1
Proof : Let q = . Then we have that + = 1, so that Hlders inequality may be used.
p1 p q
Let us note that since p > 1, for each k N,

|xk + yk |p (|xk | + |yk |)p (2 max{|xk |, |yk |)p = 2p max{|xk |p , |yk |p } 2p |xk |p + 2p |yk |p .

Hence |xk +yk |p < if {xn } and {yn } are in p. This shows that the sequence {|xn +yn |p1 }
k=1
is in q since

|xk + yk |(p1)q = |xk + yk |p < .
k=1 k=1

If |xk + yk |p = 0 the result is immediately true. So assume |xk + yk |p > 0. Then
k=1 k=1

p
|xk + yk | = |xk + yk |p1 |xk + yk |
k=1 k=1

p1
|xk + yk | |xk | + |xk + yk |p1 |yk | (triangle inequality)
k=1 k=1
1 1 1
q p p

|xk + yk |(p1)q |xk |p + |yk |p


k=1 k=1 k=1

by the Hlder inequality.


126

Thus
1 1 1
q p p

|xk + yk |p |xk + yk |p |xp |p + |yk |p .


k=1 k=1 k=1 k=1

Dividing both sides by


1
q

|xk + yk |p
k=1

we get
1 1q
1
p
1
p

|xk + yk |p |xk |p + |yk |p ,


k=1 k=1 k=1

1 1
whence the result follows since 1 = . 
q p

The Hlder and Minkowski inequalities as we have stated apply to infinite sequences. They have
the following special finitary versions that we highlight.

Let n be a natural number, x = (x1 , x2 , . . . , xn ) and y = (y1 , . . . , yn ) be elements of Fn (i.e.


Rn or Cn ). So x and y are n-tuples of real or complex number. Let x and y be the sequences
x = x1 , x2 , . . . , xn , 0, 0, . . . and y = y1 , y2 , . . . , yn , 0, 0, . . .. Then x 2 and y = 2. Since 2 > 1
1 1
and + = 1, Hlders inequality applied x and y gives
2 2

n n n
|xk yk | |x|2k |yk |2 .
k=1 k=1 k=1

On the other hand, the Minkowskis inequality applied to x and y yields

n n n
|xk + yk |2 |xk + |2 |yk |2 .
k=1 k=1 k=1

At this juncture we give several examples of vector spaces and spaces which are frequently en-
countered in Real Analysis.

Example 7.1.8 Let n be a positive integer, and put

V = {x | x = (x1 , x2 , . . . , xn ); xi F}.

Define + and multiplication by scalars pointwise, i.e.

(x1 , . . . , xn ) + (y1 , . . . , yn ) = (x1 + y1 , . . . , xn + yn ) and (x1 , . . . , xn ) = (x1 , . . . , xn ).


127 MAT3711/1

Then V is a vector space provided that if F = R then the scalars should be real numbers, whereas
if F = C, the field of scalars could be R or C. This vector space we denote by Rn or Cn (or
simply Fn ). Now Fn is also an inner product space where the inner product is defined by
n
x, y = xi yi
k=1

where x = (x1 , . . . , xn ) and y = (y1 , . . . , yn ).

Example 7.1.9 For a real number p 1, the sequence space p (F) is a vector space over F
where addition and scalar multiplication are defined pointwise, i.e. for x = {xn } and y = {yn }
in p (F), and F,

x + y = {xn + yn } = x1 + y1 , x2 + y2 , . . . , xn + yn , . . .

x = {xn } = x1 , x2 , . . .

Of course we must ascertain that x + y p (F) and x p (F) for x, y p (F) and F.
1 1
p p
If x, y p (F), then, by definition, |xp | < and |yk |p . So by Minkowskis
k=1 k=1
inequality
1 1 1
p p p
p p p
|xk + yk | |xk | + |yk | < .
k=1 k=1 k=1

Therefore x + y p (F). That x p (F) for all F and x p (F) is easy to check.

Example 7.1.10 The sequence space (F) is the set of all bounded sequences. Recall that a
sequence {xn } is bounded if there is a real number M > 0 such that |xn | M for all n. As in
the case of p we frequently write if we do not wish to specify that the terms are complex or
real numbers. Now (F) is a vector space over F where, as before, addition and multiplication
are defined pointwise. Verify this.

Example 7.1.11 The sequence space C(F) consists of all convergent sequences of reals or com-
plex numbers. It is a vector space over F where addition and scalar multiplication are defined
pointwise.

Example 7.1.12 Let X be a nonempty set. Then B(X, F), the set of all bounded functions
mapping X to F, is a vector space over F where addition and scalar multiplication are defined
as follows:
128

(f + g)(x) = f (x) + g(x), f, g B(X, F)

(f )(x) = f (x), F, f B(X, F).

Example 7.1.13 Let X be a metric space. Then C(X, F), the set of all continuous functions
mapping X to F, is a vector space over F where addition and scalar multiplication are defined
as in the case of B(X, F).

Example 7.1.14 The sequence space 0 (F) consists of all sequences {xn } in F such that xi = 0
for all but finitely many indices i. These are sequences with tails consisting of terms each equal
to 0. Defining addition and scalar multiplication pointwise makes 0 (F) a vector space over F.
We now show how an inner product can be defined on 0 (F). For any x, y 0 (F), let

x, y = xk yk .
k=1

Notice that the sum is in fact finite, so it is well-defined. Check that it defines an inner product
on 0 (F).

Example 7.1.15 For x = {xn } and y = {yn } in 2, define



x, y = xk yk .
k=1

Then , so defined is well-defined, i.e. the series converges; and in fact , is an inner
product on 2.

We close with the following quotation:

All analysts spend half their time hunting through the literature for inequalities
which they want to use but cannot prove". Harald Bohr

Exercise 7.1.16

(a) In Example 7.1.9 an astute reader will have noticed that we showed p to be a vector space
only for p > 1 because we used the Minkowski inequality. Now show that 1 is a vector
space.
129 MAT3711/1

(b) Let V be the set of all n n matrices with entries in C. For A = (Aij ) in V , let tr(A) =
n
Aii be the trace of A. For A, B V define , by
i=1

A, B = tr(B A)

where B is the conjugate transpose of the matrix B. [Consult a book on Linear Algebra
for definitions.] Show that , is an inner product of V .

(c) Let [a, b] be a compact interval in R. A function f : [a, b] C is continuous if and only if its
real and imaginary parts are continuous real-valued functions, i.e. if for f (t) = u(t) + iv(t)
where u : [a, b] R and v : [a, b] R, u and v are continuous. For such a function the
b
integral a f is defined by
b b b
f= u+i v.
a a a
Now let V be the set of all continuous complex-valued functions on [a, b]. Define , on
V by
b
f, g = f (t)g(t)dt.
a
Show that (V, , ) is an inner product space.

(d) Does , defined on R R by x, y = xy define an inner product on R? What about


z, w = zw on C?

(e) Let , be an inner product. Show that if x, y = x, z for all x; then y = z.

7.2 Normed vector spaces


In this section we introduce the core business" of Linear Analysis, namely, normed vector spaces.

Definition 7.2.1 Let V be a vector space over F. A norm on V is a real-valued function


|| || : V R which satisfies the following properties for all x, y V and F:

N1. x 0
N2. x =0x=0
N3. x = || x
N4. x+y x + y .
130

A normed vector space is a pair (V, ) where V is a vector space and is a norm on V.
If there is only one norm under consideration on a vector space V , we shall say V is a normed
vector space; and thus allow notational confusion between the normed vector space (V, || ||) and
the underlying vector space V . When we were discussing metric spaces, if we had two metric
spaces under consideration we made a point of denoting the metrics by dierent symbols. In
the case of normed vector spaces, if V and W are normed vector spaces we shall use the same
symbol || || to denote the (possibly dierent) norms on V and W .
The property N4 in the definition of a norm is called the triangle inequality for reasons that
will be apparent when we consider examples of normed vector spaces. Before proceeding to give
examples of normed vector spaces we show how a norm on a vector space induces on a metric.

Theorem 7.2.2 Let V be a normed vector space. Define a function d : V V R by

d(x, y) = ||x y||

for all x, y V . Then d is a metric on V .

Proof : We check that d satisfies properties M1 through M4 in Definition 1.1.1. So let x, y, z V .


Then

(i) d(x, y) = ||x y|| 0.

(ii) d(x, y) = 0 ||x y|| = 0 x y = 0 x = y.

(iii) d(x, y) = ||x y|| = ||(1)(y x)|| = | 1|||y x|| = ||y x|| = d(y, x).

(iv) d(x, y) = ||x y|| = ||(x z) + (z y)|| ||x z|| + ||z y|| = d(x, z) + d(z, y). 

The metric d such as in the theorem above is said to be induced (or generated) by the norm
|| ||. Thus every normed vector space is a metric space where the metric in question is the one
induced by the norm. When we have a normed vector space V and we talk about metric-related
properties such as open set, continuity, convergence, completeness, etc., these will be with re-
spect to the metric induced by the norm. Take for instance the following definition.

Definition 7.2.3 A Banach space is a complete normed vector space.


131 MAT3711/1

Just to concretise the ideas a bit, let us go through the tedium of explaining what a Banach space
is; pretending we were explaining to someone who knows normed vector spaces but not metric
spaces. We would first tell them that a sequence {xn } in a normed vector space V converges if
there is a vector x V such that for every > 0 there is a positive integer n0 with the property
that ||xn x|| < whenever n n0 . Next we would tell them that a sequence {xn } in V is a
Cauchy sequence provided for every > 0 there exists n0 N such that ||xn xm || < for all
m, n n0 . We would then tell them that a Banach space is a normed vector space in which
every Cauchy sequence converges.

In Theorem 7.2.2 we showed how a norm induces a metric. Now we show how an inner product
induces a norm.

Theorem 7.2.4 Let , be an inner product on a vector space V . Define || || : V R by

||x|| = x, x .

Then || || is a norm on V .

Proof : We check that || || satisfies N1 through N4. So let x, y V and F. Then

(i) ||x|| = x, x 0 clearly.

(ii) ||x|| = 0 x, x = 0 x, x = 0 x = 0.

(iii) ||x|| = x, x = x, x = ||2 x, x = || x, x = ||||x||.

(iv) The triangle inequality is slightly involved. Notice that if z C (or R) then z +z = 2Re(z),
where Re(z) denotes the real part of z. Now

||x + y||2 = x + y, x + y

= x, x + x, y + y, x + y, y
= x, x + x, y + x, y + y, y

= ||x||2 + 2Re x, y + ||y||2


||x||2 + 2| x, y | + ||y||2

||x||2 + 2 x, x y, y + ||y||2 (CBS inequality)

= ||x||2 + 2||x||||y|| + ||y||2

= (||x|| + ||y||)2 .
132

Taking square roots yields ||x + y|| ||x|| + ||y||. 

As previously, the norm arising from an inner product as per Theorem 7.2.4 is said to be induced
(or generated) by the inner product. Now we give a host of examples.

Example 7.2.5 Let V be R or C. For each x V define || || by ||x|| = |x|. Then || || is easily
checked to be a norm on V . Notice that the induced metric (in either case) is the usual metric.
Since R and C are complete in the usual metric, we have that V is a Banach space.

Example 7.2.6 Let n N and V be the vector space Fn on n-tuples of real or complex numbers.
Let p be a real number with p 1. Define || ||p by
1
n p

||x||p = |xk |p
k=1

where x = (x1 , . . . , xn ). Then || ||p is a norm on V . Let us verify.

(i) For each k = 1, 2, . . . , n,


n
|xk | 0, so |xk |p 0, and therefore ||x||p 0.
k=1

(ii) For any x V ,


1
n p

||x||p = 0 |xk |p =0
k=1
n
|xk |p = 0
k=1
|xk |p = 0 for all k = 1, 2, . . . , n

xk = 0 for all k = 1, 2, . . . , n
x = 0.

(iii) For any x V and scalar ,


1
n p

||x||p = |xk |p
k=1
1
n p

= ||p |xk )p
k=1
= ||||x||p .
133 MAT3711/1

(iv) For any x, y V ,


1
n p

||x + y||p = |xk + yk |p


k=1
1 1
n p n p

|xk |p + |xk |p [Apply Minkowskis inequality]


k=1 k=1
= ||x||p + ||y||p

n
Notice that the special case p = 2 yields ||x||2 = x2k on Rn , which induces the usual metric.
k=1
So again in this case we have a Banach space.

Example 7.2.7 Let E be a nonempty set and define || || on the vector space V = B(E, F) by

||f || = sup{|f (x)| : x E}.

Then || || is a norm on V called the supremum norm or simply the sup norm. We verify
only the triangle inequality. Let f, g V . For any t E we have

|(f + g)(t)| = |f (t) + g(t)|


|f (t) + g(t)|

||f || + ||g||

Since t is an arbitrary element of E, we conclude that

sup{|(f + g)(t)| : t E} ||f || + ||g|| ,

that is,
||f + g|| ||f || + ||g|| .

Whenever we consider the set B(E, F) of bounded real-valued or complex-valued functions as a


normed vector space, the norm will always be assumed to be the sup norm.

Theorem 7.2.8 B(E, F) is a Banach space.

Proof : This is just a special case of Theorem 6.1.4. 


134

Exercise 7.2.9

(a) Define || ||p on p (p real and p 1) by


1
n p

||x||p = |xk |p .
k=1

Show that || || is a norm on p.

(b) Let X be a compact metric space. Deduce from Theorem 7.2.8 that C(X, F) is a Banach
space.

(c) Let V be an inner product space and || || be the norm induced by the inner product. Why
is it true that
| x, y | ||x||||y||

for all x, y V ?

(d) Let V be an inner product and || || be the norm induced by the inner product. Show that
||x|| = sup{| x, y | : y V and ||y|| = 1} for all x V .

(e) Show that the triangle inequality in a normed real vector space is an equality if and only
if y = 0 or x = y for some 0.

(f) Let V be a vector space and d be a metric on V such that

(i) d(x, y) = d(x + z, y + z) for all x, y, z V , and

(ii) d(x, y) = ||d(x, y) for all x, y V and F.

Define || || by
||x|| = d(x, 0).

Show that || || is a norm on V . Next, show that the metric induced by || || is precisely d.

(g) Show that the discrete metric on a vector space cannot be induced by a norm.

(h) Let V be a normed vector space. For any real number r > 0 and x0 V , show that:

(i) the closure of the open ball {x V : ||x x0 || < r} is the closed ball
{x V : ||x x0 || r},

(ii) the boundary of the closed ball {x V : ||x x0 || r} is {x V : ||x x0 || = r}.


135 MAT3711/1

7.3 Bounded linear operators


Quite often in Mathematics one encounters objects of the form (X, ) where X is a set and
some relation on X. The mappings f : (X, ) (Y, ) between such objects that are then of
interest are functions f : X Y which connect the relation to the relation in some way.
For instance in groups one is interested in homomorphisms which are functions that pre-
serve the group operation. In metric spaces we were mostly interested in continuous functions
f : (X, d) (Y, ). Such function map points X which are d-close to each other to points of Y
which are -close to each other. In Linear Algebra you learnt about linear maps between vector
spaces. These are maps that preserve vector addition and scalar multiplication.

So what kind of functions between normed vector spaces should be of main interest? Well, as
a normed vector space is a vector space to start with, we should like our maps to preserve the
vector space structure. Next, as we have a norm on the vector space at hand, our functions
should relate the norm of their domain to that of their co-domain in some way.

Definition 7.3.1 Let V and W be normed vector spaces. A linear operator from V to W is
a function T : V W such that T (x + y) = T (x) + T (y) for all x, y V and all scalars
and . The set of all linear operators from V to W is denoted by L(V, W ).

If T L(V, W ) and x V , we shall frequently (if no confusion can arise) write T x in place of
T (x). Now let V and W be normed vector spaces over F. Then L(V, W ) is a vector space over
F with addition and scalar multiplication defined as follows: For S, T L(V, W ) and F,

(S + T )(x) = Sx + T x

(T )(x) = (T x).

The linear operators of main interest are, as stated earlier, the ones that connect in some way
the norm of their domain to that of their co-domain. They are defined as follows.

Definition 7.3.2 Let V and W be normed vector spaces. A linear operator T : V W is said
to be a bounded linear operator if there is a real number M > 0 such that, for all x V ,

||T x|| M ||x||.

The set of all bounded linear operators from V to W is denoted by B(V, W ).


136

Recall (from Linear Algebra) that a subset S of a vector space V is called a subspace if it
is a vector space in its own right with respect to addition and scalar multiplication as defined
for V . One then has that S is a subspace of V if and only if for all x, y S and F, x+y S.

Let us observe that B(V, W ) is a subspace of L(V, W ). Indeed if T, S B(V, W ) and is a


scalar, then there are real numbers M1 > 0 and M2 > 0 such that for all x V, ||Sx|| M1 ||x||
and ||T x|| M2 ||x||. Then

||(S + T )(x)|| = ||Sx + T x||

||Sx|| + ||T x||

= ||||Sx|| + ||T x||


||M1 ||x|| + M2 ||x||
= (||M1 + M2 )||x||,

showing that B(V, W ) is a subspace of L(V, W ).

Now let us look at the adjective bounded" in the context of bounded linear operator". When
we were dealing with functions f : E (X, d) mapping a set E into a metric space (X, d), we
said such a function is bounded provided f [E], the image of E under f , is a bounded subset of
(X, d). Recall that a subset S of (X, d) is bounded in case there is a real number M > 0 such
that d(x, y) M for all x, y S.
So if we say T : V W is a bounded linear operator, do we mean that the image of V under
T is a bounded subset of the metric space W ? In order to answer this (in the negative) let us
note that if X is a normed vector space and S is a subset of X, then S is a bounded subset of
the metric space X if and only if there is a real number K > 0 such that ||x|| K for all x S.
To see this let S be bounded and choose r R such that d(x, y) = ||x y|| r for all x, y S.
Fix any point x0 S. Then for every x S we have

||x|| = ||x x0 + x0 ||
||x x0 || + ||x0 ||
r + ||x0 ||.
137 MAT3711/1

So choosing K = r + ||x0 || yields the result. Conversely, if ||x|| K for all x S, then for any
pair x, y of points in S we have

d(x, y) = ||x y||


||x|| + ||y||

2K;

which shows S to be bounded.

Theorem 7.3.3 Let V and W be normed vector spaces and let T L(V, W ). If the image of
V under T is a bounded subset of the metric space (W, d) where d is the metric induced by the
norm, then T (x) = 0 for all x V .

Proof : Suppose not, and pick x0 V such that T (x0 ) = 0. Since {T (x) | x V } is a bounded
set, there exists M > 0 such that ||T (x)|| M for all x V . Now let = (M + 1)/||T (x0 )||.
Then x0 is a vector in V such that

||T (x0 )|| = ||T (x0 )||


= |||||T (x0 )||
M +1
= ||T (x0 )||
||T (x0 )||
= M +1

> M.

This contradiction proves the theorem. 

So when dealing with linear operators, by a bounded linear operator we mean a linear operator
with the property stated in Definition 7.3.2. Similarly, B(V, W ) denotes the set of all bounded
linear operators from V to W and should not be confused with the set of functions f : V W
for which f [V ] is a bounded subset of W and, of course, denoted by B(V, W ).
As some kind of follow-up to Theorem 7.3.3 we show that if T : V W is a bounded linear
operator, then for every subset E of V which is bounded, T [E] = {T x | x E} is a bounded
subset of W .

Theorem 7.3.4 Let V, W be normed vector spaces, T : V W be a bounded linear operator


and E V be bounded. Then T [E] = {T x | x E} is a bounded subset of W .
138

Proof : Choose M > 0 such that ||T x|| M ||x|| for all x V . This is possible since T is a
bounded linear operator. Since E is a bounded set, there exists K > 0 such that ||x|| K for
all x E. Then, if x E we have

||T x|| M ||x|| M K.

So T [E] is a bounded set. 

Particularly, if T : V W is a bounded linear operator, then {||T x|| : x V and ||x|| 1} is


a subset of R which is bounded above. Its supremum is extremely important; so important that
it is given a name.

Definition 7.3.5 Let V, W be normed vector spaces and T : V W be a bounded linear


operator. The operator norm (or simply norm) of T , denoted ||T ||, is defined to be

||T || = sup{||T x|| : x V and ||x|| 1}.

Because the set {||T x|| : x V, ||x|| 1} is bounded above, ||T || (as defined above) is a real
number. Note that for any z V we have:

||z|| < 1 ||T z|| ||T ||


||z|| = 1 ||T z|| ||T ||.

What if z V is arbitrary? First note that if z = 0 then ||z|| < 1, and so that case is covered.
1 z
Now let z V \{0}. Put w = z (we shall frequently write this as ). Then w V and
||z|| ||z||
||w|| = 1; so that ||T w|| ||T ||. This implies that

z 1
||T || T = T (z)
||z|| ||z||

1
= ||T (z)||.
||z||

Consequently, on multiplying, by ||z|| both sides, we obtain the following result.

Theorem 7.3.6 Let V, W be normed vector spaces, T : V W be a bounded linear operator


and ||T || be the operator norm of T . Then

||T x|| ||T ||||x|| for all x V.


139 MAT3711/1

Let us reflect a bit on this theorem. If T : V W is a bounded linear operator we know, by


definition, that there is a real number M such that ||T x|| M ||x|| for all x V . Now the
theorem tells us that in fact ||T || is one such number. So a natural question is how ||T || is
related to other such real numbers. We shall show that ||T || is the smallest such number.

Theorem 7.3.7 Let V, W be normed vector spaces and let T B(V, W ). Then

||T || = inf{M 0 : ||T x|| M ||x|| for all x V }.

Proof : Since ||T x|| ||T ||||x|| for all x V , if we can show that for any M with ||T x|| M ||x||
for all x V we have ||T || M , then the result will follow because then ||T || will be a lower
bound of a set to which it belongs. So let M 0 be such that ||T x|| M ||x|| for all x V . If
w V is such that ||w|| 1, then ||T (w)|| M ||w|| M . Consequently M is an upper bound
for {||T (w)|| : w V, ||w|| = 1}. Thus ||T || M . 

Now let us justify calling ||T || (as defined above) the norm of T . The justification should be
that || || : B(V, W ) R does indeed define a norm on the vector space B(V, W ).

Theorem 7.3.8 Let V and W be normed vector spaces. The function || || : B(V, W ) R
given by ||T || = sup{||T x|| : ||x|| 1} is a norm on B(V, W ).

Proof : It is clear that ||T || 0 and ||T || = 0 if and only if T is the zero map. So let us verify
axioms N3 and N4. Let S, T B(V, W ) and be a scalar. then

||T || = sup{||T (x)|| : ||x|| 1} = sup{||||T x|| : ||x|| 1}

= || sup{||T x|| : ||x|| 1} = ||||T ||.

So N3 holds. To show N4 let x be an arbitrary vector in V with ||x|| 1. Then

||(T + S)(x)|| = ||T x + Sx|| ||T x|| + ||Sx|| ||T || + ||S||.

Thus the real number ||T || + ||S|| is an upper bound for the set {||(T + S)(x)|| : ||x|| 1}. So
the supremum of this set is less than or equal to ||T || + ||S||, i.e. ||T + S|| ||T || + ||S||. 

Now let U, V and W be normed vector spaces, T B(U, V ) and S B(V, W ). Then the
composite S T : U W is a bounded linear transformation as can easily be verified. We show
that the norm of a composite is at most the product of the norms of the factors.
140

Theorem 7.3.9 Let U, V and W be normed vector spaces, and let T B(U, V ), S B(V, W ).
Then ||S T || ||S||||T ||.

Proof : Let x U be such that ||x|| 1. Then

||(S T )(x)|| = ||S(T x)||

||S||||T x|| (by Theorem 7.3.6)

||S||||T ||||x|| (Theorem 7.3.6 again)

||S||||T || (since ||x|| 1).

So ||S T || = sup{||(S T )(x)|| : ||x|| 1} ||S||||T ||. 

In the introduction to this section we mentioned that maps of interest in normed vector spaces
should connect the algebraic structures of the vector spaces and also the metric properties of
mapping points that are close together to images that are close together. This latter property
is just an informal way of describing continuity. So we ask: are bounded linear operators con-
tinuous? The answer is armative.

Theorem 7.3.10 Let V and W be normed vector spaces and let T : V W be a linear operator.
Then the following are equivalent:
(a) T is a bounded linear operator.
(b) T is continuous.
(c) T is continuous at 0.

Proof : (a) (b): We show continuity by showing continuity at each point. So let x0 be a
vector in V . Let > 0 be given. Since T is a bounded linear operator there exists a real number

M > 0 such that ||T x|| M ||x|| for all x V . Put = . Now if x V and ||x x0 || < ,
M
then
||T x T x0 || = ||T (x x0 )|| M ||x x0 || < M = .

So T is continuous at x0 , and hence on V .


(b) (c): Continuity means continuity at every point.
(c) (a): Given = 1, choose, by continuity at 0, > 0 such that if ||x 0|| < , then
||T (x) T (0)|| < 1. This means that

||x|| < ||T x|| < 1.


141 MAT3711/1

2
Now put M = . We shall show that ||T z|| M ||z|| for all z V . If z = 0, then clearly


||T z|| M ||z||. If z = 0, put x = z. Then x is a vector in V with ||x|| = < .
2||z|| 2
Thus ||T x|| < 1; that is

1 > T z
2||z||

= ||T z||.
2||z||

Consequently
2
||T z|| < ||z||

and we are done. 

There is nothing special about 0 in the preceding result. You will show in the exercises that if
a linear operator is continuous at one vector (any one vector) then it is a bounded linear operator.

In calculating the norm of a bounded linear operator T : V W using the definition, we need to
compute the supremum of the set {||T x|| : ||x|| 1}. In our next result we show that in fact the
supremum of this set equal the suprema of the sets {||T x|| : ||x|| < 1} and {||T x|| : ||x|| = 1}.
First let us recall some facts we know from metric spaces and cast them in the language of norms.

FACT 1: Let V be normed vector space, {xn } be a sequence in V and x V . Then


xn x lim ||xn x|| = 0.
n

FACT 2: If T B(V, W ) and {xn } is a sequence in V which converges to x V , then


T (xn ) T (x) in W since T is continuous.

FACT 3: If V is a normed vector space, then | ||x|| ||y|| | ||x y|| for all x, y V . Hence
if xn x in V , then lim ||xn || = ||x||.
n

Theorem 7.3.11 Let V and W be normed vector spaces and let T B(V, W ). Then

||T || = sup{||T x|| : ||x|| < 1} = sup{||T x|| : ||x|| = 1}.

Proof : Put = sup{||T x|| : ||x|| < 1} and = {sup{||T x|| : ||x|| = 1}. Note that the sets
{||T x|| : ||x|| < 1} and {||T x|| : ||x|| = 1} are bounded, so and are real numbers. Also,
142

each of these sets is a subset of the set that ||T || is the supremum of. So ||T || and ||T ||.
To complete the proof we will show that ||T || = and .

||T || = : To achieve this we need only show that ||T || . Now if we can show that given
x V with ||x|| 1 we have ||T x|| , then we will be able to conclude that ||T || because
then will be an upper bound for the set of which ||T || is the supremum. So let x V with
||x|| 1. If ||x|| < 1, then ||T x|| by the definition of . So suppose ||x|| = 1. For each
1
n N let xn = 1 x. Then
n

1
||xn x|| = x x x
n

1
= ||x||
n

1
= 0 as n .
n
So xn x in V and hence T (xn ) T (x) in W . In consequence,

lim ||T (xn )|| = ||T x||.


n

But
1 1 1
||xn || = 1 x = 1 =1 < 1.
n n n
Therefore ||T (xn )|| . So {||T (xn )||} is a sequence of real numbers each less than or equal
to and converging to . So their limit is also less than or equal to ; that is, ||T x|| , as
required.
: To show that is suces to show that if x V with ||x|| < 1, then ||T x|| .
x x
For x = 0, this is trivial. So let x = 0 and ||x|| < 1. Now has norm 1, so T ||x|| .
||x||
Then
1
||T x||
||x||
||T x|| ||x||

||T x|| since ||x|| < 1.

So all in all we have ||T || ||T ||, which proves the theorem. 

We end by showing that B(V, W ) is a Banach space whenever W is a Banach space.


143 MAT3711/1

Theorem 7.3.12 Let V and W be normed vector spaces. If W is a Banach space, then B(V, W )
is a Banach space.

Proof : Let {Tn } be a Cauchy sequence in B(V, W ). So given > 0 there exists n0 N such
that
||Tn Tm || < for all m, n n0 .

Thus, if x V \{0} and > 0 is arbitrary; choosing k N so that ||Tn Tm || < whenever
||x||
m, n k we obtain

||Tn (x) Tm (x)|| = ||(Tn Tm )(x)||

||Tn Tm ||||x||

< ||x||
||x||
= .

This shows that {Tn (x)} is a Cauchy sequence in W for each x V \{0}. Clearly {Tn (0)} is a
Cauchy sequence in W because it is the constant sequence with each term equal to 0. So {Tn (x)}
is a Cauchy sequence in W for each x V . Since W is a Banach space, {Tn (x)} converges in
W , for each x V . Define T : V W by

T (x) = lim Tn (x).


n

We check that T B(V, W ) and Tn T in B(V, W ).


We first show that T is a linear operator. Let x, y V and be a scalar. Then

T (x + y) = lim Tn (x + y)
n
= lim[Tn (x) + T (y)]
n
= lim Tn (x) + T (y)
n
= T (x) + T (y).

Next we show that T is a bounded linear operator. From | ||Tn || ||Tm || | ||Tn Tm || we
deduce that {||Tn ||} is a Cauchy sequence of real numbers. So there is a nonnegative real number
M such that
lim ||Tn || = M.
n

Now let x V . Since Tn (x) T (x), ||Tn (x)|| ||T (x)||. That is

||T x|| = lim ||Tn (x)||


n
lim ||Tn || ||x|| (since each Tn B(V, W ))
n
= M ||x||.
144

So T B(V, W ). We are left with showing that Tn T in B(V, W ). So let > 0, and n0 N
be such that

||Tm Tn || <
2
for all m, n n0 . If v V with ||x|| = 1 and m, n n0 , then

||(Tm Tn )(x)|| ||Tm Tn || ||x|| ||Tm Tn || .
2
Now hold m fixed and let n approach to obtain

lim ||Tm (x) Tn (x)|| = ||Tm (x) T (x)||.


n

We therefore have

||Tm (x) T (x)|| <
2
for all m n0 and x V with ||x|| = 1. Consequently

||Tm T || <
2
for all m n0 . This completes the proof. 

We now give some examples.

1
2
Example 7.3.13 Consider the normed vector space 2 with the norm ||x||2 = |xi |2 .
i=1
Define T : 2 2 by
T x = T (x1 , x2 , . . . , ) = (x2 , x3 , . . .).

That is, if x 2 is the sequence x1 , x2 , . . . then its image under T is the sequence x2 , x3 , . . .. It
is easy to check that T is a linear operator. Let us show that T B( 2 , 2) and compute ||T ||.
Let x = (x1 , x2 , . . .) 2.

Then

||T x||2 = ||(x2 , x3 , . . .)||2



= |xi |2
i=2


|xi |2
i=1
= ||x||2

Thus, putting M = 1, we have ||T x||2 M ||x||2 for all x 2. This shows that T is a bounded
linear operator with ||T || 1. Now before evaluating ||T ||, let us consider this general technique:
145 MAT3711/1

Suppose T : V W is a bounded linear operator and suppose M is


a constant such that ||T (z)|| M ||z|| for all z V (or for all z with
||z|| 1); so that ||T || M . Now if there is a vector z0 in V with
||z0 || 1 and ||T (z0 )|| = M , then we must have ||T || = M ; for otherwise
a contradiction arises as follows: M = ||T (z0 )|| ||T || ||z0 || ||T || < M.

Going back to the problem at hand, we have ||T || 1. Let z = (0, 1, 0, 0, . . .). Then z 2,

||z|| = 1 and ||T z|| = 1. Hence ||T || = 1.

Example 7.3.14 Let T : R2 R be the map T (x, y) = x + 2y for all (x, y) R2 . Let the
norms under consideration be ||(x, y)|| = x2 + y 2 and ||w|| = |w| for (x, y) R2 and w R. It
is easy to check that T is a linear operator. Let us show that T B(R2 , R) and compute ||T ||.
First let us note that

T (x, y) = (x, y), (1, 2)

where , is the inner product as per Example 7.1.8. Thus, by the CBS inequality

||T (x, y)|| = | (x, y), (1, 2) |


||(x, y)|| ||(1, 2)||

= 5||(x, y)||.

1 2
Therefore ||T || 5. Now the vector , R2 is such that
5 5

1 2 1 2 5
, = 1 and T , = = 5.
5 5 5 5 5

We therefore conclude that ||T || = 5.

Example 7.3.15 Let h C[a, b], where C[a, b] denotes the normed vector space of all real-
valued continuous functions endowed with the sup norm. Suppose h(x) 0 for all a x b.
Define T : C[a, b] R by
b
T (g) = g(t)f (t)dt.
a
146

Then T is easily checked to be a linear operator. We show that T is a bounded linear operator
and compute ||T ||. Now
b
|T (g)| = g(t)h(t)dt
a
b
|g(t)| |h(t)| dt
a
b
||g|| h(t)dt (since h 0 and |g(t)| ||g|| t)
a
= M ||g||,

b
where M = a h(t)dt. Therefore T is a bounded linear operator and
b
||T || h(t)dt.
a

Now let f : [a, b] R be given by f (t) = 1 for all t [a, b]. Then f C[a, b] and ||f || =
sup{|f (t)| : a t b} = 1. Furthermore
b
T (f ) = h(t)dt.
a

We therefore have that


b
||T || = h(t)dt.
a
Exercise 7.3.16

(a) Let T : R2 R be given by T (x, y) = 2x + 3y for all (x, y) R2 where R2 has the norm

||(u, v)|| = u2 + v2 . Show that T B(R2 , R) and compute ||T ||.

(b) Let C[a, b] be the normed vector space of all continuous function f : [a, b] R with the
sup norm. Let g C[a, b] be such that g(t) 0 for all t [a, b]. Define T : C[a, b] R by
b
T (f ) = f (t)g(t)dt.
a

Show that T is a bounded linear operator and find ||T ||.

(c) Let V, W be normed vector spaces and T L(V, W ). Show that if there is a vector x0 V
such that T is continuous at x0 , then T B(V, W ).

(d) In the proof of Theorem 7.3.12, when showing that Tn T in B(V, W ), we could have let
U = {x V : ||x|| 1} and then noted that T, Tn B(V, W ), and then used the fact
that {Tn } converges uniformly on U to T in order to conclude that Tn T in B(V, W ).
Show it.
147 MAT3711/1

(e) Let V, W be normed vector spaces and T B(V, W ). Show that

||T x||
||T || = sup : x=0 .
||x||

(f) Let C[a, b] be as in (b). Let x0 be a real number with a < x0 < b. Define T : [a, b] R by
T (f ) = f (x0 ) for all f C[a, b]. Show that T is a bounded linear operator with ||T || = 1.

(g) Let T : 2 2 be given by T (x1 , x2 , . . .) = (0, x1 , x2 , . . .). Show that T B( 2 , 2) [ 2 is


endowed with the || ||2 norm] and find ||T ||.
148
149 MAT3711/1

Chapter 8

The Riemann-Stieltjes Integral


From elementary Calculus you are familiar with the geometrical basis of an integral being the
area under the curve. Also in the first course on rigorous Analysis (MAT2613) you learnt about
the Riemann integral as a concept on which area under a curve was based rather than the other
way around.

In this chapter we study a concept which is more general than that of the Riemann integral,
and one of which the Riemann integral is a special case.

8.1 The integral


Throughout this section we shall be working with a closed and bounded interval [a, b], and all
functions will be real-valued and bounded on [a, b].

By a partition of [a, b] we mean a finite set of points

P = {x0 , x1 , . . . , xn }

such that a = x0 < x1 < . . . < xn1 < xn = b. A partion Q of [a, b] is said to be finer than P
(or a refinement of P ) in case Q P . The k th subinterval of a partition P = {x0 , x1 , . . . , xn }
is the interval [xk1 , xk ]. Its length xk xk1 is denoted by xk . If : [a, b] R, we denote
(xk ) (xk1 ) by k . By the norm of a partition P , denoted ||P ||, we mean the length of
the longest subinterval of P ; that is,

||P || = max{ xk | k = 1, 2, . . . , n}.

Note that if P, Q are partitions of [a, b] and Q is finer than P then ||Q|| ||P ||. Symbolically,

Q P ||Q|| ||P ||.

Thus, refinement of a partition reduces its norm.


150

Recall that a function : [a, b] R is said to be increasing on [a, b] if for all x, y [a, b],
(x) (y) whenever x y.

Definition 8.1.1 Let f : [a, b] R be bounded, : [a, b] R be increasing on [a, b] and


P = {x0 , x1 , . . . , xn } be a partition of [a, b]. For each k = 1, 2, . . . , n let

Mk (f ) = sup{f (x) | xk1 x xk }, and


mk (f ) = inf{f (x) | xk1 x xk }.

The numbers
n n
U (P, f, ) = Mk (f ) k and L(P, f, ) = mk (f ) k
k=1 k=1

are called respectively, the upper Stieltjes sum and the lower Stieltjes sum of f with respect
to for the partition P .

Note that mk (f ) Mk (f ) for each k, and k 0 since is increasing on [a, b]. Consequently
we have mk (f ) k Mk (f ) k for each k, whence we deduce that

L(P, f, ) U (P, f, )

for every partition P of [a, b]. Since f is bounded there are real numbers m and M such that
m f (x) M for all x [a, b]. Now note that
n
k = (x1 ) (x0 ) + (x2 ) (x1 ) + + (xn ) (xn1 )
k=1
= (b) (a).

Thus, for any partition P of [a, b], we have


n n n
m k mk k M k
k=1 k=1 k=1

m[(b) (a)] L(P, f, ) M [(b) (a)].

Similarly we have
m[(b) (a)] U (P, f, ) M [(b) (a)].

This shows that the sets

{L(P, f, ) | P is a partion of [a, b]} and {U (P, f, ) | P is a partion of [a, b]}


151 MAT3711/1

are bounded. So, in particular, the supremum of the first set, and the infimum of the second
exist and are real numbers.

Definition 8.1.2 Let f and be as above. The upper Stieltjes integral of f with respect to
is defined by
b
f d = inf{U (P, f, ) | P is a partition of [a, b]}.
a
The lower Stieltjes integral of f with respect to is defined by
b
f d = sup{L(P, f, ) | P is a partition of [a, b]}.
a

We say f is Riemann-integrable (or simply integrable) with respect to , and write f R(),
in case the lower and upper Stieltjes integrals are equal. The common value is then denoted by
b
f d
a

and called the Riemann-Stieltjes integral (or simply the Stieltjes integral) of f with respect
to on [a, b]. The functions f and are respectively called the integrand and the integrator.

If we take to be the function (x) = x, then the lower and upper Stieltjes sums, the lower
and upper Stieltjes integrals and the Riemann-Stieltjes integral are denoted by
b b b
L(P, f ), U (P, f ), f, f and f.
a a a

They are precisely the concepts discussed in the MAT2613 course. Thus a function f is Riemann-
integrable on [a, b] if and only if it is Riemann-Stieltjes-integrable on [a, b] with respect to ,
where (x) = x for all x [a, b].

b b
A word about notation: The integral a f d is sometimes denoted by a f (x)d(x). In such a
case it should be noted that the variable x contributes nothing to the numerical value of the
integral, which depends only on f, , a and b. Thus the variable x is just a dummy variable"
and can be replaced by any other convenient variable.

b
Definition 8.1.3 Let a < b. If the integral a f d exists, we define
a b
f d = f d.
b a

Also, for any a R we define


a
f d = 0.
a
152

Now let us investigate some properties of lower sums, upper sums, lower integrals and upper
integrals. Our first result shows that refinement of partition increases lower sums and decreases
upper sums. From that we obtain that for any two partitions, the lower sum with respect to
the one partition is always less than or equal to the upper sum with respect to the other.

Theorem 8.1.4 Let P be a partition of [a, b] and P be a refinement of P . Then

L(P, f, ) L(P , f, ) and U (P , f, ) U (P, f, ).

Proof : First suppose P contains just one more point than P . Say this point is c and lies in the
kth subinterval of P ; so that
xk1 < c < xk .

Now put
M = sup{f (x) | xk1 x c} and M = sup{f (x) | c x xk }

and notice that


M Mk (f ) and M Mk (f ).

Now
k1 n
U (P , f, ) = Mi (f ) i + M [(c) (xk1 )] + M [(xk ) (c)] + Mi (f ) i
i=1 i=k+1
n
Mi (f ) i + Mk (f ) [(c) (xk1 )] + Mk (f )[(xk ) (c)]
i=1
i=k
n
= Mi (f ) i + Mk (f )[(xk ) (xk1 )]
i=1
i=k
= U (P, f, ).

If P has points more than P , we simply repeat this argument times. The proof that
L(P, f, ) L(P , f, ) is similar; define m and m analogously to M and M and note that
mk (f ) m and mk (f ) m . 
153 MAT3711/1

Corollary 8.1.5 Let P and Q be any two partitions of [a, b]. Then L(P, f, ) U (Q, f, ).

Proof : Put P = P Q and note that P refines both P and Q. Therefore

L(P, f, ) L(P , f, ) U (P , f, ) U (Q, f, ). 

We have used the fact we observed earlier that for the same partition R we always have
L(R, f, ) U(R, f, ). The corollary tells us that if we start with any partition and then
keep refining it, the lower sums increase and the upper sums decrease, but all the time the lower
sums stay smaller than or equal to the upper sums. Thus, informally, integrability occurs if in
the limit of refining this increasing and decreasing culminates with equality.

b b
Theorem 8.1.6 f d f d.
a a

Proof : Let P be an arbitrary partition of [a, b]. For any partition Q of [a, b] we have

L(Q, f, ) U (P, f, ).

Therefore the real number U (P, f, ) is an upper bound for the set of all lower sums. It is
therefore greater than or equal to the supremum of this set; that is,
b b
f d f d. 
a a

We now give a characterisation of integrability which is often useful in practice.

Theorem 8.1.7 A function f is integrable with respect to on [a, b] if and only if for every
> 0 there is a partition P of [a, b] such that U (P, f, ) L(P, f, ) < .

Proof : () Let f R() on [a, b] and let > 0 be given. Since


b

> 0 and f d = sup{L(P, f, ) | P is a partition of [a, b]},
2 a

there exists a partition Q of [a, b] such that


b

f d < L(Q, f, ).
a 2
b
Likewise, since a f d = inf{U (P, f, ) | P is a partition of [a, b]}, there exists a partition R of
[a, b] such that
b

U (R, f, ) < f d + .
a 2
154

From the first inequality we have


b

f d + < L(Q, f, ) + .
a 2
Now let P = Q R and note that P refines both Q and R. Therefore
b

U (P, f, ) U (R, f, ) < f d + < L(Q, f, ) + L(P, f, ) + ;
a 2
whence the result follows.

b b
() We must show that a f d = a f d. Let > 0 and choose a partition P such that
U (P, f, ) L(P, f, ) < . Then
b b
L(P, f, ) f d f d U (P, f, ),
a a

which gives
b b
0 f d f d U (P, f, ) L(P, f, ) < .
a a

Since this is true for every > 0 we must have


b b
f d f d = 0. 
a a

Next we show that a continuous function is integrable. But before doing so, let us reiterate
what we said earlier. Our integrators will always be assumed to be increasing; by which we
mean (x) (y) for all x, y with x y. Note in particular that if (a) = (b), then is
constant on the interval [a, b]. Thus, for any partition P = {x0 , . . . , xn } of [a, b], k = 0.
Hence L(P, f, ) = U (P, f, ) = 0 for any bounded function f on [a, b]. Consequently, for an
b
integrator which is constant we have f R() and a f d = 0.

Theorem 8.1.8 If f is continuous on [a, b], then f R().

Proof : If is constant, there is nothing to prove. So suppose is not constant, so that


(b) (a) > 0. Now let > 0 be given. Put

= .
(b) (a)
Then > 0. Since f is continuous on [a, b] and [a, b] is compact, f is uniformly continuous on
[a, b]. So there exists > 0 such that

|f (x) f (y)| < whenever x, y [a, b] and |x y| < .


155 MAT3711/1

Now let P be any partition of [a, b] with ||P || < . Say P = {x0 , x1 , . . . , xn }. For any
i = 1, 2, . . . , n we have

Mi (f ) mi (f ) = sup{f (z) f (w) : z, w [xi1 , xi ]}


sup{|f (z) f (w)| : z, w [xi1 , xi ]}
.

Consequently we have
n
U (P, f, ) L(P, f, ) = [Mi (f ) mi (f )] i
i=1
n
i
i=1
= [(b) (a)]
< .

In view of Theorem 8.1.7, f R(). 

We now want to prove the linear properties of the integral. For this we observe the following fact
which facilitates the proofs. Note that if A R is bounded above, then the set B = {a | a A}
is bounded below and inf B = sup A. Similarly, if A is bounded below then B = {a | a A}
is bounded above and sup B = inf A.

Now let f be bounded on [a, b], P be a partition of [a, b] and f be the function defined by
(f )(t) = f (t) for all t [a, b]. Then Mk (f ) = mk (f ) and mk (f ) = Mk (f ) for all
k = 1, 2, . . . , n. A straightforward calculation shows that if f R() on [a, b] then f R()
b b
on [a, b] and a (f )d = a f d.

Theorem 8.1.9 Let f, g R() on [a, b] and let c R. Then

(a) f + g R () on [a, b] and


b b b
(f + g) d = f d + gd.
a a a

(b) cf R () on [a, b] and


b b
cf d = c f d.
a a
156

(c) if f (x) g (x) for all x [a, b] ,


b b
f d gd.
a a

(d) for any d with a < d < b, f R () on [a, d] and on [d, b] and
b d b
f d = f d + f d.
a a a

(e) if |f (x)| M for all x [a, b] ,


b
f d M [a (b) (a)] .
a

Proof : We show only (a) and leave the other proofs (which are similar to that of (a)) as an
exercise. For brevity denote f + g by h. For any partition P of [a, b] we have

L(P, f, ) + L(P, g, ) L(P, h, )

U (P, h, )
U (P, f, ) + U (P, g, ).

Let > 0 be given. Find partitions R, Q of [a, b], such that



U (R, f, ) L(R, f, ) < and U (Q, g, ) L(Q, g, ) < .
2 2
Let P = R Q. Then the two inequalities above hold with R and Q each replaced by P . This
then implies that
U (P, h, ) L(P, h, ) < ,

and therefore f + g R() by Theorem 8.1.7.


Next, with the partition P as above we have
b b

U (P, f, ) < f d + and U(P, g, ) < gd +
a 2 a 2
which implies that
b b b
hd U (P, h, ) < f d + gd + .
a a a
Since this holds for all > 0, we have that
b b b
hd f d + gd.
a a a
157 MAT3711/1

Since the inequality in () holds for any two function f, g R(x), it holds for f and g.
Applying it to these two functions we get

b b b
[(f ) + (g)]d (f )d + (g)d
a a a
b b b
[(f + g)]d f d gd
a a a
b b b
(f + g)d f d + gd
a a a
b b b
f d + gd (f + g)d. ()
a a a

Now () and () yield the desired result. 

Theorem 8.1.10 Let f R() and R() on [a, b] and let c be a real number with c 0. Then
Proof : Exercise. 

Question: Why did we put the restriction c 0 in the foregoing theorem?

Theorem 8.1.11 If f R() on [a, b], then |f | R() on [a, b] and

b b
f d |f |d.
a a

Proof : For any partition P of [a, b] we have

Mk (|f |) mk (|f |) Mk (f ) mk (f )

since | |f (x)| |f (y)| | |f (x) f (y)| for all x, y. So given > 0, find a partition P such that

U (P, f, ) L(P, f, ) < .

Then for this partition we have

U (P, |f |, ) L(P, |f |, ) < .


158

Thus |f | R(). Now note that |f | f |f |, so


b b b
|f |d f d |f |d
a a a

by Theorem 8.1.9(c). Since, for r 0 we have |w| r r w r, we conclude that


b b
f d |f |d. 
a a

Example 8.1.12 This example shows that |f | R() f R(). Let (x) = 2x on [0, 1].
Define f : [0, 1] R by
1 if x is rational
f (x) =
1 if x is irrational.

For any partition P of [0, 1],

mk (f ) = 1 and Mk (f ) = 1

since in each subinterval [xk1 , xk ] there are rational and irrational numbers. This shows that
n n
U (P, f, ) L(P, f, ) = Mk (f ) k mk (f ) k
k=1 k=1
n n
= k + k
k=1 k=1
= [(1) (0)] + [(1) (0)]
= 4.

So for < 4, there is no partition P such that

U(P, f, ) L(P, f, ) < .

1
Of course |f | R() and 0 |f |d = 2 (verify).

Theorem 8.1.13 If f R() on [a, b], then f 2 R() on [a, b].

Proof : For any partition P of [a, b],

Mk (f 2 ) = [Mk (|f |)]2 and mk (f 2 ) = [mk (|f |)]2 .

Therefore

Mk (f 2 ) mk (f 2 ) = [Mk (|f |) + mk (|f |)][Mk (|f |) mk (|f |)]

2M [Mk (|f |) mk (|f |)]


159 MAT3711/1

where M > 0 is such that |f (x)| M for all x [a, b]. So given > 0, if P is a partition of
[a, b] such that

U (P, |f |, ) L(P, |f |, ) <
2M
then
U (P, f 2 , ) L(P, f 2 , ) < . 

Theorem 8.1.14 If f R() and g R() on [a, b], then the product f g R() on [a, b].

Proof : Note that f g = 12 [(f + g)2 f 2 g 2 ] and apply earlier results. 

In Theorem 8.1.8 we saw that continuous functions are integrable. Recall that a function

f : [a, b] R

is said to be monotonic on [a, b] in case it is either increasing (x y f (x) f (y)) or


decreasing (x y f (x) f (y)). Our next result shows that a monotonic function is always
integrable with respect to a continuous integrator.

Theorem 8.1.15 If f is monotonic on [a, b] and is continuous on [a, b], then f R() on
[a, b].

Proof : Let > 0 be given. Choose a natural number n such that


[(b) (a)]|f (b) f (a)|
n> .

We are going to construct a partition P of [a, b] such that

U (P, f, ) L(P, f, ) < .

If is constant, there is nothing to prove. So we may assume (a) < (b). Then
1
(a) < (a) + [(b) (a)] < (b).
n
Since is continuous on [a, b] and [a, b] is connected, there is a point x1 such that
1
a < x1 < b and (x1 ) = (a) + [(b) a].
n
That is,
(b) (a)
x0 < x1 < xn and (x1 ) (x0 ) = ,
n
160

where we have set a = x0 and b = xn . Repeating this process n times, we obtain points
x0 < x1 < x2 < . . . < xn such that
(b) (a)
(xk ) (xk1 ) = for all k = 1, 2, . . . , n.
n
Now f is either increasing or decreasing. Say it is decreasing. Then

Mk (f ) = f (xk1 ) and mk (f ) = f (xk )

for all k = 1, 2, . . . , n. Thus, for the partition P = {x0 , x1 , . . . , xn }, we have


n
U (P, f, ) L(P, f, ) = k [Mk (f ) mk (f )]
k=1
n
(b) (a)
= [f (xk1 ) f (xk )]
n
k=1
(b) a
= [f (a) f (b)]
n
(b) (a)
= |f (b) f (a)|
n
= < .

If f is increasing, the proof is similar. 

b
Corollary 8.1.16 If f is monotonic on [a, b], then the Riemann integral a f (x)dx exists.

Let us take another look at Theorems 8.1.8 and 8.1.15. The first ensures integrability of a
continuous function with respect to any integrator (which of course should be increasing). The
second ensures integrability of any monotonic function (continuous or not) with respect to a
continuous integrator. The next result allows the integrand f to be discontinuous at finitely
many points but then the integrator must be continuous at those points.

Theorem 8.1.17 If f is discontinuous at finitely many points of [a, b], and is continuous at
those points; then f R().

Proof : Let the points where f is discontinuous be d1 , d2 , . . . , dm . Let > 0 be given. Let M > 0
be such that |f (x)| M for all x [a, b]. By the continuity of on the set E = {d1 , . . . , dm }, we
can find neighbourhoods (recall that in R a nbd of a point contains an open interval containing
the point) (ui , vi ), i = 1, 2, . . . , m of di such that if t (ui , vi ) then

|(t) (di )| < .
2m+1
161 MAT3711/1


This implies that (vi ) (ui ) < for each i (verify this). Furthermore, the points ui , vi can
2m
be chosen such that the intervals [ui , vi ] are disjoint. Notice that
m
[(vi ) (ui )] < .
i=1

Now let K [a, b] be the set


m
K = [a, b]\ (ui , vi )
i=1
= [a, u1 ] [v1 , u2 ] [um , b]

where we have assumed u1 < v1 < u2 < v2 < . . . < um < vm ; which we may without loss of
generality.
Thus K is compact since it is a closed and bounded subset of R. Since f is continuous on K, it
is uniformly continuous on K. So there exists > 0 such that

s, t K and |s t| < |f (s) f (t)| < .

Now form a partition P = {x0 , x1 , . . . , xn } as follows: each ui occurs in P and each vi occurs in
P . No point x with ui < x < vi occurs in P . If xk1 {u1 , u2 , . . . , um }, then xk xk1 < .
Now note that Mk (f ) mk (f ) 2M and Mk (f ) mk (f ) unless xk1 {u1 , u2 , . . . , um }.
Consequently,

U (P, f, ) L(P, f, ) [(b) (a)] + 2M


= [(b) (a) + 2M ].

Since is arbitrary, it follows that f R(). 

In the case of the Riemann integral, we can actually say more. We need the following definition
in order to state the result we are alluding to.

Definition 8.1.18 A set S R is said to be of measure zero if, for every > 0, there are
countably many open intervals (a1 , b1 ), (a2 , b2 ), . . . such that S (ak , bk ) and (bk ak ) < .
k k

In the above definition, if the collection of the covering intervals is finite, then the index k runs
over a finite set. If the collection is countably infinite, then k goes from 1 to .

Lemma 8.1.19 A countable union of sets of measure zero is a set of measure zero.
162

Proof : Let S1 , S2 , . . . be sets of measure zero and let > 0 be given. For each k = 1, 2, . . . let Sk

be covered by countably many intervals the sum of whose lengths is less than k . The collection
2
of all these coverings is a countable collection, and the sum of the lengths of all the intervals is
less than

= . 
2k
k=1


Example 8.1.20 A singleton {x} is of measure zero since the interval x , x + has length
3 3
2
< . Hence every countable subset of R is of measure zero. Note that there does exist an
3
uncountable set of measure zero. Also, a subset of a set of measure zero is itself of measure zero.

The proof of the characterisation we wish to state is rather too involved, and we will therefore
not provide it. It can however be found, for instance, in T.M. Apostols Mathematical Analysis.

Theorem 8.1.21 (Lebesgues criterion for Riemann-integrability). Let f : [a, b] R be a


bounded function and let D denote the set of points in [a, b] where f is discontinuous. Then f
is Riemann-integrable on [a, b] if and only if D is of measure zero.

We end this section with the following result which is known as the change of variable technique
for integration. Recall that a function g is said to be strictly increasing in case

x < y g(x) < g(y) for all x, y

in the domain of g.

Theorem 8.1.22 (Change of variable) Let f R() on [a, b] and be a strictly increasing
function defined on some interval [c, d]. Assume that a = (c) and b = (d). Define and g on
[c, d] by
(t) = ((t)) and g(t) = f ((t)).

Then g R() on [c, d] and


d b
gd = f d;
c a
that is
d g(d)
f (g(x))d[(g(x))] = f (t)d(t).
c g(c)

In particular,
g(d) d
f (t)dt = f (g(x))d[g(x))].
g(c) c
163 MAT3711/1

Proof : To each partition P = {x0 , . . . , xn } of [a, b] corresponds a partition Q = {y0 , . . . , yn } of


[c, d] such that xi = (yi ). In fact all partitions of [c, d] are of this nature. Since the values
taken by f on [xi1 , xi ] are exactly the same as those taken by g on [yi1 , yi ], we have that

U (Q, g, ) = U (P, f, ) and L(Q, g, ) = L(P, f, ).

Since f R() on [a, b], P can be chosen so that both U (P, f, ) and L(P, f, ) are close to
b
a f d. More precisely, what we mean is: given > 0 we can find a partition P such that
b b

U (P, f, ) + < f d and L(P, f, ) < f d.
2 a 2 a

This then yields


U (Q, g, ) L(Q, g, ) < ,

showing that g R() on [c, d]. The claimed equality of integrals then follows easily. 

Let us now consider some examples.

Example 8.1.23 Let f, and be functions defined on [0, 1] by



0, if 0 x < 1
2
f (x) =
2, if 1 x 1
2

0, if 0 x < 1
2
(x) =
1, if 1 x < 1
2

0, if 0 x 1
2
(x) =
1, if 1 < x 1
2

Let us determine if f R() and f R() on [0, 1], and evaluate the integral where it exists.
1 1
Let P = {x0 , . . . , xn } be a partition of [0, 1] such that 2 P . Say 2 = xj . Now

Mk (f ) = 0 for k = 1, . . . , j 1.
Mk (f ) = 2 for k = j, . . . , n.
mk (f ) = 0 for k = 1, . . . , j.

mk (f ) = 2 for k = j + 1, . . . , n.
164

k = 0 for k = 1, . . . , j 1.

j = (xj ) (xj1 )
= 1 0 = 1.
k = 1 1 = 0 for k = j + 1, . . . , n.

k = 0 for k = 1, . . . , j.
j+1 = 1 0 = 1.

k = 1 1 = 0 for k = j + 2, . . . , n.

Thus
U (P, f, ) = 2 and L(P, f, ) = 0.

So for < 1, for instance, there is no partition Q such that U (Q, f, ) L(Q, f, ) < . If there
1
were such a partition Q, then, as seen above, 2 would not be in Q. Hence taking P = Q { 12 }
would give a partition finer than Q which would mean

U (P, f, ) L(P, f, ) U (Q, f, ) L(Q, f, ) < 1,

contrary to what we have observed about partitions containing the point 12 .


On the other hand we have, for P as before,

U (P, f, ) = 2 j+1 , since other terms vanish


= 2, and

L(P, f, ) = 2 j+1

= 2.

Then f R() on [0, 1]. Since any lower sum is a lower bound for the set of all upper sums, we
have
1 1
2 f d = f d.
0 0
Similarly, since any upper sum is an upper bound for the set of all lower sums, we have
1 1
f d = f d 2.
0 0

Consequently,
1
f d = 2.
0
165 MAT3711/1

Example 8.1.24 Let f be a bounded Riemann-integrable function on [a, b]. Let

g : [M, M ] R

be continuous where M = sup{|f (x)| : a x b}. Then g f is Riemann-integrable on [a, b].


To prove this let D = {x [a, b] | f is not continuous at x}. Since f is Riemann-integrable
on [a, b], D is of measure zero. Since g is continuous on the range of f , we have that g f is
continuous at every point where f is continuous. Consequently the set of points where gf is dis-
continuous is a subset of D, and hence of measure zero. Thus gf is Riemann-integrable on [a, b].

Exercise 8.1.25

(a) Let f be nonnegative on [a, b]; that is, f (x) 0 for all x [a, b]. Suppose f is continuous
b
on [a, b] and a f (x)dx = 0. Show that f (x) = 0 for all x [a, b].

(b) Let be increasing on [a, b], a x0 b and be continuous at x0 . Let f : [a, b] R


given by
1, if x = x
0
f (x) =
0, if x = x0 .
b
Show that f R() on [a, b] and a f d = 0.

(c) Define : [0, 1] R by


0, if 0 x 1
2
(x) =
1, if 1 < x 1.
2
Let f be a bounded function on [0, 1]. Show that f R() if and only if f is continuous
1 1
at 2 from the right [this means: lim f (x) = f ( 12 ).] In this case, show that 0 f d = f ( 12 ).
x 12
x> 12

(d) Let be an increasing function on [a, b] and suppose R() on [a, b]. Show that
b
a d = 12 [(b)2 (a)2 ].

8.2 The Fundamental Theorem of Calculus


In this section we consider the relationship between integration and dierentiation. Throughout
this section we will deal with the Riemann integral. As before, all functions are assumed to be
bounded.

Theorem 8.2.1 Let f be Riemann-integrable on [a, b]. Define F : [a, b] R by


x
F (x) = f (t)dt.
a
166

Then F is uniformly continuous on [a, b]. Furthermore, if f is continuous at a point x0 [a, b],
then F is dierentiable at x0 with F (x0 ) = f (x0 ).

Proof : Pick M > 0 such that |f (t)| M for all t [a, b]. Now if a x < y b, then
y
|F (y) F (x)| = f (t)d(t) M |y x|.
x

Thus given > 0, if we set = , then we get
M

|x y| < |F (x) F (y)| < M = .
M
This shows that F is uniformly continuous on [a, b]. Now suppose f is continuous at x0 . Given
> 0, choose > 0 such that

t [a, b] and |t t0 | < |f (t) f (x0 )| < .

Thus, if x0 < s x0 t < x0 + and a s < t b, then


t
F (t) F (s) 1
f (x0 ) = [f (u) f (x0 )]du < .
ts ts s

This proves that


F (x0 + h) F (x0 )
lim = f (x0 ). 
h0 h

Theorem 8.2.2 (Fundamental Theorem of Calculus) If f is Riemann-integrable on [a, b] and


there is a dierentiable function F on [a, b] such that F = f , then
b
f (x)dx = F (b) F (a).
a

Proof : Let > 0. Pick a partition P of [a, b] such that

U (P, f ) L(P, f ) < .

Now for any k = 1, 2, . . . , n, let tk [xk1 , xk ]. Then

mk (f ) f (tk ) Mk (f )

which implies that


n
L(P, f ) f (tk ) xk U (P, f ) ()
k=1

where xk = xk xk1 . Since f is integrable we also have


b
L(P, f ) f (x)dx U (P, f ). ()
a
167 MAT3711/1

Combining () and () yields


n b
f (tk ) xk f (x)dx < . ( )
k=1 a

Now apply the Mean Value Theorem to F on [xk1 , xk ] to obtain points sk [xk1 , xk ] such
that

F (xk ) F (xk1 ) = F (sk )(xk xk1 )


= f (sk ) xk .

Summing over all k we get


n
F (b) F (a) = f (sk ) xk .
k=1
Since ( ) holds for all points tk in [xk1 , xk ], we have
b
F (b) F (a) f (x)dx < . ( )
a

Nothing on the left hand side of ( ) depends on P ; and this inequality holds for all > 0.
Thus
b
f (x)dx = F (b) F (a). 
a

Example 8.2.3 Let f : [0, 1] R be defined by



0, if is irrational or x = 0
f (x) =
1 , if x = p where p, q are integers in lowest form.
q q

Define F : [0, 1] R by
x
F (x) = f (t)dt.
0
Let us show that

(i) F (x) = 0 for all x [0, 1].

(ii) F exists on [0, 1], but F (x) = f (x) when x is rational.

Solution:

(i) We must show that f is integrable on [0, x] for all x [0, 1] and that the value of the
integral is zero. We show integrability by proving that f is continuous at every irrational
point. So the set of points where f is discontinuous will be a subset of Q, and hence of
measure zero.
168

pn
So let be an irrational number in [0, 1] and xn = qn be rational numbers such that pn
and qn have no common factor other than 1 and limn xn = . We claim that the set

S = {qn | n N}

is infinite. If not, then S = {q1 , q2 , . . . , qk } for some k N. But now, for each

i = 1, 2, . . . , k,

there are at most qi integers such that c


qi [0, 1]. This means that the sequence { pqnn }
has only finitely many terms. As each term of the sequence is rational, the sequence can
therefore not converge to an irrational number. This contradicts the fact that

lim xn = R\Q.
n

So, in view of the set S being infinite, it follows that

1
lim = 0.
n qn

Consequently, if {tn } is a sequence in [0, 1] such that tn , then f (tn ) 0 = f (). So


f is Riemann-integrable on [0, 1], and therefore on [0, x] for all 0 x 1.
Now for any partition P of [0, x], mk (f ) = 0 since between any two reals there is an
irrational number. Thus
x x
f (t)dt = f (t)dt = 0.
0 0

(ii) Let x (0, 1) and h be suciently small so that 0 x + h 1. Then

F (x + h) F (x)
lim = 0.
h0 h

Similarly,
F (0 + h) F (0) F (1 + h) F (1)
lim = 0 = lim .
h0+ h h0 h
This shows that F exists on [0, 1] and in fact F (x) = 0 for all x [0, 1]. So, for x rational,
x = 0, we have F (x) = f (x).

Exercise 8.2.4

1
(a) With reference to the example above, show by some other method that 0 f (t)dt = 0.

(b) Why does F (x) = f (x) in Example 8.2.3 not contradict Theorem 8.2.1?
169 MAT3711/1

(c) The largest integer function is the function [ ] : R Z defined by

[x] = the largest integer less than or equal to x.

For instance, [3] = 3, [3.6] = 3, [5.8] = 6, etc. Now let f : [0, 2] R be given by
f (x) = [x]. Show that f is integrable on [0, 2], but there is no function F such that F = f
on [0, 2].

8.3 Integration and convergence


We saw in Chapter 6 that if {fn } is a sequence of integrable functions that converges pointwise
to an integrable function f , then the sequence of integrals does not have to converge to the
integral of the limit function.
If the convergence is uniform, then the result holds. The proof of this fact is the content of this
section.

Theorem 8.3.1 Let be increasing on [a, b], and let {fn } be a sequence of functions such that
fn R() on [a, b] for each n. Assume that fn f uniformly on [a, b] and define gn : [a, b] R
by
x
gn (x) = fn d.
a

Then we have:
(a) f R() and [a, b], and
x
(b) gn g uniformly on [a, b], where g(x) = a f d.

Proof :

(a) If is constant, the result is immediate. So we may assume (a) < (b). Let > 0 be
given. Choose n0 N such that


n n0 |f (x) fn (x)| < for all x [a, b].
3[(b) (a)]

Then, for any partition P of [a, b], we have


|U (P, f fn0 , )| and |L(P, f fn0 , )| .
3 3

Since fn0 R() on [a, b], there is a partition Q of [a, b] such that


U (Q, fn0 , ) L(Q, fn0 , ) < .
3
170

For this partition Q we have

U (Q, f, ) L(Q, f, ) U (Q, f fn0 , ) L(Q, f fn0 , ) + U (Q, fn0 , )

L(Q, fn0 , )

< |U (Q, f fn0 , )| + |L(Q, f fn0 , )| +
3
.

So f R() on [a, b].

(b) Let > 0 be given and choose n0 N such that


n n0 |fn (t) f (t)| < for all t [a, b].
2[(b) (a)]

If x [a, b] we have

x
|gn (x) g(x)| ||fn f ||d where ||fn f || = sup{|fn (t) f (t)| : a t b}.
a

Thus

|gn (x) g(x)| < for all x [a, b].
2

This proves (b). 

Corollary 8.3.2 Let be increasing on [a, b] and {fn } be a sequence of functions such that

fn R() for each n = 1, 2, . . . on [a, b] and fn = f uniformly on [a, b]. Then we have
n=1
Proof : Apply Theorem 8.3.1 to partial sums. 

The preceding result says a uniformly convergent series can be integrated term by term.

Exercise 8.3.3

(a) Prove Corollary 8.3.2.

(b) Let fn (x) = nx(1 x2 )n on [0, 1]. Show, using integration, that {fn } does not converge
uniformly on [0, 1].
171 MAT3711/1

8.4 Improper integrals


In the previous section we have studied integrals of functions that are bounded on compact
intervals [a, b]. In this section we consider integrals where at least one of these conditions is
relaxed.
We shall not discuss improper integrals at any length, save to show how they are defined. By
this we are by no means suggesting that these kinds of integrals are not important. It so happens
that general integrals (improper or otherwise) are, in our view, best discussed in the context of
Measure Theory; a subject matter beyond the scope of this course.

First we define an improper integral over a finite interval. We restrict the discussion to the
Riemann integral. Let f be a function which is defined for all x such that a < x b. Suppose
b
for each c with a < c b that f is Riemann-integrable over [c, b]. If the limit of c f (x)dx as
c approaches a from the right exists, we say this limit is the improper integral of f over [a, b]
b
and denote it by a f. The improper integral of a function defined on [a, b) is defined analogously.

Next let f be a function mapping [a, ) into R. If f is Riemann-integrable on [a, b] for every
b > a and the limit
b
lim f (x)dx
b a

exists, we say this limit is the improper integral of f on [a, ) and denote it by a f. Thus
b
f = lim f (x)dx
a b a
a
if the limit exists. The improper integral f is defined in a similar way; to wit,
a a
= lim f (x)dx
b b

if the limit exists. Lastly, let f be defined on the whole of R, and suppose f is Riemann-integrable
c
on [a, b] for all a, b R with a < b. If for some c R, f and c f both exist, we define the
improper integral
c
f= f+ f.
c

It should be noted that if f exists, then its value is also equal to the symmetric limit
b
lim f (x)dx.
b b

However, the symmetric limit above might exist even when f does not exist, as the following
example shows.
172

Example 8.4.1 let f (x) = x on R. Then


b
1 2
lim f (x)dx = lim [b b2 ] = 0.
b b b 2
a
On the other hand, for any a R, neither f nor a f exists.

b
If the limit limb f (x)dx exists, it is called the Cauchy principal value of the integral
b

f .

También podría gustarte